Nothing Special   »   [go: up one dir, main page]

Orthobullets HAND

Download as pdf or txt
Download as pdf or txt
You are on page 1of 113

(HAND QUESTIONS) DONE BY: MOHAMMAD HAMADIN

1) (OBQ10.59) Which of the following skeletal dysplasias is caused by a sex linked mutation of the short stature
homeobox (SHOX) gene? QID: 3147
A. Cleidocranial dysplasia
B. Leri-Weil dyschondrosteosis
C. Pseudoachondroplasia
D. Ellis-van Creveld (EVC) syndrome
E. Achondroplasia

PREFERRED RESPONSE ▼ 2
Leri Weill dyschondrosteosis is a skeletal dysplasia characterized by short stature and bilateral Madelung
deformities of the wrist. The SHOX gene is located on the X and Y chromosomes and a mutation on either of the
sex chromosomes leads to the dysplasia (sex linked dominant).

Madelung deformity is a result of disruption of the volar ulnar physis of the distal radius (leading to radial inclination
and a radiopalmar tilt). Symptoms from the wrist deformity include ulnocarpal impaction, restricted forearm rotation,
and median nerve compression.

Cleidocranial dyplasia is an autosomal dominant defect in CBFA-1, a transcription factor that activates osteoblast
differentiation. Pseudoachondroplasia is an autosomal dominant mutation in COMP on chromosome 19. Ellis-van
Creveld (EVC) syndrome is an autosomal recessive mutation in the EVC gene causing acromesomelic shortening
and postaxial polydactyly among other anomalies. Achondroplasia is an autosomal dominant mutation of FGFR-3.

Illustration A demonstrates a Madelung deformity of the wrist with characteristic increased radial inclination and
increased volar tilt.

2) (OBQ05.218) Vitamin B12 deficiency is a known cause of which the following? QID: 1104
A. Inability to whistle
B. Peripheral sensory neuropathy
C. Increased deep tendon reflexes
D. Urinary retention
E. Hydrophobia

PREFERRED RESPONSE ▼ 2
Vitamin B12 deficiency is a known cause of peripheral sensory neuropathy and B12 levels should be evaluated in
patients presenting with peripheral sensory neuropathy. It is associated with decreased deep tendon reflexes,
pathologic reflexes like Babinski's sign, and fatigue/depression. The inability to whistle is associated with
fascioscapular dystrophy. Hydrophobia is associated with rabies infection.

Smith and Singleton evaluated 138 patients referred with predominantly sensory symptoms to identify a
standardized approach to diagnosis. They recommend that patients be tested for glucose tolerance and vitamin
B(12) concentration in all cases, but that other tests should be performed only when the clinical scenario is

1
suggestive.

Steiner et al. describe a case report of a patient with vitamin B12 sensory peripheral neuropathy and associated
EMG evidence of nerve demyelination as the potential cause for the observed clinical symptoms.
3) (OBQ06.208) A 65-year-old female presents with a pulsatile mass in the palm of the hand a few weeks after a
traumatic laceration in a kitchen accident. An MRA of the mass is seen in figure A. What is the most appropriate
treatment? QID: 219

A
A. Aspiration
B. Surgical exploration
C. Compression bandage
D. Steroid injection
E. Strict elevation and observation

PREFERRED RESPONSE ▼ 2
The MRA shown in the figure shows a false aneurysm of the superficial palmar arch. Traumatic palmar artery
aneurysms and pseudoaneurysms are rare and can be caused by penetrating or blunt trauma. The few reports in
the literature all support surgical exploration with either ligation, excision, or repair depending on the extent of the
lesion. The diagnosis is mainly clinical with the pulsatile mass as described by Yajima. Koman recommends a
magnetic resonance angiography if the diagnosis is not clear.
4) (OBQ12.2) A 42-year-old sustains a left finger injury while attempting to catch a baseball for his son. He presents
with left, long finger pain and an inability to extend his middle finger at the distal interphalangeal joint. A radiograph
after closed reduction and splinting is shown in Figure A. What is the best course of treatment? QID: 4362

A
A. Reduction and pinning
B. Repeat splinting of the distal interphalangeal joint in extension
C. Splinting of the distal and proximal interphalangeal joints in extension
D. Observation
E. Fusion of the distal interphalangeal joint

PREFERRED RESPONSE ▼ 1
The radiograph depicts a bony mallet injury with volar subluxation of the distal phalanx after splinting of the DIP joint
in extension, which is an indication for reduction and pinning.

A mallet deformity is caused by disruption of the terminal extensor tendon distal to DIP joint. Occasionally, a bony
2
avulsion of the distal phalanx is noted on radiographs. "Bony" mallet fingers will rarely require surgical fixation. It is
important to attempt to splint a bony mallet injury and get a new radiograph prior to making the decision for
operative treatment. Indications for surgical management of this condition include volar subluxation of the distal
phalanx even after DIP splinting.

Stern et al. found a higher long-term complication rate with surgical treatment of mallet injuries. He also noted 15
degrees more DIP flexion at follow-up in the splinting group compared to the surgical group.

Pegoli et al. describe an extension block technique for treatment of this injury with good results. Their indications for
surgery included the presence of a large bone fragment, and palmar subluxation or the loss of joint congruity of the
distal interphalangeal joint.

Theivendran et al. review the surgical treatment of DIP joint fractures and state that 30% articular involvement is an
indication for operative treatment.

Figure A shows a lateral radiograph with a large intra-articular bony avulsion fragment and volar subluxation of the
distal phalanx.

Incorrect Answers:
Answer 2,3,4: This patient meets the indications for ORIF and nonoperative modalities would not be appropriate.
Answer 5: A DIP fusion in a young patient would not be appropriate.
5) (OBQ04.257) Which of the following peripheral nerve structures functions to cushion the nerve against external
pressure? QID: 1362
A. Endoneurium
B. Fibronectin
C. N-cadherin
D. Epineurium
E. Perineurium

PREFERRED RESPONSE ▼ 4
The epineurium is a supportive sheath surrounding peripheral nerves that cushions fascicles against external
pressure. It is comprised of a loose meshwork of collagen and elastin fibers that are aligned parallel with the nerve
fibers.

Illustration A & B depicts the contents of a nerve including epineurium, perineurium, and endoneurium.

Incorrect Answers:
Answer 1: Endoneurium is a fibrous tissue that covers the axon, Schwann cell, and myelin of each nerve fiber.
Answer 2: Fibronectin and laminin are extracellular matrix glycoproteins that facilitate directional nerve fiber
branching.
Answer 3: N-cadherin is an adhesive membrane glycoproteins on neural ectoderm and facilitate growing axons.
Answer 4: Perineurium is a dense connective tissue which surrounds nerve fascicles. It provides high tensile
strength. The perineurium also limits diffusion within the intraneural environment and subsequently prevents injury
from edema.
Illustrations: A B

3
6) (OBQ13.24) Which statement most accurately describes the physiology of peripheral nerve regeneration
following a axonotmesic lesion? QID: 4659

A. The proximal nerve segment undergoes Wallerian degeneration


B. Axon growth occurs from the distal segment to proximal segment
C. Neurotrophic factors direct phagocytic activity
D. Proximal axon budding allows for antegrade (or distal) axon migration
E. Axoplasm and myelin are degraded distally by Schwann cells

PREFERRED RESPONSE ▼ 4
Axonomesis is a disruption of the nerve axon following injury. Repair/regeneration of the nerve occurs via proximal
budding, followed by antegrade (or distal) axon migration.

The peripheral nerve regeneration process begins with the distal segment undergoing Wallerian degeneration
(axoplasm and myelin are degraded distally by phagocytes). Existing Schwann cells proliferate and line-up along the
basement membrane. Proximal budding occurs after a one-month delay. This is followed by sprouting axons that
migrate in an antegrade fashion to connect to the distal tube. Repair of the nerve can take months, and often have
poor outcomes.

Lee et al. reviewed peripheral never injury and repair. They commented that Wallerian degeneration (i.e.,
breakdown of the axon distal to the site of injury) is initiated 48 to 96 hours after transection. The Schwann cells
then align themselves longitudinally, creating columns of cells called Büngner bands. At the tip of the regenerating
axon is the growth cone.

Illustration A shows a chart of peripheral nerve injury. The two main classification systems are Seddon and
Sunderland. Video V is a lecture discussing peripheral nerve injury and management.

Incorrect Answers:
Answer 1: The distal nerve segment undergoes Wallerian degeneration.
Answer 2: Axon growth occurs from the proximal to distal segment.
Answer 3: Neurotrophic factors do not direct phagocytic activity.
Answer 5: Schwann cells do not degrade axoplasm and myelin.
Illustrations: A V
7) (OBQ07.127) A 20-year-old skateboarder fell 6 months ago and has had radial-sided wrist pain since. His
radiograph upon presentation to your office is shown in figure A. What is the most appropriate treatment at this
time? QID: 788

A
A. four corner fusion
B. long arm thumb spica cast
C. wrist arthroscopy to evaluate intercarpal ligaments
D. open reduction internal fixation with autologous bone graft
E. wrist arthrodesis

4
PREFERRED RESPONSE ▼ 4
This patient has a scaphoid waist fracture nonunion. Several studies indicate that scaphoid nonunions left untreated
have a determined course of collapse and progressive arthritis (scaphoid nonunion advanced collapse - SNAC). Per
Markiewitz et al, the standard treatment of scaphoid nonunions is open reduction internal fixation with bone graft;
non-operative treatment is not appropriate. Proximal row carpectomy and wrist fusion are salvage procedures
reserved for patient that has an advanced scaphoid nonunion, collapse and wrist arthritis.
8) (OBQ04.210) Approximately what percentage of pre-operative grip strength would be expected 3 months after
carpal tunnel release? QID: 1315
A. 10%
B. 25%
C. 50%
D. 100%
E. 150%

PREFERRED RESPONSE ▼ 4
Gellman et al quantified grip and pinch strength post-operatively after carpal tunnel release. They found grip
strength was 28% of preoperative level at 3 weeks; 73% by 6 weeks, returned to the preoperative level by 3 months,
and 116% at 6 months. Pinch strength was 74% of preoperative level at 3 weeks, 96% at 6 weeks, 108% at 3
months, and 126% at 6 months.
9) (OBQ06.242) A 50-year-old woman is diagnosed with carpal tunnel syndrome. She is prescribed a cock-up wrist
splint at 30 degrees of extension to wear at night. This splint has what effect on the carpal tunnel? QID: 253
A. Decreases carpal tunnel pressure
B. Increases carpal tunnel pressure
C. No effect on carpal tunnel pressure
D. Enlarges the carpal tunnel volume
E. Improves nerve conduction studies

PREFERRED RESPONSE ▼ 2
This question is based on the fact that carpal tunnel canal pressure varies with wrist position. Use of neutral wrist
splints for carpal tunnel syndrome is most useful for improving noctural symptoms. The reason for this is the
functional position of the wrist is approximately 30 degrees of extension, and the neutral splints can be functionally
limiting when used during productive daytime hours.

The reference by Gerritsen et al is a randomized controlled study of splinting versus surgery for carpal tunnel. They
found a 80% success rate for surgery at final follow-up versus 54% for splinting at 3 months, which increased to
90% at 18 months for surgery and 75% for splinting.

The reference by Omer is a review of carpal tunnel, and it covers the diagnosis, treatment, and follow-up care of
these patients. They note the need for careful diagnosis to avoid unnecessary or inappropriate surgery.

Weiss et al showed that carpal tunnel pressures are elevated when the wrist is in extension, and are lowest at near
neutral. If one couples this with the inherent tunnel pressure increase from the disease itself, its easy to see that
extension splinting is a double hit and can lead to increased symptoms.

5
10) (OBQ12.135) A 42-year-old male sustains the injury seen in Figure A. Which of the following is true regarding
management of this injury? QID: 4495

A
A. Replantation at this level has a worse functional outcome when compared to above the elbow amputations
B. The veins should be reconstructed prior to the arteries
C. Bony stabilization should occur at the end of the procedure
D. Vein grafting should not be used at this level of amputation
E. A warm ischemia time of 8 hours is a relative contraindication to replantation

PREFERRED RESPONSE ▼ 5
Wrist-proximal amputations should be performed before 12 hours of cold ischemia time or 6 hours of warm ischemia
time have elapsed.
Wrist-proximal replantation should be strongly considered for patients in whom the mechanism allows adequate
debridement, the cold ischemia time is less than 12 hours, and whose general health and comorbidities allow the
patient to tolerate an extended surgical procedure. In general, amputation at the distal forearm and wrist have
excellent functional results with replantation when compared to amputations at other levels. Similar to other major
amputations, replantation should proceed in the following sequence: 1) bone, 2) extensor tendons, 3) flexor
tendons, 4) arteries, 5) nerves, 6) veins (can be done prior to nerve repair) , 7) skin.

Sabapathy et al. review replantation surgery in the upper extremity. They discuss that a “functional extremity” could
be reconstructed at the upper-arm level in 22% to 34%, at the proximal forearm level in 30% to 41%, and at the
distal forearm level in 56% to 80% of cases.

Hanel et al. review wrist level and proximal amputations in the upper extremity. Among other things, they state that
wrist-proximal amputations should be performed before 12 hours of cold ischemia time or 6 hours of warm ischemia
time have elapsed.
Figure A shows a sharp transcarpal amputation.

Incorrect Answers:
Answer 1: Replantation at the distal forearm and wrist have better functional outcomes than above the elbow
replantations.
Answer 2: Arteries should be reconstructed prior to veins.
Answer 3: Bony stabilization should be performed at the beginning of the procedure.
Answer 4: Vein grafting should be used generously.
11) (OBQ06.52) A 22-year-old rugby player presents with a mass at the base of his ring finger 5 months after
sustaining an injury while making a tackle. Physical examination demonstrates a lack of active distal interphalangeal
joint flexion, but full passive range of motion of all joints of the ring finger. Radiographs are normal. What is the most
appropriate treatment to regain normal finger function? QID: 163
A. excision of the palmar mass and 2-stage tendon grafting
B. excision of the palmar mass and single stage tendon grafting
C. excision of the palmar mass and distal interphalangeal joint fusion
D. active silicone rod implantation
E. flexor digitorum profundis repair
6
PREFERRED RESPONSE ▼ 1
This patient has a chronic jersey finger (flexor digitorum profundis avulsion). With the chronicity of the injury, it would
be more amenable to grafting rather than direct repair given retraction of the FDP tendon that occurs with time that
makes direct repair impossible. According to Green’s text, active silicone tendon rod implants have not proven to be
effective. A 2-stage tendon grafting is the treatment of choice in cases of neglected or chronic (>3mo) tendon
injuries or when previous surgery has failed. Two-stage flexor tendon grafting involves implanting a silicone rod
(flexible silicone–Dacron-reinforced gliding implant) in the first stage and a free tendon graft (usually palmaris longus
or plantaris) through the pseudosheath formed around the silicone in the second stage as intitially described by
Hunter and Salisbury in 1971. In single-stage flexor tendon grafting, the tendon graft notoriously adheres to the
surrounding fibro-osseous tunnel significantly limiting range of motion, but in 2-staged tendon grafting the
pseudosheath that is formed around the silicone implant in the first stage greatly reduces the formation of post-
operative adhessions to the tendon graft in the second stage.

Amadio et al showed at 6 month follow-up of staged flexor tendon reconstruction, patients expressed 54% good to
excellent results, but 16% of patients required tenolysis following the second stage of the procedure. LaSalle et al
followed 43 flexor tendon 2 stage reconstructions by comparing passive ROM after stage 1 to postop active ROM
after stage 2. They reported 16% excellent results, 23% good, 26% fair, and 35% poor. They stated that tenolysis
following the second stage improved results on the patients reporting poor outcomes. A distal interphalangeal fusion
would be reserved for failed reconstruction or a patient that does not desire/will not be compliant with likely lengthy
postoperative therapy needed for a staged tendon grafting.
12) (OBQ11.112) Figures A through E depict various conditions affecting the pediatric hand and wrist. For which of
the depicted conditions is temporary scaphotrapeziotrapezoidal pinning most indicated? QID: 3535

A B C D E
A. A
B. B
C. C
D. D
E. E

PREFERRED RESPONSE ▼ 4
Temporary scaphotrapeziotrapezoidal (STT) pinning is indicated for treatment of Kienbocks disease in adolescents
as shown in Figure D. The radiograph shows increased density and slight lunate collapse. The result is a decrease
in radiolunate contact stress while increasing the load on the radioscaphoid articulation. STT pinning is not indicated
in any of the conditions explained below.

Ando et al retrospectively reviewed the results of six adolescents treated with temporary scaphotrapezoidal (ST)
7
pinning. All patients had an increase in wrist flexion/extension arc, strength, and lunate intensity on MRI from their
preoperative baseline.

Shigematsu et al published a case study on a single 11-year-old patient with wrist pain at rest and with use who was
treated with temporary scaphotrapeziotrapedoidal (STT) pinning and cast immobilization for 8 weeks. Both wrist
ROM and grip strength improved. Lunate revascularization was also seen on subsequent MRI.

Incorrect Answers:
Answer 1,2,3: Radial clubhand, scaphoid fracture, and hypoplastic thumb are not treated with temporary
scaphotrapeziotrapezoidal pinning.
Answer 5: Gymnast’s wrist is a distal radius physeal injury due to repetitive axial loading. Plain films will show
physeal widening and hazy irregularity. The condition is not treated with temporary scaphotrapeziotrapezoidal
pinning.
13) (OBQ06.14) A 28-year-old factory worker has his ring finger caught in the machinery at work. A photograph of
the injury is shown in Figure A. Which of the following procedures will best supply coverage of the wound? QID: 25

A
A. Amputation through the proximal interphalangeal joint
B. Shortening of the distal phalanx, nail bed removal, and V-Y flap
C. Cross-finger flap
D. Groin flap
E. Thenar flap

PREFERRED RESPONSE ▼ 2
The clinical presentation is consistent with a transverse fingertip amputation. Shortening of the distal phalanx, nail
bed removal, and dorsal V-Y flap would be the most appropriate treatment.

The V-Y flap is useful for extending dorsal skin to cover a transverse or dorsally angulated fingertip injury. They are
typically used for finger tip amputations which have more dorsal soft tissue loss than palmar loss. Nail bed removal
is important to prevent a subsequent hook nail deformity.

Fassler reviews the proper management of fingertip injuries including the different flap coverage outlines discussed
above. If there is no exposed bone, healing by secondary intention is acceptable. If bone is exposed in the wound
but there is enough nail bed remaining to support a nail plate, then a local advancement flap is a good option. If
local flap coverage is not feasible due to the extent of the soft tissue injury, a regional flap (cross-finger or thenar) is
necessary. If the wound is proximal to the nail bed, then shortening and primary closure is most reasonable.

Figure A shows a transverse fingertip amputation. Illustration A shows the different zones of fingertip amputations:
Zone I is distal to the phalanx; Zone II is distal to the lunula; Zone III is proximal to the lunula. Illustration B shows
the basic steps of a V-Y flap.

8
Incorrect Answers:
There is too much exposed bone remaining to simply cover with sterile dressings.
Answer 1: An amputation would not be indicated.
Answer 3: Cross-finger flaps are useful for fingertip injuries with volar tissue loss only.
Answer 4: A groin flap would not be indicated in this clinical situation.
Answer 5: Thenar flaps are reserved for index and middle fingertip injuries and carry a risk of postoperative flexion
contractures.

14) (OBQ12.26) A 22-year-old college student presents with significant finger pain after coming into contact with
liquid nitrogen in his chemistry lab. A clinical photo of the affected finger in shown in Figure A. What is the most
appropriate next step in treatment? QID: 4386

A
A. Blister debridement and hyperbaric oxygen therapy
B. Drainage of the blister with the overlying skin left intact
C. Full thickness blister and skin debridement with local flap coverage
D. MRI scan of the digit to assess degree of soft tissue damage
E. Wet to dry twice-daily dressing changes to the digit

PREFERRED RESPONSE ▼ 2
The clinical presentation is consistent with a hemorrhagic blister due to acute frostbite injury. Of the options
presented, the most appropriate treatment is drainage of the blister with the overlying skin left intact. Hemorrhagic
blisters represent deeper injuries, and débriding them could lead to desiccation of the underlying dermis.
Alternatively, intact blisters can be left in place and wrapped in dry gauze dressings until they resolve.

Golant et al completed a review article discussing cold exposure injuries to the extremities. They state that frostbite,
the most serious peripheral injury, results in tissue necrosis from direct cellular damage and indirect damage
secondary to vasospasm and arterial thromboses. With regards to treatment, the authors conclude that débridement
of necrotic tissues is generally delayed until there is a clear demarcation from viable tissues, a process that usually
takes from 1 to 3 months from the time of initial exposure. They advocate drainage of hemorrhagic blisters, leaving
the overlying skin intact.

Bruen et al review the treatment of digital frostbite in their current concepts review. They state that physical
examination results that are concerning for severe upper-limb injury include the absence of Doppler pulse signals,
9
absent capillary refill, dark purple discoloration of the digits, and hemorrhagic blisters. They state that intact blisters
should left in place and wrapped in dry gauze dressings until they resolve. These findings should lead to further
perfusion evaluation with technetium-99m triple-phase bone scanning or angiography.

Incorrect Answers:
Answer 1: Hyperbaric oxygen therapy is not indicated as initial treatment of frostbite injury.
Answer 3: Full thickness skin débridement should be delayed until full demarcation of the injury is determined. This
can take as long as three months in some cases.
Answer 4: MRI scan is not indicated initially.
Answer 5: Wet to dry dressing changes are most beneficial in the presence of an open wound, as removal of the
dressing when it dries removes drainage and debris. This treatment would not be beneficial in the presence of a
blister with the overlying skin intact.
15) (OBQ07.218) A 27-year-old man falls on his hand at work. He notices an immediate deformity of his ring finger.
Radiographs are provided in Figure A. Which of the following is the most appropriate initial treatment?QID: 879

A
A. Closed reduction, buddy taping, and early motion to prevent stiffness
B. Closed reduction and full time extension splinting
C. Open reduction and repair of the central slip of the extensor tendon
D. Open reduction and repair of the volar plate
E. Amputation and immediate return to play

PREFERRED RESPONSE ▼ 2
The radiograph demonstrates a volar PIP dislocation. The central slip of the extensor tendon is frequently ruptured
and will lead to a boutonneire deformity if left untreated. The PIP must be immobilized in extension to allow the
extensor mechanism to heal. Immobilization in extension should be maintained for 6 weeks to allow soft tissue
healing. Open reduction and repair of the central slip would be the appropriate treatment for a developing
boutonneire deformity that presents in a subacute or chronic time basis. Illustrations A and B demonstrate a
schematic and clinical photo of central slip disruption and secondary deformity with PIP flexion and DIP
hyperextension (Boutonniere Deformity). Posner et al reviewed 7 patients with chronic palmar dislocations of the
PIP joint who were treated with open reduction and reconstruction of the extensor mechanism. All patients acheived
satisfactory range of motion and the authors concluded that this technique is preferable to arthrodesis. Peimer et al
reviewed 15 patients with palmar dislocations of the PIP joint. Twelve of the fifteen were evaluated on a delayed
basis (average 11 weeks following injury) and underwent open reduction and surgical repair of the extensor tendon.
Three of the fifteen were seen earlier following injury and were treated with closed reduction and pinning. All fifteen
patients acheived satisfactory clinical outcomes although finger range of motion was not fully recovered in any case.
Illustrations: A B

10
16) (OBQ09.122) A 60-year-old man has chronic pain at the base of this thumb and weakness on attempted thumb
pinch. A radiograph is shown in Figure A. Which injection would likely reduce his pain and increase his function?

A
A. Saline
B. Steroid
C. Hylan
D. All of the above are equally effective
E. All of the above are detrimental

PREFERRED RESPONSE ▼ 4
The patient has basal joint arthritis of the thumb and randomized controlled trials have failed to demonstrate an
advantage of steroid or hylan over saline.

Heyworth et al demonstrated that all three injections were similarly effective for approximately 3 months at reducing
pain and increasing thumb function over baseline levels.

Stahl found that steroid and hylan were equally effective, but did not control with saline.

Henderson found no advantage of hyaluronan over saline for knee arthritis during a 5 week treatment course.
17) (OBQ11.166) A 25-year-old right-hand dominant construction worker suffers an industrial injury as seen below.
He is hemodynamically stable and his only injury is to the limb below. In terms of replantation of the affected limb,
which of the following is true? QID: 3589

A B C
A. Transpositional microsurgery offers the best results
B. Replantation is contraindicated as the injury is through flexor zone II
C. Replantation is contraindicated because of the extent of injury
D. Heterotopic transplantation would offer the best function
E. Anatomic replantation of hand offers the best results

PREFERRED RESPONSE ▼ 5
The clinical scenario and images depict a through the palm amputation with the digits intact. The injury is a clean
amputation with minimal avulsion. Thus, anatomic replantation of the entire hand is indicated. Digit transposition

11
refers to using the salvageable digits and replanting them on the functionally important positions in the hand.
Transpositional replantation is not indicated in the above scenario and would be more appropriate for a multidigit
amputation as shown in Illustration A.

Soucacos et al reviewed their results of transpositional digital microsurgery in 34 patients. They found that
transposition of a digit to the most functional part of the hand lead to a 2-point discrimination of 10-14mm in
transposed digits and equivalent functionality of transposed digits with anatomically replanted digits.

Schwabegger et al presented a case series of 13 patients with multiple digit amputations. The primary goal of
surgery was function and secondly, cosmesis. They found the results of transpositional replantation similar to
conventional replantation.

18) (OBQ05.273) Which of the following structures is an anatomical component of the triangular fibrocartilage
complex? QID: 1159
A. Extensor carpi ulnaris tendon sheath
B. Lunotriquetral interosseous ligament
C. Extensor digiti minimi tendon sheath
D. Radioscaphocapitate ligament
E. Flexor carpi ulnaris tendon sheath

PREFERRED RESPONSE ▼ 1
Palmer et al studied the anatomy and function of the triangular fibrocartilage complex (TFCC) through anatomical
dissections and biomechanical testing. The TFCC was found to be composed of the sheath of the extensor carpi
ulnaris (ECU), an articular disc, the dorsal and volar radioulnar ligaments, the meniscus homologue, and the ulnar
collateral ligament. Biomechanically, they determined that the TFCC functions as a cushion at the ulnocarpal
interface, and is a major stabilizer of the DRUJ. Nakamura et al histologically examined the origins and insertions of
the TFCC in fresh-frozen cadaver wrists. They found that the floor of the ECU sheath originated from the dorsal side
of the fovea of the ulna, through an arrangement of Sharpey's fibers. Illustration A shows the anatomy of the TFCC.

12
19) (OBQ05.55) All of the following are predictive findings for correctly diagnosing carpal tunnel syndrome
EXCEPT: QID: 941
A. Abnormal hand diagram
B. Abnormal Semmes-Weinstein testing in wrist-neutral position
C. Positive median nerve compression test (Durkan's sign)
D. Presence of night pain
E. Loss of small digit adduction (Wartenberg sign)

PREFERRED RESPONSE ▼ 5
All of the listed physical exam findings, except for loss of small digit adduction (Wartenberg sign), has been found to
be predictive for diagnosing carpal tunnel syndrome.

Szabo et al in a Level 3 study used a regression model to analyze the most predictive factors for correctly
diagnosing carpal tunnel syndrome (CTS). Their analysis found that with an abnormal hand diagram, abnormal
sensibility by Semmes-Weinstein testing in wrist-neutral position, a positive Durkan's test, and night pain, the
probability that carpal tunnel syndrome will be correctly diagnosed is 0.86. They found the tests with the highest
sensitivity were Durkan's compression test (89%), Semmes-Weinstein testing after Phalen's maneuver (83%), and
hand diagram scores (76%). Night pain was a sensitive symptom predictor (96%). The most specific tests were the
hand diagram (76%) and Tinel's sign (71%). The authors concluded that the addition of electrodiagnostic tests did
not increase the diagnostic power of the combination of these 4 clinical tests, and proceeding with surgical release
is appropriate even if the EMG is normal.

Wartenberg sign is persistent abduction and extension of the small digit when a patient is asked to adduct the digits
and is seen in cubital tunnel syndrome, but not carpal tunnel syndrome.

Illustration V demonstrate the Durkan's Compression test for carpal tunnel syndrome.
Illustrations: A V
20) (OBQ07.92) A 64-year-old female presents with pain in the base of her thumb and a positive grind test. A
radiograph is shown in Figure A. Which of the following surgical steps is the most crucial in surgical treatment of this
condition to achieve a successful clinical outcome? QID: 753

A
A. Suspension of the first metacarpal
B. Deep dissection of thenar/extensor compartments
C. Interposition material within CMC joint
D. Excision of the trapezium
E. Alignment of the skin incision

PREFERRED RESPONSE ▼ 4
This patient is presenting with signs/symptoms consistent with 1st CMC arthritis. The most important step in a 1st
CMC arthroplasty for basilar thumb arthritis is excision of trapezium.

The various surgical options that have been discussed in the literature include excision of the trapezium, excision of

13
the trapezium with interposition arthroplasty and suspensory ligament reconstruction, and excision with interposition
arthroplasty only.

Davis et al showed that the outcome of three common CMC arthroplasty procedures were similiar as long as
trapeziectomy was included.
21) (OBQ06.231) All of the following are considered the cardinal signs of flexor tenosynovitis EXCEPT: QID: 242
A. Tenderness along the flexor tendon sheath
B. Flexed resting posture of the finger
C. Fusiform swelling of the finger
D. Pain on passive extension of the finger
E. Pain on passive flexion of the finger

PREFERRED RESPONSE ▼ 5
The cardinal signs of pyogenic flexor tenosynovitis (Kanavel signs) include:
1) partially flexed resting posture of the involved finger
2) pain with passive extension
3) fusiform swelling of the finger
4) volar tenderness along the flexor sheath

Patients commonly present 24 to 48 hours after onset of symptoms. The standard of care is “urgent surgical
drainage” to avoid tendon scarring or necrosis with subsequent impairment of finger function followed by
intravenous antibiotic administration.

According to Hand Surgery Update 3, open sheath irrigation has been replaced largely by closed sheath irrigation.
These authors cite a retrospective study that showed no statistical difference in resolution of infection using open
sheath irrigation or closed sheath irrigation, however, there was a trend towards more frequent complications and
reoperations in the open drainage group.

Lille et al reviewed the records of 75 patients with pyogenic flexor tenosynovitis and found that there was no
difference in outcomes between those who received intraoperative irrigation only versus those receiving
intraoperative irrigation and continuous postoperative irrigation.
22) (OBQ10.213) Creation of a Stener lesion, as found in Gamekeeper's thumb, requires combined tears of the
proper and accessory ulnar collateral ligaments in order for the ligament to be displaced by the adductor
aponeurosis. Which of the following most accurately describes the role these ulnar collateral ligaments (PCL/ACL)
play in thumb MCP joint stability? QID: 3306

A. PCL is primary restraint to radial deviation with MCPJ in flexion, ACL provides restraint to radial deviation with
MCPJ in extension
B. PCL is primary restraint to radial deviation with MCPJ in extension, ACL provides restraint to radial deviation with
MCPJ in extension
C. ACL is primary restraint to ulnar deviation with MCPJ in flexion, PCL provides restraint to ulnar deviation with
MCPJ in extension
D. ACL is primary restraint to radial deviation with MCPJ in flexion, PCL provides restraint to radial deviation with
MCPJ in extension
E. PCL is primary restraint to ulnar deviation with MCPJ in flexion, ACL provides restraint to radial deviation with
MCPJ in extension

PREFERRED RESPONSE ▼ 1
The proper ulnar collateral ligament(PCL) runs from the metacarpal head to the volar aspect of proximal phalanx
and resists ulnar stress with the thumb MCPJ in flexion. The accessory ulnar collateral ligament(ACL) lies palmar to
14
the proper ligament, and inserts onto the volar plate. The volar plate and ACL function as the principle restraints to
ulnar stress with the thumb MCPJ in extension.

The function of the ulnar collateral ligaments is shown in Illustration A.(Please note the distal phalanx of the thumb
has been removed in Illustration A.) A Stener lesion is described by displacement of the distal end of the completely
ruptured UCL such that it comes to lie superficial and proximal to the adductor aponeurosis. This is shown in
Illustration B.

Thrikannad and Wolff report a case of distal pull-off of the ulnar collateral ligament (UCL) of the thumb MCPJ with
two fracture fragments. They identify the need to look for a second fragment of bone in these injuries, where an
apparently undisplaced fracture is noted at the base of the proximal phalanx. They suggest that this second
fragment probably indicates the location of the distal end of the UCL and may identify a Stener lesion. A
radiographic example from their paper is shown in Illustration C.

Newland, in his review article on Gamekeeper's Thumb, states that criteria for judging what constitutes a complete
tear vary from 15 deg to 45 deg difference with respect to the opposite side. He goes on to state, however, that
many authors choose an absolute value of >35 degrees of joint laxity compared to the contralateral side when
judging a tear to be complete or incomplete. When an acute tear is identified, surgical repair is recommended.

23) (OBQ07.24) A 20-year-old football player presents with a one week history of right index finger pain which
started after his hand got caught in a face mask during a tackle. Physical exam shows swelling of the digit with no
breaks in the skin, and no active flexion. AP, lateral, and oblique radiographs are provided in Figures A, B, and C
respectively. Which of the following structures most often prevents closed reduction of this injury? QID: 685

A B C
A. Volar plate
B. Collateral ligaments
C. FDP tendon
D. Central slip
E. Dorsal capsule

PREFERRED RESPONSE ▼ 1

15
Figures A, B, and C demonstrate a dorsal dislocation of the DIP joint without associated fracture.

Abouzahr et al conducted a case report and literature review on irreducible dorsal DIP dislocations. The authors
found that the most common block to reduction with a closed injury is an interposed volar plate, which is avulsed
from its origin on the middle phalanx. They recommened open reduction and extraction of the volar plate if one is
unable to achieve concentric stable reduction after two attempts. Furthermore, the authors determined that in open
injuries, the FDP tendon is primarily responsible for irreducibility. The collateral ligaments are less likely to be
involved in this case because there is little coronal deformity present. The dorsal capsule is typically not a block to
reduction, and the central slip is disrupted (but does not block reduction) in volar PIP joint dislocations.
24) (OBQ04.133) All of the following have been implicated in the pathogenesis of Duputryen's contracture
EXCEPT? QID: 1238
A. Fibroblast growth factor (FGF)
B. Transforming growth factor- beta (TGF-beta)
C. Myofibroblasts
D. Platelet-derived growth factor (PDGF)
E. CBFA-1

PREFERRED RESPONSE ▼ 5
Of the answers listed, only CBFA-1 has NO known role in the pathogenesis of Duputryen's contracture.

Dupuytren contracture, a disease of the palmar fascia, results in the thickening and shortening of fibrous bands in
the hands and fingers. The offending cells are thought to be myofibroblasts and fibroblasts. Growth factors such as
basic fibroblast growth factor (FGF), platelet-derived growth factor (PDGF), and transforming growth factor-beta
(TGF-beta) may signal the overproduction of the myofibroblasts and/or myofibroblastic activity of the fibroblasts. In
addition, high levels of TGF-Beta may hinder apoptosis of the active myofibroblasts, unlike normal tissue healing.

McGrouther discusses how the pathophysiology of Dupuytren's is related to the anatomy of the palmar ligaments.
The article discusses the proposed etiology of Dupuytren's with a loss of normal motion between palmar fascial
ligaments causes stress concentrations which stimulate fibrous tissue deposition and contracture.

Baird et al performed a tissue analysis of 12 patients with Dupuytren's contracture compared to 12 control patients.
They found that Dupuytren's expressed a higher percentage of peptide regulatory factors including interleukin-1
alpha, interleukin-1 beta, transforming growth factor beta, and basic fibroblast growth factor.
25) (OBQ11.74) A 7-year-old boy sustains a ring finger injury after falling from his bike. The fingernail has been torn
transversely beneath the eponychium and the surgeon has removed the nail as shown in Figure A. Radiographs are
shown in Figure B. What is the next best step in management? QID: 3497

A B
A. Irrigation and debridement with alumafoam placement and immobilization
B. Irrigation and debridement followed by percutaneous pinning and immobilization
C. Irrigation and debridement followed by reduction, nail bed repair and immobilization
D. Betadine soaks at home three times daily with intermittent alumafoam splint placement and immobilization
E. Alumafoam splint placement and immobilization
16
PREFERRED RESPONSE ▼ 3
The clinical presentation is consistent with a physeal separation and a nail bed injury. This is also called a Seymour
fracture which is a juxta-epiphyseal fracture of the distal phalanx. Treatment of a nail bed avulsion and physeal
separation is irrigation and debridement, physeal reduction, nail bed repair and immobilization. The primary goals
are to achieve a stable, viable nail and good cosmetic results.

Inglefield at al retrospectively reviewed 19 children with 22 nail bed injuries. Early operative repair led to good to
excellent results in 91% of patients. They concluded that repair of the nail bed at the time of injury is superior to
secondary correction.

Fassler reviewed fingertip injuries, providing recommendations for treatment based on degree of soft tissue loss,
bone exposure, feasibility for flap coverage and the presence or absence of mitigating systemic conditions. He also
concluded that the outcome of nail bed injuries is dependent on the severity of injury to the germinal matrix.

Illustration A shows the makeup of the terminal phalanx. Illustration B and C show a Seymour Fracture before and
after irrigation and debridement and reduction.

26) (OBQ07.257) Each of the following are indications for microvascular replantation EXCEPT? QID: 918
A. Thumb amputation
B. Index finger amputation in a child
C. Through wrist amputation
D. Long finger amputation through the proximal phalanx
E. Mid-palm amputation of all four fingers

PREFERRED RESPONSE ▼ 4
As reviewed by Soucacos, there are several major indications for single digit replantation: 1) Level of the amputation
is distal to the insertion of FDS. 2) Amputations at the level of the distal phalanx. 3) Ring avulsion injuries involving
both the dorsal and palmar skin and blood supply in an isolated finger, as long as FDS is intact. 4) Any amputation
in a child. 5) Thumb amputation. Replantation of a single digit, which is amputated at the level of the proximal
phalanx or at the PIP joint, particularly in avulsion or crush injury is contra-indicated. Soucacos also discusses
appropriate surgical teams, transport, and other related issues surrounding a "transplant team."

17
27) (OBQ11.83) A 46-year-old homeless IV drug abuser presents with the hand infection shown in Figure A, which
developed after sustaining a superficial laceration. Cultures are taken during operative irrigation and debridement,
and he is started on antibiotic therapy. Based on the patients history, what is the most common pathogen in this
setting? QID: 3506

A
A. Herpes simplex virus
B. Candida albicans
C. Escherichia coli
D. Eikenella corrodens
E. Methicillin-resistant staphylococcus aureus

PREFERRED RESPONSE ▼ 5
Figure A shows an abscess over the metacarpophalangeal joint of the thumb. Infections with these characteristics in
IV drug abusers are most commonly caused by MRSA, and can affect any portion of the hand.

Imahara et al retrospectively reviewed 159 hand infections treated in the operating room over an 11-year period.
The examined data included known risk factors for MRSA, including human immunodeficiency virus infection,
diabetes mellitus, intravenous drug use, incarceration, and homelessness. Intravenous drug use was the only
independent risk factor for CA-MRSA infections.

Boucher et al examined the trends in both nosocomial and community-associated MRSA infections and explored
recent studies of the mechanisms that allow S. aureus to become resistant to currently available drugs.

Incorrect Answers:
1-Herpes simplex virus can cause Herpetic whitlow, as shown in Illustration A, typically presents on the fingers
health care workers exposed to a carriers mouth. Usually, this infection appears as small ulcers or vesicles, and
operative debridement is contraindicated.
2-Candida albicans is a more rare hand infection typically associated with chronic paronychia, as shown in
Illustration B.
3-Escherichia coli is a less common cause of abscess formation in the hand.
4-Eikenella is usually associated with "fight-bite" infections on the dorsal aspect of the MCP joint, and does not
commonly occur after superficial lacerations. It can also rarely occur in IV drug users who clean their needles with
saliva, as Eikenella is part of the normal oral flora. An example of an Eikenella infection is shown in Illustration C.
Illustrations: A B C +

18
28) (OBQ12.163) A 65-year-old man fell and injured his right wrist. Radiographs taken in the emergency room are
seen in Figure A. He was treated as a sprain and no further follow-up was planned. He sustained 2 minor falls over
the next 6 years and his wrist pain recurred. Recent radiographs are seen in Figure B. Surgical treatment that will
best address his symptoms and preserve wrist motion consists of QID: 4523

A B
A. Anterior and posterior interosseous neurectomy
B. Scaphotrapezialtrapezoidal (STT) fusion
C. Complete wrist arthrodesis
D. Proximal row carpectomy
E. Four-corner fusion with scaphoidectomy

PREFERRED RESPONSE ▼ 5
Four-corner fusion with scaphoidectomy is indicated for Stage III SLAC wrist.

Surgical treatment of SLAC wrist is stage dependent. Stage I disease (scaphoid-radial styloid arthritis) is treated
with AIN/PIN neurectomy. This procedure can also be done in addition to other bony procedures for Stages II-III
disease. Stage II (scaphoid-entire scaphoid facet) is treated with PRC or scaphoid excision with 4-corner fusion
(4CF). Stage III (capitolunate arthritis with proximal migration of the capitate into the scapholunate interval) is
treated with either scaphoidectomy with 4CF or total wrist fusion.

Some other conditions exist: If capitolunate arthritis exists, PRC is contraindicated and 4CF is performed. If
radiolunate arthritis exists, both PRC and 4CF are contraindicated and total wrist fusion is performed. If both
radiolunate and capitolunate surfaces are preserved, then either PRC or a 4CF may be performed.

Cohen et al. compare PRC with 4-corner fusion plus scaphoid excision. PRC is technically easier, but leads to
shortening of the carpus with weakness and incongruity exists between the capitate and lunate fossa of the distal
radius. Scaphoid excision and four-corner fusion maintains carpal height and preserves the radiolunate relationship,
but is more technically demanding, there is risk of nonunion, and it requires longer postop immobilization. Pain relief
is more reliable following 4-corner fusion.

Figure A shows scapholunate ligament disruption. Figure B shows late stage SLAC wrist. There is capitolunate
arthritis but no radiolunate arthritis. Illustration A shows an example of PRC. Illustration B shows an example of 4CF
and scaphoidectomy.

Incorrect Answers
Answer 1. Neurectomy of AIN and PIN is performed for Stage I disease and can also be done in addition to other
bony procedures for Stages II-III.
Answer 2. STT fusion is indicated for chronic scapholunate instability, STT arthritis and Kienbock's disease. It is not
19
appropriate for Stage III SLAC wrist as it does not address capitolunate arthritis.
Answer 3. Complete wrist arthrodesis is indicated for pancarpal arthritis in a young patient. It is less appropriate for
this 71-year-old patient. It sacrifices wrist motion. Wrist arthrodesis would be performed if BOTH capitolunate and
radiolunate arthritis were present
Answer 4. Proximal row carpectomy is indicated for Stage II disease. It is contraindicated where capitolunate
arthritis is present (Stage III).
Illustrations: A B
29) (OBQ04.71) A 54-year-old female presents with a hand deformity. A surgical procedure is being considered that
relocates the lateral bands dorsally to counteract the pathophysiology of the deformity. Which of the following
deformities does this patient most likely have? QID: 1176
A. Boutonneire finger deformity
B. Lumbrical plus finger deformity
C. Mallet finger deformity
D. Jersey finger deformity
E. Swan neck finger defomity

PREFERRED RESPONSE ▼ 1
Boutonniere deformity is characterized with the PIP in flexion and the DIP in hyperextension as shown in Illustration
A. It is caused by central slip rupture or attenuation (secondary to capsular distention, e.g., rheumatoid arthritis),
laceration, or traumatic disruption. Volar subluxation of the lateral bands due to incompetence or disruption of the
triangular ligaments leads to increased deformity as the lateral bands become flexors of the PIP. Relocation of the
lateral bands to their original dorsal position to counteract the pathophysiology of the deformity is an option for
patients that have an approximately 40 degree active flexion contracture but full passive extension.

Illustrations: A

30) (OBQ05.239) A 34-year-old male undergoes a thumb replantation after an industrial meat slicer accident. At 4
hours postoperatively there is a drop from 33 degrees celsius to 29 degrees celsius and the pulse oximetry monitor
on the thumb reads 87%. All of the following are treatment options for the management of his arterial inflow
insufficiency EXCEPT: QID: 1125
A. Administer IV heparin
B. Administer a stellate ganglion block
C. Placement of leeches on the thumb
D. Inspect the dressing for any constriction
E. Place the thumb in a dependent position.

PREFERRED RESPONSE ▼ 3
Arterial thrombosis after digit replantation typically occurs within the first 12 hours postoperatively whereas venous
thrombosis/congestion occurs after the first 12 hours postoperatively. Leeches excrete Hirudin, which is 100 times
more potent than heparin, but are typically used for the treatment of venous thrombosis/congestion and not arterial
thrombosis. Miller's review states a drop in temperature >2 C in 1 hour or temperature below 30 C indicates
decreased digital perfusion. If arterial insufficiency develops: release constrictive bandages, place the extremity in a
dependent position, consider heparinization, consider stellate ganglion blockade, or explore early if these
maneuvers do not work.

20
31) (OBQ04.21) A 24-year-old professional baseball outfielder reports persistent pain in the hypothenar region when
batting for the past year. His CT scan is shown in Figure A. What is the recommended treatment?QID: 132

A
A. pisiform excision
B. hook of hamate excision
C. carpal tunnel release
D. decompression of Guyon's canal
E. open reduction and internal fixation

PREFERRED RESPONSE ▼ 2
The history is typical of a hook of the hamate fracture, which is confirmed on the CT image. A carpal tunnel view
radiograph of this injury is shown in Illustration A. It commonly occurs in baseball players and golfers. Physical exam
findings include point tenderness at the hamate, ulnar nerve paresthesias (hemorrhage within Guyon's canal), and
pain with axial load of ring and little fingers. For cases seen late, with few exceptions, the recommended treatment
has been excision of the hook fragment. Marchessault provides a review of diagnoses and treatment for carpal
fractures. They discuss the treatment of these injuries, indicating that acute, nondisplaced fractures may be placed
in a cast, and excised if nonunion develops. The authors go on to say that certain investigators recommend excision
of asymptomatic nonunions to minimize the risk for flexor tendon rupture.
Illustrations: A
32) (OBQ11.8) Figure A depicts a child with a congenital abnormality. Which of the following is true regarding this
condition? QID: 3431

A
A. Circumferential trunk involvement is more common than distal extremities involvement
B. Risk factors include late gestation (>44 weeks) and high birth weight (>3500g)
C. Incomplete circumferential bands not directly interfering with lymphatic circulation should be resected
D. There is a strong correlation with anterolateral tibial bowing
E. Complete circumferential bands that interfere with lymphatic drainage can be treated with band excision and z-
plasty.

21
PREFERRED RESPONSE ▼ 5
The image and vignette describe a patient with constriction band syndrome (CBS). In the case of lymphatic
obstruction or vascular compromise, the treatment of CBS is band excision. There are many terms used to describe
this phenomenon. However, the etiology is the entanglement of fetal parts in the amniotic membrane.

Foulkes et al reviewed 71 cases of congenital constriction band syndrome (CCBS). They found the average patient
had three involved limbs, with a predilection for distal, central digits of the upper extremity. There was a strong
correlation with abnormal gestation and clubfoot. Treatment included distraction osteogenesis and free
osteocutaneous transfer.

Goldfarb et al reviewed amniotic constriction band syndrome (ABS), highlighting its association with annular
constriction of multiple extremities. They classified ABS into classic (disruptions and deformations) and non-classic
(malformations). ABS is due to disruptions (amputations, acrosyndactyly), deformations (oligohydraminos, scoliosis,
talipes equinovarus) and malformations (body-wall defects, cleft lip/palate). As there is moderate overlap between
the classic and nonclassic, additional research into the underlying cause is being investigated.

Green described a one-stage release of circumferential constriction bands in three patients. The advantages of this
technique are the decreased need for anesthesia and subsequent procedures as well as facilitating postoperative
care.

Kawakura et al reviewed the intrinsic and extrinsic theories of (CBS). The most common manifestations are distal
extremity involvement, intrauterine amputations and acrosyndactyly. Excision of bands and mobilization of
subcutaneous adipose tissue as described by Upton is seen in Illustration A.

Incorrect Answers:

Answer 1: Distal extremities are more affected than the trunk


Answer 2: Risk factors include low birth weight (<2500g), prematurity (<37wks), maternal drug exposure, trauma
during pregnancy and attempted fetal termination during the first trimester.
Answer 3: Shallow bands that do not interfere with circulation or lymphatic drainage do not need to be released.
Answer 4: There is a strong correlation with clubfoot, not anterolateral bowing.
Illustrations: A
33) (OBQ13.48) A 36-year-male was using a high-pressure paint gun when he suffered the injury shown in Figure A.
Which of the following variables would have the worst impact on his prognosis? QID: 4683

A
A. Delay in surgical treatment
B. Injected solvent was grease
C. Injected solvent was water-based paint
D. An entry wound of greater than 3 cm
E. Injected solvent was at room temperature
22
PREFERRED RESPONSE ▼ 1
The clinical presentation is consistent for a high-pressure injection injury. Delays in surgical treatment are
associated with serious sequelae.

High-pressure injection injuries are characterized by extensive soft tissue damage associated with a benign high-
pressure entry wound. They should be treated with irrigation & debridement, foreign body removal and broad-
spectrum antibiotics. There is a higher rates of amputation when surgery is delayed.

Bekler et al. looked at the results of 14 surgically treated high-pressure injection injuries of the hand with a minimum
of two years follow-up. Ten of the injuries required formal operative debridement and foreign body removal. Six
required reconstructive microsurgical procedures and one underwent digital tip amputation. They concluded that
high-pressure injection injury to the hand is a significant problem, which can easily lead to serious sequelae and,
even, amputation.

Rosenwaser et al. report wide débridement of all involved tissues, decompression of tissue compartments,
exploration and incision of tendon sheaths, removal of injected material, and saline irrigation are critical in the
management of high-pressure injection injuries to the hand. They emphasize delayed surgery has been associated
with increased incidence of morbidity and amputation.

Figure A shows a typical high-pressure injection injury. Notice the benign looking entry wound.

Incorrect Answers:
Answer 2: Grease as an injected solvent has a more favorable prognosis when compared to industrial solvents & oil
based paints.
Answer 3: Water-based paint as an injected solvent has a more favorable prognosis when compared to industrial
solvents & oil based paints.
Answer 4: The size of the entry wound does not have a strong correlation with the severity of injury. Often times
these injuries have a benign looking entry wound.
Answer 5: Injected solvent at high temperatures are associated with a worse prognosis.
34) (OBQ09.14) Repair of a nailbed injury with 2-octylcyanoacrylate (Dermabond) provides what distinct advantage
over standard suture repair? QID: 2827
A. Better ultimate cosmesis
B. Better functional outcome
C. Faster procedure
D. Less pain at follow up
E. Lower infection rate

PREFERRED RESPONSE ▼ 3
Octylcyanoacrylate (Dermabond) has been found to be a viable method in nailbed repair, and has the advantage of
being a faster procedure.

Strauss et al performed a level 1 randomized trial of 2-octylcyanoacrylate (Dermabond) versus suture repair of
nailbed injuries. They found the only significant difference was that 2-octylcyanoacrylate had a shorter procedure
time. All other indices were similar.

Richards et al provide a description of their technique using dermabond to secure the nail following a nailbed repair.
They found good results with no complications in their cohort of 22 patients, and recommend it as a technique.

Illustration A shows a nail bed bed repair.


23
Incorrect Answers
Answer 1,2,4,5: There is no difference in these outcomes when comparing 2-octylcyanoacrylate (Dermabond)
versus suture repair

35) (OBQ04.86) Which of following most appropriately details the anatomic orientation of the digital nerve and artery
in the finger? QID: 1191
A. Artery palmar and radial to nerve
B. Nerve dorsal and ulnar to artery
C. Nerve medial to artery
D. Artery ulnar to nerve
E. Artery dorsal to nerve

PREFERRED RESPONSE ▼ 5
The digital nerve is palmar to the artery in the finger. A helpful way of remembering this orientation is that sensation
is performed with the pads (nerve is palmar) of your fingers and you test for cap refill at the fingernail (artery is
dorsal).

Illustration A shows a cross section of the finger and the relative position of the artery and the nerve. Illustrations B
and C depict cross-sectional anatomy of the finger including Grayson's (involved in Dupuytrens) and Cleland's
ligament (not involved in Dupuytren's)
Illustrations: A B C
36) (OBQ05.193) All of the following factors are favorable for digit survival after replantation surgery EXCEPT? 1079

A. Sharply amputated digit


B. Crushed amputated digit
C. Warm ischemia time of 8 hours
D. Patient age of 10 years
E. Female gender

PREFERRED RESPONSE ▼ 2
Waikakul et al. in a study of 1018 replantations found that type of injury was the most important factor influencing
immediate and late outcome. Extensively crushed injuries had the worst outcome, followed by degloving and
avulsion injuries. Sharp cut injuries fared the best. Regular cigarette smoking resulted in poor immediate survival
rate and prolonged ischemia had a significant influence in final functional outcome, but neither were as important as
mechanism of injury. Alcohol consumption was also a negative predictor. Favorable factors for replantation survival
were female gender, age under 13 years old, and nonsmokers. Regarding ischemia time, Miller recommends <12
hours of warm ischemia or <24 hours of cold ischemia for a digit to obtain optimal outcomes.

24
37) (OBQ04.65) A 25-year-old left hand dominant musician sustains an injury to the left thumb shown in Figure A.
He is unable to extend the interphalangeal joint and has less than 2 second capillary refill at the thumb. What is the
most appropriate method to achieve soft tissue coverage after extensor tendon repair or transfer? QID: 1170

A
A. Moberg advancement flap
B. Vacuum-assisted wound closure
C. Wet-to-dry dressings
D. First dorsal metacarpal artery (Kite) flap
E. V-Y advancement flap

PREFERRED RESPONSE ▼ 4
The clinical scenario is consistent with a dorsal thumb avulsion with missing extensor tendon and exposed bone
necessitating soft tissue coverage. The first dorsal metacarpal artery (Kite) flap is the most appropriate flap for
defects of the dorsal aspect of the thumb.

Fassler et al in a Level 5 review state that the first dorsal metacarpal artery (Kite) flap is appropriate for defects of
the dorsal aspect of thumb. The flap is performed in one stage with the skin over the dorsum of the proximal index
finger elevated with incisions on all four sides. An incision is extended proximally over the dorsum of the first web
space, and a pedicle containing the first dorsal metacarpal artery, the subcutaneous veins, and branches of the
dorsal sensory branch of the radial nerve is isolated. The skin island with the attached pedicle is transferred to the
thumb defect and sutured in place.

Illustration A shows the technical steps of the first dorsal metacarpal artery (Kite) flap. Illustration B shows the final
functional results of the first dorsal metacarpal artery (Kite) flap are shown in Illustration B.

Incorrect Answers:
Answer 1: Moberg advancement flaps are indicated for volar thumb defects.
Answer 2 & 3: Wet to dry dressings or vaccuum-assisted wound closure would be inappropriate in this situation.
Answer 5: V-Y advancement flaps are most appropriate for transverse or dorsal oblique fingertip amputations.

25
38) (OBQ08.140) The parents of a 2-year-old girl are concerned that their daughter has difficulty feeding herself
from a bottle. They have noticed that she rotates her elbow in front of her body when trying to bring the bottle to her
mouth. Physical exam demonstrates 10 degrees of elbow hyperextension and 160 degrees of flexion. The forearm
does not actively or passively rotate. A radiograph is provided in figure A. Which of the following would be an
indication for a future surgical intervention? QID: 526

A
A. Forearm fixed in 45 degrees of pronation
B. Forearm fixed in 30 degrees of supination
C. Patient younger than 3 years of age
D. An affected older sibling
E. Bilateral involvement

PREFERRED RESPONSE ▼ 5
Radiograph demonstrates proximal radioulnar synostosis. This rare congenital deformity is most frequently a
pronation deformity, but is not frequently a functionally limiting deformity. Observation is the treatment of choice in
most cases. Surgical osteotomy and fusion is beneficial for bilateral involvement with the objective of achieving one
arm fixed in modest pronation and the other fixed in modest supination to facilitate competence in activities of daily
living and hygeine. Surgical excision and reconstruction has not demonstrated successful outcomes.
39) (OBQ09.24) A 50-year-old man complains of numbness and tingling along his right small finger. Physical exam
is notable for the finding demonstrated in Figure A. Elbow flexion reproduces the numbness and tingling. Physical
therapy and splinting have failed to relieve the symptoms. Which of the following is the most appropriate surgical
intervention to alleviate the symptoms while minimizing complications?QID: 2837

A
A. Simple ulnar nerve decompression at the cubital tunnel
B. Ulnar nerve decompression at the cubital tunnel with anterior submuscular transposition
C. Ulnar nerve decompression at the cubital tunnel with anterior subcutaneous transposition
D. Open carpal tunnel release
E. Endoscopic carpal tunnel release

PREFERRED RESPONSE ▼ 1

26
The patient's clinical presentation and physical exam are consistent with cubital tunnel syndrome. The clinical
photograph demonstrates Froment's sign; compensatory IP hyperflexion of FPL (AIN) to compensate for the loss of
adductor pollicis (ulnar nerve) during key pinch. Simple decompression of the ulnar nerve is less invasive and
achieves clinical outcomes equivalent to decompression with transposition.

Zlowodzki et al conducted a meta-analysis evaluating anterior transposition and simple decompression of the ulnar
nerve. No difference in motor nerve-conduction velocities or clinical outcome scores was found.

Bartels performed a prospective randomized trial (included in the Zlowodski meta-analysis) on 152 patients
comparing simple decompression to transposition. No difference in clinical results at 1 year were reported, but a
significantly higher complication rate occurred in the transposition group (31%) compared to simple decompression
(9.6%).

Nabhan et al performed a level 1 study randomizing 66 patients to simple decompression or subcutaneous ulnar
nerve transposition. No differences were found with respect to clinical outcome or nerve conduction velocities.

Illustration V is an educational presentation discussing ulnar nerve transposition at the elbow.

Illustrations: V

40) (OBQ04.38) A 45-year-old male sustained a fall onto his right wrist 2 weeks ago. A radiograph is shown in figure
A. What joint is first affected if left untreated with subsequent development of a SLAC (scapholunate advanced
collapse) wrist? QID: 99

A
A. Capitolunate joint
B. Radioscaphoid
C. Radioulnar
D. Radiolunate
E. STT (scaphotrapezotrapezoidal)

PREFERRED RESPONSE ▼ 2
The clinical presentation is consistent with a SLAC wrist. The radioscaphoid joint is the first to be affected in this
process.

The radiographs of the right wrist demonstrate a scapholunate dissociation, as evidenced by an increased
scapholunate joint space, referred to as scapholunate diastasis (abnormal when the gap is greater than 2 mm and
27
increased from the opposite extremity and other intercarpal spaces).

If left untreated, the wrist may progress to a "SLAC" wrist, as originally described by Watson and Ballet in 1984,
which is the most common form of wrist arthritis. The repetitive sequence of degenerative changes is based on and
caused by articular alignment problems between the scaphoid, the lunate and the radius.

Kuo et al. review the stages of SLAC wrist. They report stage I SLAC wrist involves changes limited to an area of
abnormal contact between the abnormally rotated scaphoid and the radial styloid. In stage II the remaining
radioscaphoid joint is affected, as persistent abnormal load transfer and shear across the cartilaginous surfaces
leads to degeneration of the proximal scaphoid facet. In stage III, the dorsally translated capitate migrates proximally
into the widened scapholunate interval, and degenerative changes occur at the capitolunate joint. The relative
congruency of the radiolunate joint in all positions of lunate rotation due to the spherical shape of the lunate facet
preserves this articulation, and at all stages of SLAC wrist the radiolunate joint is not involved. The lunate is
congruently loaded in every position and, thus, highly resistant to degenerative changes.

Illustration A below shows the stages of involvement in the SLAC wrist.

41) (OBQ05.134) Which of the following complications is most likely to occur following the procedure on the middle
finger of a right hand shown in Figure A? QID: 1020

A
A. Flap failure
B. Cosmetic mismatch of the skin
C. Median nerve motor branch injury
D. Recipient site sensitivity
E. Flexion contracture at the recipient proximal interphalangeal joint

28
PREFERRED RESPONSE ▼ 5
The image shows a thenar flap. The digit is flexed at the PIPJ and extended at the DIPJ during the period prior to
flap division, leading to PIPJ stiffness and flexion contracture.

Thenar flaps can be used for coverage of digital tip injuries where there is exposed bone or extensive pulp loss.
Advantages include more subcutaneous fat than a cross finger flap, good color and texture match, and primary
closure of the donor site. Other disadvantages include limited flap size and donor site tenderness. Contraindications
include RA, Dupuytren’s contracture and advanced age with degenerative disease as these predispose to joint
stiffness.

Fassler et al. reviewed fingertip injuries. The thenar flap can be used for any finger, although the small finger can be
difficult to position comfortably. The flap can be as wide as 2 cm and should be 1.5 times as wide as the
defect so as to restore the normal rounded contour to the tip. To decrease the amount of PIPJ flexion required, the
MCPJ and DIPJ should be flexed as much as possible.

Figure A demonstrates a thenar flap of the middle finger of the right hand. Illustration A shows a preoperative image
with surgical planning marks on the right hand.

Incorrect Answers:
Answer 1: The flap is attached by a pedicle prior to flap division. After division, there is generally good flap take
because of adequate neovascularization of the flap.
Answer 2: Thenar skin is a good cosmetic match for digital pulp skin (both volar skin).
Answer 3: Injury to the recurrent motor branch of the median nerve is distinctly uncommon with this flap.
Answer 4: Donor site sensitivity (not recipient site) is a known complication of this flap.
Illustrations: A
42) (OBQ12.46) The patient sustains the injury seen in Figure A from a gunshot injury. The physical exam is notable
for lack of sensation in his fourth and fifth digits as well as a positive Froment's sign. Which of the following factors
has not been shown to be a significant prognostic indicator of functional recovery following nerve repair? QID: 4406

A
A. Duration to time of repair
B. Repair level
C. Length of repair
D. Postoperative physical rehabilitation
E. Type of autograft used

PREFERRED RESPONSE ▼ 5
The clinical scenario describes an ulnar nerve laceration. Studies have shown that the ulnar nerve does not typically
have good outcomes after nerve repair. (worse recovery than repairs of the tibial, radial, femoral, and
musculocutaneous nerves)

29
Nerve injuries from gunshot injuries (GSWs) can cause both a direct injury to the nerve as well as surrounding
structures (zone of injury). Many factors including age of patient, time to repair, repair level, and length of repair
have been shown to be important determinants in nerve recovery following repair. The type of nerve graft (sural,
saphenous, etc) used has not shown to be statistically significant in terms of functional recovery after nerve repair.

Secer et al.(J. Neurosurg) reviewed 2210 peripheral nerve lesions in 2106 patients which were injured by a GSW
and who were treated surgically. Of the peripheral nerves repaired surgically, the tibial, median, and femoral nerve
lesions showed the best recovery rate. The deep peroneal nerve, ulnar nerve, and brachial plexus lesions had the
worst recovery.

Secer et al.(Surg. Neur.) found that of 455 patients with 462 ulnar nerve lesions only a good outcome was noted in
15.06% of patients who underwent high-level repair, 29.60% of patients who underwent intermediate-level repair,
and 49.68% of patients after low-level repair. The authors also noted that a better functional recovery was noted in
patients who were treated earlier.

Figure A shows a distal humerus fracture caused by a GSW.

Incorrect Answers
Answer 1: Earlier nerve repairs typically have better functional results.
Answer 2: The lower level of nerve repair (more distal), the better functional results.
Answer 3: Shorter length of the nerve repair typically leads to better functional results.
Answer 4: Pre and post operative physical rehabilitation after nerve repairs has been shown to have better results.
43) (OBQ08.254) Replantation is indicated for which of the following amputations. ID: 640
A. multiple level amputation of the small finger at the proximal and distal interphalangeal joints
B. crushed amputation through the distal phalanx of the middle finger
C. complete ring avulsion of the ring finger with tendon injury
D. thumb amputation though the proximal phalanx shaft
E. middle finger amputation though the proximal phalanx shaft

PREFERRED RESPONSE ▼ 4
As outlined by Pederson, the contraindications to replantation are more relative than the indications, but they include
the following: Single-finger replantations at the level of zone II (from the A1 pulley to the distal sublimis tendon
insertion) are rarely indicated, with the notable exception of the thumb. Amputated parts that are severely crushed
and those with multiple level injuries have poor function even if they survive replantation.

While ring avulsion injuries with a vascular injury and no bone, tendon or nerve injury (Urbaniak type 2A ring
avulsion injuries) should be repaired, ring avulsion injuries with bone, tendon or nerve injury (Urbaniak type 2B) or
with complete degloving (Urbaniak type 3) have poor outcomes and Urbaniak and colleagues recommend
amputation for such injuries. Very distal amputations at the level of the nail bed are marginally indicated as there
needs to be approximately 4 mm of intact skin proximal to the nailfold for adequate veins to be present.

Indications for replantation that rule out the other 4 choices of this question include the following: Overall, thumb
replantation probably offers the best functional return. Even with poor motion and sensation, the thumb is useful to
the patient as a post for opposition. A replanted thumb offers the best reconstruction available, toe transfers
notwithstanding. Replantation beyond the level of the sublimis tendon insertion (zone I) usually results in good
function. Multiple finger amputations present reconstructive difficulties that may be difficult to correct without
replantation of one or all of the amputated digits. Any hand amputation from zone III (distally) to zone V (proximally)
offers the chance of reasonable function after replantation, usually superior to available prostheses. Although
usually indicated, the replantation of any hand or arm proximal to the level of the mid-forearm must be carefully
considered.
30
44) (OBQ12.148) A 45-year-old carpenter sustained a table saw injury to his right hand while at work earlier today.
Evaluation in the Emergency Department reveals the defect depicted in Figure A. An island volar advancement flap
was selected for wound closure. This method of thumb reconstruction is best indicated for which of the following
sized defects? QID: 4508

A
A. less than 1 cm
B. 1 cm
C. 1.5 cm
D. 2 cm
E. 2.5 cm

PREFERRED RESPONSE ▼ 5
The clinical vignette is consistent with an oblique amputation of the distal phalanx of a thumb with a defect
measuring >2.5 cm. Island volar advancement flaps are a safe and effective procedure for single-stage closure of
considerably large thumb defects measuring up to 3.5 cm in length.

The operative technique chosen for reconstruction of distal volar thumb defects depends largely on the size of the
defect. Island volar advancement flaps used for defects up to 3.5 cm are pure island flaps in which all of the
proximal attachments, with the exception of the neurovascular bundles, are divided to provide maximal
advancement. Mobility up to 4 cm can be achieved with elevation of the entire volar skin of the thumb from the
underlying tendon sheath providing a considerable advantage in thumb reconstruction. The island volar
advancement flap is useful for coverage of the entire distal phalanx from the IP joint crease to the nail bed.

Foucher et al. reviewed long-term clinical results of 13 neurovascular palmar advancement flaps for thumb tip
coverage. Specifically, they reported on Moberg and O’Brien flaps. The Moberg flap is a pedicled advancement flap
proximally-based on an intact skin pedicle of the thumb including both neurovascular bundles. The O’Brien flap is a
modification of the Moberg technique which advances a volar flap based on a subcutaneous pedicle including both
neurovascular bundles by incising the proximal skin and skin grafting the donor site. The study found that both flaps
preserved near-normal pulp sensibility, MP and IP joint motion, and grasp and pinch strength. They suggested that
Moberg and O’Brien flaps remain the first choice for coverage of 1-2 cm thumb pulp defects.

Baumeister et al. reported on the functional outcomes of 25 patients that underwent thumb pulp reconstructions
utilizing Moberg volar advancement flaps. They found that 72% of patients had no or only minor subjective
complaints, 74% had normal sensitivity, DASH scores showed only minor impairments, no flaps resulted in
decreased grip strength, and only minor restrictions were identified in active IP joint motion. All defects with a length
less than or equal to 2 cm were successfully reconstructed, whereas, patients presenting with defects >2 cm
developed complications.

31
Mutaf et al. reviewed outcomes of 12 patients that underwent thumb reconstruction utilizing an island volar
advancement flap for traumatic distal thumb injuries measuring 3 to 3.5 cm in length. Their results showed that none
of the flaps failed, no patients had limited mobility or scar contractures, near-normal sensation was achieved,
excellent recovery of pinch strength occurred, and maximal preservation of thumb length was possible in all
patients.

Figure A and Illustrations A through C represent a case example presented by Mutaf et al. Figure A depicts an
oblique amputation of the distal phalanx of a right thumb. Illustration A reveals elevation of an island volar
advancement flap on both sides of the digital neurovascular bundles in the same thumb. Illustration B reveals flap
advancement and Illustration C reveals a postoperative image of the same thumb 4 months after surgery.

Incorrect Answers:
Answers 1 & 2: Small or superficial defects may be amenable to conservative treatment or local flaps depending on
the location of the defect.
Answers 3 & 4: The Moberg flap with modifications to lengthen distal advancement as necessary is considered a
standard option for medium-sized defects of the thumb pulp less than or equal to 2 cm.

45) (OBQ05.270) A 34-year-old male sustains amputations of the 4th and 5th fingers at the level of the middle
phalanx after cutting them with a butcher knife. Which of the following techniques would most likely increase total
surgical time during replantation?QID: 1156
A. Utilizing an on-call surgical team experienced at digit replantations
B. Digit-by-digit repair method
C. Structure-by-structure repair method
D. Bone shortening procedures to avoid the need for vein grafts
E. Obtaining a thorough inventory of the digit structures that are deficient and the structures that are available for
reconstruction

PREFERRED RESPONSE ▼ 2
Surgical time in multiple digit replantation is increased by digit-by-digit repair techniques and decreased by
structure-by-structure repair techniques.

The Level 5 article by Morrison and McCombe reviews the indications and results of finger replantation. Results of
replantation from the DIP to PIP joint typically have good outcomes whereas replantations at the proximal
interphalangeal (PIP) joint to MCP joint have poor outcomes due to flexion contractures. The review article by Wang
cites that tenolysis and tendon procedures were needed in 47.2% of the published cases of digit replantation and is
the leading type of secondary operation.

The classic article by Waikakul et al reviewed 1018 digital replantations in 552 patients. They had a 92% rate of
successful outcome and found that type of injury was the most important factor influencing immediate and late
outcome.

32
46) (SBQ07.3) A 19-year-old football player suffers a fall onto a pronated, extended wrist. He has pain with resisted
ulnar deviation and is tender to palpation just distal to the ulnar styloid. He has no tenderness over the extensor
carpi ulnaris (ECU) tendon. Current radiographs are shown in in Figures A and B and and MRI of the wrist is shown
in FIgure C. Which of the following is the most likely diagnosis? QID: 1388

A B C
A. ECU tendon rupture
B. Triangular fibrocartilaginous complex (TFCC) tear
C. Hook of hamate fracture
D. Scapholunate ligament injury
E. Perilunate dislocation

PREFERRED RESPONSE ▼ 2
Fall from standing onto an extended and pronated wrist is a risk factor for injuries to the soft tissues of the wrist. The
structures at risk include the triangular fibrocartilaginous complex (TFCC), the lunotriquetral ligament, ulnolunate
ligament, hook of hamate, ulnar styloid, and the extensor carpi ulnaris (ECU) tendon sheath. Pain with resisted ulnar
deviation and ulnar catching are all concerning for injury to the TFCC. MRI is useful for diagnosing TFCC tears (
Illustration A shows another example).

Papapetropoulos et al in their review article discuss the evaluation and arthroscopic treatment of TFCC injuries.
Specifically they discuss that most tears in athletes are acute and amenable to repair by repair of the dorsal tear to
the ECU tendon sheath.

Cohen in his review of injuries in athletes discusses scapholunate ligament, lunotriquetral ligament, and midcarpal
injuries. Of note he divides scapholunate and lunotriquetral ligament injuries into dissociative lesions (abnormal
motion within proximal carpal bones) vs. midcarpal lesions which are generally considered nondissociative
(abnormal motion between proximal and distal carpal bones).

Rettig in his review of sports injuries of the extremities discusses the Palmer classification of TFCC tears.
Specifically he notes that central tears are more associated with repetitive activities in patients with positive ulnar
variance.

Incorrect Answers:
Answer 1: The patient is not tender in the region of the ECU tendon sheath.
Answer 3: The carpal tunnel view radiograph shows no hook of hamate fracture.
Answer 4 and 5: Wrist radiographs shows no scapholunate widening or perilunate dislocation. Physical exam in this
case is more consistent with a TFCC injury.
Illustrations: A

33
47) (OBQ06.1) A 32-year-old male sustains a type IIIb open proximal third tibia fracture. Four days after
intramedullary nailing of the tibia, the wound is clean and ready for coverage with a medial gastrocnemius rotational
flap. What is the dominant arterial blood supply to this flap? QID: 12
A. Superior and inferior genicular arteries
B. Anterior tibial artery
C. Posterior tibal artery
D. Sural artery
E. Saphenous artery

PREFERRED RESPONSE ▼ 4
The dominant arterial blood supply to a medial gastrocnemius muscle flap is the sural artery.

Rotational gastrocnemius flaps are useful for coverage of the proximal third of the tibia and some wounds/defects
about the knee. Medial and lateral gastrocnemius arterial supply is from the medial and lateral sural arteries
respectively. Coverage of the middle third of the tibia requires use of a rotational soleus muscle flap, supplied by the
peroneal artery proximally and the posterior tibial artery distally. Coverage of the distal third of the tibia requires a
free muscle flap transfer, based on a specific vascular pedicle.

Illustration A depicts the medial gastrocnemius flap with its sural artery pedicle.
Illustrations: A
48) (OBQ06.71) Ischemia induced by the conversion of hypoxanthine to xanthine is thought to be involved in the
mechanism of reperfusion injury following restoration of circulation after replantation. Which of the following agents
inhibits conversion to xanthine and has been shown to improve outcomes following digit replantation? QID: 182
A. Tissue plasminogen activator
B. Heparin
C. COX 2 inhibitor
D. Allopurinol
E. Leeches

PREFERRED RESPONSE ▼ 4
Allopurinol is a xanthine oxidase inhibitor and may have a beneficial role in replantation. Inhibition of xanthine
oxidase also decreases uric acid in patients with gout.

Waikakul et al. published a randomized control trial with a 2-year follow-up comparing thumb replantation with and
without adjunctive allopurinol. There were 60 patients in the trial group, and 38 patients in the control group. All were
young, healthy laborers who had sharp or locally crushed amputations of the thumb at the proximal phalanx with a
total ischemic time >10 hours. The standard management for thumb replantation was used in these patients, except
that 300mg allopurinol was given orally in the trial group on admission and a further 300mg for another 5 days. After
the operation, the trial group had a lower infection rate, and less postoperative pain and chronic swelling than the
control group. Recovery of sensation was also better in the trial group.
49) (OBQ08.227) Flexor tendons of the fingers within Zone 2 receive their primary nutritional supply from: QID: 613
A. Vinculae
B. Phalangeal periosteum
C. Musculotendon junction
D. Tendon insertion
E. Diffusion from the synovial sheath

PREFERRED RESPONSE ▼ 5
The vascularity of tendon varies depending on the type of tendon (e.g. with or without a sheath) and the location.
Sheathed tendons (e.g. flexor tendons of the hand) have a dual blood supply via both vascular perfusion but also
34
have regions that are relatively avascular where they receive nutrition through synovial diffusion. This is the case in
zone 2 of the digital flexor tendons where the primary nutritional supply is from synovial diffusion through the parietal
paratenon which allows for passive nutrient delivery to the flexor tendon within the sheath. The digital flexor tendons
also receive minor direct arterial perfusion in zone 2 through the vinicular system, osseous bony insertions, reflected
vessels from the tendon sheath and longitudinal vessels from the palm, but this is not the major blood supply.

Tendons not enclosed by a sheath receive their blood supply directly from vessels entering from the tendon surface
or from the tendon-to-bone insertion.

50) (OBQ08.111) Percutaneous screw fixation for non-displaced scaphoid waist fractures has been shown to have
which of the following differences compared to closed treatment? QID: 497
A. Increased direct and indirect cost
B. Slower return to work
C. Higher union rates
D. Reduced time to fracture union
E. Improved motion and grip strength after 2 years

PREFERRED RESPONSE ▼ 4
Fixation of non-displaced scaphoid fractures with a percutaneous screw has resulted in a shorter time to union (6-7
weeks versus 10-12 weeks) and faster return to work or sports.

Arora et al found the indirect cost reduction by a quicker return to work was shown to offset the direct costs of
surgical intervention.The operatively treated group had a better mean DASH-score than the conservative group.
Fracture union was seen in the screw fixation group at a mean of 43 days and in the cast immobilization group at a
mean of 74 days.

Bond et al found in active military personnel there was faster healing but no difference in ultimate union rates or final
grip strength or range of motion between percutanous screw fixation and non-operative groups. The average time to
fracture union in the screw fixation group was seven weeks compared with twelve weeks in the cast immobilization
group. There was no significant difference in the range of motion of the wrist or in grip strength at the two-year
follow-up evaluation.

-------------------------------------------------------------(50)----------------------------------------------------------------------------------------

35
1) (OBQ11.143) A child is seen in the pediatric orthopedic hand clinic for evaluation of a congenital deformity. A
clinical photograph and radiograph are seen in Figures A and B. What is the next best step in this child's evaluation
to rule out an associated autosomal-recessive lethal condition? QID: 3566

A. Cardiac ultrasound and renal ultrasound


B. Cardiac ultrasound, barium swallow and MRI
C. LFTs, CBC and cardiac ultrasound
D. Cardiac ultrasound, peripheral blood smear and MRI
E. CBC, peripheral blood smear and chromosomal breakage analysis

PREFERRED RESPONSE ▼ 5
The clinical and radiographic images depict a patient with radial club hand. This is associated with a number of
congenital anomalies including Fanconi’s Anemia (FA), thrombocytopenia absent radius (TAR), Holt-Oram
syndrome, VACTERL syndrome, and VATER syndrome. Although all these congenital anomalies are important to
recognize and treat, none is more life-threatening than FA. FA is an autosomal-recessive condition resulting in
aplastic anemia and eventual death. The typical presentation is between 6-9 years of age. It is the most common
inherited form of aplastic anemia. Genetic testing will reveal increased chromosomal breakage. A CBC will show
decreased leukocytes, red blood cells and platelets. Of the choices above, it is the only one which requires bone
marrow transplantation for survival.

DeKerviler et al reviewed many clinical and radiological features of FA. Congenital malformations affect multiple
systems including the radial aspect of the forearm as well as the urinary system. They recommended ultrasound
and imaging modalities for early detection of FA.

Alter described hematologic disorders manifest in the pediatric upper extremity. Pediatric orthopedic surgeons may
be the first to detect FA, Diamond-Blackfan anemia and TAR. As such, an understanding of the syndromes and
inheritance patterns may aid in early detection and aid in future genetic counseling.

2) (OBQ07.33) In the majority of patients, the deep palmar arch has a main contribution from what vessel which
travels between the two heads of the first dorsal interosseous muscle? QID: 694

A. Superficial palmar branch of the radial artery


B. Princeps pollicis artery
C. Deep dorsal (terminal) branch of the radial artery
D. Common palmar digital artery
E. Ulnar artery

PREFERRED RESPONSE ▼ 3
The deep palmar arch (deep volar arch) is an arterial network found in the palm. In the majority of patients it is
formed mainly from the terminal part of the radial artery, with the ulnar artery contributing via its deep palmar
36
branch. This is in contrast to the superficial palmar arch, which is formed predominantly by the ulnar artery with
some contribution by the superficial palmar branch of the radial artery. The deep palmar arch lies upon the bases of
the metacarpal bones and on the interossei of the hand, being covered by the oblique head of the adductor pollicis
muscle, the flexor tendons of the fingers, and the lumbricals of the hand. At the wrist the radial artery passes
between the radial collateral ligament and the first dorsal compartment after which it dives between the heads of the
first dorsal interossei where it anastomoses with the deep palmar branch from the ulnar artery, completing the deep
volar arch
Illustrations: A
3) (OBQ04.152) Syndactyly is associated with all of the following conditions EXCEPT?QID: 1257
A. Apert syndrome
B. Poland's syndrome
C. Holt-Oram syndrome
D. Carpenter syndrome
E. Tay-Sach's Disease

PREFERRED RESPONSE ▼ 5
Syndactyly is one of the most common congenital hand deformities with an incidence of 1 in 2000 live births and is
associated with Poland's syndrome (chest wall deformity with limb hypoplasia), Apert syndrome (autosomal
dominant inheritance with mental retardation, craniosynostosis), Holt-Oram syndrome (also known as "heart-hand
syndrome" with abnormalities of the heart and upper limbs), and Carpenter syndrome (acrocephaly, peculiar facies).
There is no association of syndactyly with Tay-Sach's disease.

Ireland and colleagues reviewed 43 consecutive cases of Poland's syndrome and found that clinical features were
variable but always included congenital aplasia and syndactyly which is usually incomplete and simple. It can
involve all fingers and not uncommonly the thumb as well.

Galland and Bora wrote a review on congenital disorders of the upper extremity, and describe that syndactyly may
occur as part of a syndrome including Poland's syndrome (chest wall anomalies and syndactyly) or Apert's
syndrome (severe syndactyly with craniosynostosis, mental retardation, ankylosed IP joints, flattened faces and
hypotelorism). There is no known increased incidence in Tay-Sach's disease, however.
4) (OBQ07.50) A 54-year-old gentleman presents to your office with a mass on top of the distal phalanx that has
enlarged over the last three months. His main complaint is significant tenderness to palpation over the mass. There
is no pain with forcible movement of his fingers. A clinical photo is shown in Figure A. A dedicated radiograph of the
distal phalanx is shown in Figure B. What treatment option is most appropriate for the best patient outcome? 711

A B
A. Observe
B. Needle aspiration
C. Fusion of distal interphalangeal joint
D. Removal of bone spur and cyst
E. Obtain infectious work-up

37
PREFERRED RESPONSE ▼ 4
This is a classic presentation of a mucous cyst. The most appropriate treatment would be excision of the cyst and
removal of the underlying bone spur.

A mucous cyst of the hand is usually a small, soft, benign structure. They are associated with osteoarthritis and
develop around bone spurs near a joint. Surgery is typically recommended if there is significant pain at the site of
the cyst or with range of motion of the involved joint. Nail bed deformity may occur with disease progression if left
untreated.

Rizzo et al. examined a series of 132 patients with mucous cysts, comparing outcomes between injection and
surgery. They found that 60% of people with aspiration and steroid injection had complete resolution of the cyst
compared to 100% with excision.

Figure A shows a small mucous cyst just proximal to the nailbed. Figure B shows a radiograph of the distal
interphalangeal joint. There is extensive joint arthritis with dorsal bone spurs.

Incorrect Answers:
Answer A: Pain from a mucous cyst is usually constant, but in some people it may come and go. Rarely the cysts
will resolve over time. Typically, the mucous cyst will progress with time and cause nail deformity.
Answer B: Aspiration will lead to a >40% recurrence.
Answer C: Removal of cyst and joint fusion would be indicated if there was pain with with any forcible movement of
the joint.
Answer E: Infection is usually not associated with a benign mucous cyst.
5) (OBQ11.149) A 7-month-old boy has radial longitudinal deficiency. The initial work-up is negative for any cardiac,
hematopoetic or renal abnormalities. He has good active elbow flexion and no other deformities. A clinical image
and radiograph are seen in Figures A and B. Surgical management should include which of the following? 3572

A B
A. Definitive splinting and stretching
B. Pollicization of the index finger
C. ECRB and ECRL transfers to the ulna
D. Free fibula transfer to the forearm
E. Ulna centralization and possible tendon transfers

PREFERRED RESPONSE ▼ 5
The image and vignette are consistent with non-syndromic radial longitunidal deficiency (RLD). He has a viable
thumb with good active elbow flexion, therefore the treatment of choice is centralization and tendon transfers to re-
establish balance across the wrist. The goal of centralizing the carpus on the ulna is to improve reach and to
stabilize tendons and muscle balance across the wrist.

38
The decision for surgery is based on the range of motion of the elbow. In the case of a stiff elbow, a centralization
should not be performed as doing so would prohibit the patient from reaching his/her face for dentition and feeding.

RLD is commonly associated with Thrombocytopenia absent radius (TAR), Holt-Oram and Fanconi’s anemia.
Classification of RLD is based on the Bayne and Klug system and takes into account the amount of remaining
radius present:
Type I: short distal radius
Type II: short distal radius with residual growth plates
Type III: small proximal radius
Type IV: absent radius

Incorrect Answers:
Answer 1: For advanced cases (types III/IV), surgery is recommended
Answer 2: The patient in the scenario has a thumb so pollicization is not indicated
Answer 3: This is not a surgical option
Answer 4: MTPJ transfers have been explored but not free fibula
6) (OBQ10.235) Which of the following hand injuries is most appropriately treated with a volar advancement
(Moberg) flap closure? QID: 3334

A B C D E

A. Figure A
B. Figure B
C. Figure C
D. Figure D
E. Figure E

PREFERRED RESPONSE ▼ 4
Figure D shows a volar thumb defect which can be best covered with a Moberg advancement volar flap (if < 2 cm).
FDMA (1st dorsal metacarpal artery) and neurovascular island flaps are typically used to cover larger soft tissue
defects of volar aspect of the thumb. FDMA (1st dorsal metacarpal artery) flaps can also be used for dorsal thumb
wounds as shown in Figure B. The cross-finger flap is a useful heterodigital flap for digital wounds with primarily
volar tissue loss (Figure A). Additionally, several articles have advocated secondary intention healing even if bone is
exposed as discussed in the 2009 OITE question #48. The thenar flap is useful for volar defects of the index and
middle fingers (Figure C). Figure E represents a ring avulsion injury and it is treated with vessel repair if there is
inadequate circulation and the bone, tendon, and nerve components are intact. Amputation of the digit is chosen if
there is inadequate circulation concomitant with bone, tendon, or nerve injury.

The referenced articles by Martin and Hynes are review articles discussing the treatment options available for digit
injuries. Illustration A shows the planned incisions for a moberg advancement flap on a volar thumb defect and

39
Illustration B shows the completed Moberg.

7) (OBQ12.235) Figure A is a radiograph of a 35-year-old women who sustained an isolated left wrist injury after a
fall onto an outstretched hand. She has been complaining of left dorsal wrist pain since the fall. Examination reveals
a positive Watson's scaphoid shift test. What ligamentous structure is an important secondary stabilizer to prevent
dorsal intercalated segment instability (DISI) deformity in this patient? QID: 4595

A
A. Transverse carpal ligament
B. Dorsal intercarpal ligaments
C. Triangular fibrocartilage complex
D. Dorsal lunotriquetral ligament
E. Volar lunotriquetral ligament

PREFERRED RESPONSE ▼ 2
The integrity of the dorsal intercarpal ligaments is important in preventing dorsal intercalated segment instability
(DISI) deformity and persistent scapholunate instability.

Scapholunate instability is the most common carpal instability. The primary stabilizing structure of the scaphoid and
lunate bones is the scapholunate ligament(dorsal stronger than volar), which is commonly injured with a fall on an
outstretched hand. Secondary stabilizers of the scaphoid and lunate include the dorsal intercarpal ligaments and the
dorsal radiocarpal ligaments. Failure to recognize injury of these structures can cause persistent dorsal intercalated
segment instability (DISI). This can predispose patients to a SLAC wrist and early wrist osteoarthritis.

Mitsuyasu et al. examined the role of dorsal intercarpal ligaments (DIC) in scapholunate instability. They showed
that the DIC had an important role in stabilizing the scaphoid and lunate bones with static and dynamic movements.
The authors of this study suggest that the DIC ligament should be assessed intraoperatively and consideration
should be given to repair and/or reconstruction with surgical management of scapholunate ligament tears.

Viegas et al. showed that the dorsal intercarpal and the dorsal radiocarpal ligaments form a lateral V configuration
over the dorsal wrist. This configuration acts as an indirect dorsal stabilizing effect on the scaphoid throughout the
range of motion of the wrist. Their integrity acts to ensure normal wrist kinematics.

40
Figure A shows an AP and lateral radiograph of the left hand. There is significant gapping between the scaphoid
and lunate articulation. This is indicative of a complete scapholunate dissociation, however both wrists should be
imaged as this deformity may exist without injury. Illustration A shows the anatomy of the dorsal intercarpal and the
dorsal radiocarpal ligaments.

Incorrect Answers:
Answers 1, 3, 4, 5: Carpal ligamentous injuries can occur with falls onto an outstretched hand. However, the clinical
and radiographic features of this patient do not show injury to these structures.

8) (OBQ13.240) A 30-year-old male sustains a 3.5 cm long thumb pulp injury seen in Figure A. He undergoes a
procedure to restore the soft tissue envelope. Which treatment option is contraindicated because of increased risk of
interphalangeal joint stiffness? QID: 4875

A
A. Moberg volar advancement flap
B. Foucher first dorsal metacarpal artery flap
C. Littler neurovascular island flap
D. Free great toe pulp transfer
E. Holevich first dorsal metacarpal artery flap

PREFERRED RESPONSE ▼ 1
This patient has a large thumb pulp defect measuring 3.5 cm in length, extending proximal to the interphalangeal joint (IPJ) crease.
Inset of a Moberg flap large enough to cover the defect would necessitate IPJ flexion >45 degrees, increasing the risk of IPJ
stiffness.

Thumb pulp defects may be resurfaced by different means, depending on size. The Moberg flap is suited for medium (1.8-3 cm)
defects. For defects >1.5 cm, there is increased risk of wound dehiscence, parrot beak nail deformity, and decreased soft tissue
padding. Modifications such as V-Y flaps, bilateral Z-plasties, Burrow triangles, 2 lateral triangular flaps at the proximal edge of the
flap, or advancement of an island flap with skin grafting of the secondary defect (O’Brien modification), are recommended.

Baumeister et al. reviewed the functional outcome of Moberg flaps. These flaps do not cause marked impairment of active ROM

41
and any reduction in the AROM of the IP joint is because of a loss of hyperextension.

Horta et al. reviewed the use of multiple flaps (Moberg, radial innervated cross-finger, Venkataswami-Subramanian, Foucher,
Tezcan, and Littler). They recommended the Foucher flap because of good sensibility, single-stage surgery, and no need for
cortical reintegration (unlike the Littler flap)

Figure A shows a large thumb pulp defect. Illustration A shows the options for resurfacing thumb pulp defects of different sizes.
Illustration B is a diagram of these options. Illustrations C and D depict the Holevich dorsal metacarpal artery flap (with overlying
skin strip). Illustrations E and F depict the Foucher dorsal metacarpal artery flap (islanded).

Incorrect Answers:
Answers 2, 3, 4, 5: These flaps are all possible options for large thumb pulp defects.

9) (OBQ06.120) A collegiate baseball player injures his left small finger sliding into third base. He complains of pain
and swelling. The finger is ecchymotic, swollen throughout, and painful with attempted range of motion of the PIP
joint. No sensory or vascular deficits are present. A radiograph is provided in Figure A. Which of the following
interventions will provide the best outcome? QID: 306

A
A. Buddy taping the small finger to the ring finger
B. Immobilization of the MCP in flexion and the PIP and DIP in extension with a custom splint
C. External fixation
D. Open reduction internal fixation
E. PIP joint arthrodesis

PREFERRED RESPONSE ▼ 4

42
The radiograph shows an oblique fracture of the distal proximal phalanx that extends into the joint with an articular
step off. Open reduction internal fixation will correct the deformity, expedite finger rehabilitation, and prevent early
degenerative arthritis. Closed treatment without fixation will not reliably hold the fracture reduced while the bone
heals. Arthrodesis is unnecessary in this young, active patient.
10) (OBQ04.266) An ulnar shortening osteotomy would be MOST indicated for which of the following patients
presenting with longstanding ulnar sided wrist pain refractory to conservative measures? QID: 1371

A. 34-year-old female with an ulnar neutral wrist and distal radioulnar joint incongruity
B. 34-year-old female with an ulnar positive wrist and distal radioulnar joint incongruity
C. 34-year-old female with an ulnar negative wrist and distal radioulnar joint incongruity
D. 78-year-old female with ulnar positive wrist and distal radioulnar joint arthritis
E. 78-year-old female with ulnar negative wrist and distal radioulnar joint arthritis

PREFERRED RESPONSE ▼ 2
Ulnar shortening osteotomy is the best procedure for young adults with longstanding ulnar sided wrist pain due to
ulnar positive variance and associated distal radioulnar joint (DRUJ) incongruity. Ulnar positive variance causes an
"ulnar impaction syndrome" as the distal ulnar styloid can cause damage to the triangular fibrocartilage complex
(TFCC), and ulnocarpal joint (illustration A.)

Advantages of an ulnar shortening osteotomy include preservation of ulnar dome articular cartilage and DRUJ joint,
and also tightens the TFCC and ulnocarpal ligaments as the distal ulna is translated and fixed proximally after the
osteotomy.

It is also important to note that ulnar shortening in the setting of preoperative DRUJ incongruity may simultaneously
decrease ulnocarpal abutment and improve congruity at the distal radioulnar articulation. One specific instance in
which to avoid an ulnar shortening in an ulnar positive wrist with DRUJ incongruity is a joint with a reverse oblique
inclination in the coronal plane. This may create abnormally high radioulnar contact and may lead to joint
degeneration

11) (OBQ09.1) All of the following are possible sites of compression for the ulnar nerve EXCEPT: QID: 2814

A. arcade of Struthers
B. ligament of Struthers
C. flexor carpi ulnaris fascia
D. medial intermuscular septum
E. Osborne's ligament

PREFERRED RESPONSE ▼ 2
43
There are five sites of potential ulnar nerve entrapment around the elbow: arcade of Struthers, medial intermuscular
septum, medial epicondyle, cubital tunnel, and deep flexor pronator aponeurosis. (me as mi of cd)

The ulnar nerve emerges from the medial intermuscular septum, under the arcade of Struthers, and lies on the
medial head of the triceps. At the level of the elbow, the ulnar nerve continues distally toward the posterior aspect of
the condylar groove, passing between the medial epicondyle and olecranon to enter the cubital tunnel. The roof is
formed by the arcuate (Osborne’s) ligament. This ligament blends distally with the antebrachial fascia superficial to
the aponeurosis and connects the ulnar and humeral heads of the FCU. The ligament of Struthers is a fibrous band
extending from the supracondylar process of the humerus to the medial epicondyle which can cause compression of
the median nerve.

Elhassan et al discuss the pathogenesis, evaluation, and treatment of entrapment neuropathy of the ulnar nerve.

Illustration A shows the various site of compression at the elbow. Illustration V shows a submuscular ulnar nerve
transposition performed Dr. Susan E. Mackinnon
Illustrations: A V
12) (OBQ08.100) A 28-year-old professional baseball player injures his middle finger sliding into the catchers shin
guard at home plate. He complains of pain and deformity of the middle finger. A radiograph is provided in figure A.
All of the following are true EXCEPT: QID: 486

A
A. Anatomic reconstruction of the articular surface is prognostic of clinical function
B. Proximal interphalangeal joint subluxation precludes a normal gliding flexion arc
C. Hinging at the fracture site must be avoided
D. Early motion should be initiated in postoperative therapy
E. Early degenerative arthritis can be expected if the joint is not adequately reduced.

PREFERRED RESPONSE ▼ 1
The radiograph demonstrates a dorsal fracture dislocation of the proximal interphalangeal joint of the middle finger.
Kiefhaber and Stern review the presentation, evaluation, and treatment of PIP fractures. Congruent reduction of the
joint to allow the middle phalanx to glide around the proximal phalangeal head is paramount to prevent joint
subluxation and instability. Anatomic reconstruction of the articular surface is desirable but not necessary for
successful clinical outcome.
13) (OBQ12.274) Compressive injury to the posterior interosseous nerve will lead to EMG fibrillations in which of the
following muscles? QID: 4634
A. Extensor Carpi Radialis Longus/Extensor Carpi Radialis Brevis/Brachoradialis
B. Extensor Carpi Radialis Longus/Supinator/Abductor Pollicis Longus
C. Extensor Pollicis Longus/Supinator/Abductor Pollicis Longus
D. Brachoradialis/Supinator/Extensor Pollicis Longus
E. Extensor Pollicis Longus/Supinator/Abductor Pollicis Brevis

PREFERRED RESPONSE ▼ 3

44
Based on the choices above, fibrillations will be seen in the extensor pollicis longus, supinator and abductor pollicis
longus muscles.

The radial nerve splits into the superficial radial branch and the posterior interosseous nerve (PIN) at the anterior
aspect of the radiocapitellar joint, just proximal to the supinator muscle. The PIN innervates the EDC, EDM, ECU,
EPB, EPL, EIP, APL and sometimes the ECRB(!!supinator). Compressive neuropathy of the PIN leads to motor
dysfunction, namely weakness with wrist and finger extension.

Lubhan et al. review uncommon compression neuropathies affecting the upper extremity. They indicate that PIN
syndrome may be caused by rheumatoid arthritis and compressive ganglion cysts. Depending on which nerve
branch is affected, partial lesions may develop. They recommend use of conservative measures (rest, activity
modification and splinting) first. Decompressive procedures may be indicated in symptoms lasting greater than 3
months.
Illustration A shows the course of posterior interosseous nerve from proximal to distal along the course of the
supinator. This proximal edge of the supinator (Arcade of Froshe), the fibrous edge of the ECRB and the leash of
Henry are three main points of compression of the PIN.
Incorrect Answers
Answer 1, 2, 4: The radial nerve proper innervates the ECRL, ECRB and Brachoradialis
Answer 5: The recurrent motor branch of the median nerve innervates the APB
Illustrations: A
14) (OBQ07.235) A collegiate rower complains of dorsal wrist pain for 6 weeks refractory to NSAIDs and bracing.
Maximal tenderness is palpated on the dorsoradial forearm approximately 5 cm proximal to the wrist. Pain is
exacerbated with resisted wrist extension. Radiographs are unremarkable. A steroid injection should be directed into
the compartment containing which of the following structures? QID: 896
A. APL and EPB tendons
B. ECRL and ECRB tendons
C. EPL tendon
D. APL and ECRB tendons
E. Brachoradialis tendon

PREFERRED RESPONSE ▼ 2
The clinical scenario is consistent with intersection syndrome, a inflammatory response to overuse at the site of the
second dorsal compartment crossing under the first dorsal compartment approximately 5 cm proximal to the wrist.
An anatomical depiction is provided in illustration A. Injections of the second dorsal compartment, which includes
ECRL and ECRB, may relieve symptoms and quell inflammation. Intersection must be differentiated from
DeQuervain's syndrome, which is tenosynovitis of the first dorsal compartment. Injections of the first dorsal
compartment, which includes APL and EPB, are part of the treatment algorithm for Dequervain's. Wood et al
summarizes the evaluation and treatment of sports-related wrist injuries. Grundberg et al demonstrates the
pathologic abnormality of intersection syndrome is stenosing tenosynovitis of the second compartment explaining
the rationale behind steroid injections into the sheath.
Illustrations: A
15) (OBQ09.80) Chronic injury to what anatomic structure can lead to a boutonnière deformity of the finger?2893
A. terminal extensor tendon
B. sagittal band
C. volar plate
D. flexor digitorum profundis tendon insertion
E. central slip of the extensor tendon

PREFERRED RESPONSE ▼ 5

45
Rupture of the central slip of the extensor tendon and subsequent subluxation of the lateral bands leads to a
boutonnière deformity, which is characterized by PIP flexion and DIP extension. Central slip injuries can be caused
by a laceration or traumatic avulsion. In the listed reference, Imatami et al treated a series of central slip injuries
associated with attachment fractures successfully with ORIF. As stated by Tuttle et al, rupture of the terminal
extensor tendon leads to a mallet finger. Sagittal band injury can lead to subluxation of the extensor tendon at the
level of the MCP joint. Chronic volar plate injuries can lead to swan neck deformities. Avulsion of the FDP insertion
leads to a jersey finger. Illustrations A and B are a clinical photograph and anatomic diagram of a boutonneire
deformity.
Illustrations: A B
16) (OBQ11.144) A 39-year-old male presents with longstanding right wrist pain. He has failed conservative
measures including prolonged immobilization. His radiographs and MRI are seen in figures A and B. Which of the
following options is an accepted treatment option? QID: 3567

A B
A. Ulnar shortening osteotomy
B. TFCC repair
C. Radius core decompression
D. Arthroscopic lunate chondroplasty and debridement
E. Scapholunate ligament reconstruction

PREFERRED RESPONSE ▼ 3
The patient in the clinical scenario has Kienbock's disease. Treatment options include a joint leveling procedure, or
radius core decompression, which is thought to incite a local vascular healing response in the lunate.

Sherman et al did a biomechanical study reviewing distal radius core decompression for Kienbock's disease.
Although the procedure has good clinical outcomes for this disease process, their findings did not show any
biomechanical explanation for these good outcomes.

Illarramendi et al reviewed results of curettage of the distal radius and ulna metaphyseal bone through small cortical
windows for the treatment of Kienbock's disease. They concluded that the decompression procedure had good
results without any complications. Most patients had improvement in pain and were able to return to work.

Incorrect Answers:
Answer 1: Kienbock's disease is commonly associated with ulnar negative variance which is thought to lead to
increased forces on the lunate leading to this disease. Therefore a ulnar shortening osteotomy would not be
appropriate.
Answer 2,4,5: Are not treatment options for this disease process.

46
17) (OBQ08.19) A 2-year-old boy has the upper limb deformity seen in Figures A and B. All of the following are
associated with this condition EXCEPT? 405

A B
A. Fanconi's Anemia
B. Holt-Oram syndrome
C. VATER syndrome
D. VACTERL syndrome
E. Osteogenesis Imperfecta

PREFERRED RESPONSE ▼ 5
The clinical presentation is consistent with radial longitudinal deficiency, also known as "radial clubhand", which is
associated with all of the listed conditions except for osteogenesis imperfecta.

Maschke et al report "radial longitudinal deficiency encompasses a spectrum of upper limb dysplasias and
hypoplasias. The clinical presentation of the involved upper limb is often more obvious than the potentially life-
threatening associated systemic conditions. All children presenting with radial longitudinal deficiency, regardless of
severity, require a renal ultrasound, echocardiogram, and complete blood count to evaluate the potential for
associated systemic conditions; these include Fanconi’s anemia, the Holt-Oram syndrome, and the VATER
(vertebral anomalies, anal atresia, tracheoesophageal fistula, esophageal atresia, renal agenesis) syndrome or
VACTERL (vertebral anomalies, anal atresia, cardiac abnormalities, tracheoesophageal fistula, renal agenesis, and
limb defects) association."
18) (OBQ08.90) Madelung's deformity of the distal radius is caused by which of the following?QID: 476

A. Premature fusion of the distal radial ulnar joint


B. Physeal growth mismatch between the distal radius and ulna
C. Nutritional deficiency affecting the physeal zone of provisional calcification
D. Impaired growth of the volar and ulnar aspect of the distal radial physis
E. Unrecognized trauma

PREFERRED RESPONSE ▼ 4
Madelung's deformity is that of excessive ulnar/palmar angulation of the distal radius caused by impaired growth of
the volar and ulnar aspect of the distal radial physis(leads to radial and volar displacement of hand ). It may be caused by
either a bony lesion in the palmar/ulnar corner of the distal radial physis or an abnormal radial-carpal ligament
(Vicker's ligament). The other answers do not cause Madelung's deformity. Leri-Weill dyschondrosteosis is a rare
genetic disorder caused by mutation in the SHOX gene that causes mesomelic dwarfism with associated
Madelung's defomity of the forearm.

Illustration A is a radiographic example of Madelung's deformity.

47
19) (OBQ13.15) A 45-year-old man presents with a three-month history of unilateral symptoms in his right wrist and
hand. He first noticed a palpable nodule over the volar aspect of his wrist about three months ago. The nodule
would become painful after weekends of heavy drinking at which time he noticed tingling sensation in his index and
middle fingers. He notes that ibuprofen has helped improve the pain in the past. On clinical examination, he has a
palpable, painless, solid nodule over the volar aspect of his wrist. He has no motor or sensory deficits and negative
carpal tunnel provocative tests. An axial CT and MRI image are provided in figures A and B. What would be the
most appropriate next step in the management of his symptoms? QID: 4650

A B
A. Fine needle aspiration
B. Open biopsy
C. Night splints
D. Colchicine and referral to rheumatologist
E. Surgical excision

PREFERRED RESPONSE ▼ 4
The clinical presentation is consistent with carpal tunnel syndrome caused by an atypical space occupying lesion -
in his case, gout. The most appropriate next step in the management of his symptoms would be medical therapy,
such as prophylaxis with colchicine referral to a rheumatologist.

Carpal tunnel syndrome is the most common compressive neuropathy, affecting up to 10% of the general
population. Risk factors include female sex, advanced age, obesity, and repetitive motion activities. Typically,
patients will develop symptoms of median nerve compression including thenar muscle atrophy, numbness in the
radial 3.5 digits, night pain, and positive Tinel's and Phalen tests. First line management is non-operative, including
NSAIDs, night splints, and activitiy modification. Carpal tunnel release surgery is indicated for those who have failed
conservative management.

Chen et al. described 23 unusual cases of CTS in which space-occupying lesions were responsible for the
symptoms and signs of median nerve compression. In patients with an atypical presentation, such as male gender,
non-middle-aged, or unilateral involvement, space-occupying lesions such as gout, synovial sarcoma, lipoma, and
ganglions should be investigated as a cause.

Fitzgerald et al. discussed gout affecting the hand and wrist. The medical treatment of gout includes NSAIDs such
as indomethacin or ibuprofen for acute flares, and colchicine and allopurinol for chronic prophylaxis.

Figures A and B represent axial CT and MRI images showing calcification and gouty tophi deposition in the carpal
tunnel floor.

Incorrect Answers:
Answer 1: Aspiration is not a first line treatment for tophaceous gout.
Answer 2: The clinical picture is not suspicious for a malignancy, therefore a biopsy would not be indicated.

48
Answer 3: Night splints would not help diminish the space occupying lesion, in this case, tophaceous gout.
Answer 5: Chronic tophaceous gout that has failed medical therapy may require surgical excision.
20) (OBQ05.246) A 16-year-old football player sustains an injury to his ring finger after making a tackle. A clinical
photograph is shown in Figure A. What is the most likely diagnosis? QID: 1132

A
A. Flexor digitorum superficialis avulsion
B. Central slip rupture
C. Sagittal band rupture
D. Distal extensor tendon rupture
E. Flexor digitorum profundus avulsion

PREFERRED RESPONSE ▼ 5
The photograph demonstrates the inability to flex the ring finger DIP. Based on the mechanism and clinical findings
this injury represents a "rugby jersey finger", which is an avulsion of the flexor digitorum profundus (FDP) tendon.

Tuttle et al reviewed these injuries and concluded treatment for an acute injury is FDP tendon reinsertion. For
chronic injuries, a 2-staged tendon grafting is required.
21) (OBQ09.268) You are seeing a 24-year-old male in the emergency room after he was involved in a knife fight.
He has severed the common digital nerve to the index finger on his dominant hand, with an 8mm gap between
nerve ends. In counseling him about repair, which of the following options is as good as autologous nerve
grafting? 3081
A. Glycolide trimethylene carbonate conduit
B. Collagen conduit
C. Silicone sleeve
D. Primary end-to-end repair
E. Polyglycolic acid conduit

PREFERRED RESPONSE ▼ 2
Repair of segmental nerve loss in the hand using collagen conduits allows for nutrient exchange and accessibility of
neurotrophic factors to the axonal growth zone during regeneration. While the other listed answers have been used,
none has shown the efficacy of collagen conduits or autograft.

Li et al. describe the repair of peripheral nerves with a tubular collagen conduit and review supporting data from in
vitro and in vivo primate studies to this regard.

Bertleff et al. describe the recovery of sensory nerve function after treatment of traumatic peripheral nerve lesions
with a biodegradable poly(DL-lactide-epsilon-caprolactone) neurolac nerve guide, compared to their control of end-
to-end repair, no autologous grafting. They show equal results between primary end-to-end repair and their
synthetic graft.

Waitayawinyu et al. compared 2 synthetic polyglycolic acid conduits to autogenous nerve grafting using
49
histopathologic and neurophysiologic analyses in a segmental defect rat model. They found that collagen conduits
and autografts produced comparable results, which were significantly better than polyglycolic acid conduits.

Video V is a lecture discussing peripheral nerve injury and management.


Illustrations: V
22) (OBQ08.274) Which of the following flexor tendon annular pulleys originate from palmar plates overlying joints?
QID: 660

A. A1, A3, A5
B. A2, A4
C. A1, A2, A4
D. A1, A2, A3
E. A2, A4, A5

PREFERRED RESPONSE ▼ 1
The pulley system governs the moment arm, excursion and joint rotation produced by the flexor tendons. The A2
and A4 pulleys are the most biomechanically important to these functions. A2 and A4 arise from the periosteum of
the proximal half of the proximal phalanx, and the midportion of the middle phalanx, respectively. A1, A3 and A5 are
joint pulleys arising from the palmar plates of the MP, PIP, and DIP joints respectively. C1, C2, and C3 are thin,
condensable, cruciate sections of the flexor sheath which permit the annular pulleys to approximate each other
during flexion.
Illustrations: A
23) (OBQ05.130) A 35-year-old woman reports wrist pain after a fall onto an outstretched hand. On exam, she has
focal tenderness over the wrist snuffbox. A radiograph and CT image are shown in Figures A and B. What is the
proper treatment of her injury? QID: 1016

A B
A. Rest, ice, elevation
B. Removable splint for comfort
C. Thumb spica cast
D. Open reduction, internal fixation
E. Vascularized bone grafting

PREFERRED RESPONSE ▼ 4
The radiograph and CT scan show a displaced scaphoid waist fracture. Optimal treatment is ORIF with screw
fixation.

The usual mechanism of injury to the scaphoid is axial load across a hyperextended wrist. Pain with resisted
pronation, snuffbox tenderness and scaphoid tuberosity tenderness should all raise suspicion for a scaphoid
fracture. AP and lateral X-rays, as well as PA view with the hand in ulnar deviation and an oblique 45 degree view
with the hand in pronation can help to identify the fracture. Bone scan, CT and MRI can also be used to make the
diagnosis if radiographs are indeterminate. ORIF is recommended for any fracture displaced more than 1mm, with a
50
radiolunate angle greater than 15 degrees, with intrascaphoid angle greater than 35 degrees, associated with
perilunate dislocation or with a proximal pole fracture. Optimal treatment is ORIF with screw fixation. For minimally
displaced fractures, percutaneous or mini-open fixation allows minimal dissection and preservation of extrinsic
ligaments.

Incorrect Answers:
Answer 1,2,3: Nonoperative management is not indicated in displaced scaphoid fractures
Answer 5: Vascularized bone grafting is reserved for cases of scaphoid nonunion.

24) (OBQ12.186) A 6-year-old girl sustains transverse amputations through her long and ring fingertips after getting
her hand caught in a lawn mower. She presents to the emergency room 30 minutes after the injury with the
amputated tissue which was placed on ice in a waterproof bag. On physical exam the amputation levels are found to
be 6 millimeters distal to the lunula. The wounds are noted to be fairly contaminated with no evidence of exposed
bone. Skin defects are less than 1 centimeter. Which of the following is the most appropriate management at this
time? QID: 4546

A. Emergent replantation of the amputated parts


B. Revision amputation through the distal interphalangeal joint
C. Thorough irrigation and debridement followed by elective Moberg advancement flaps
D. Thorough irrigation and debridement followed by elective Z-plasty reconstruction
E. Thorough irrigation and debridement, soft dressing application, and follow-up within 1 week

PREFERRED RESPONSE ▼ 5
Distal fingertip amputations can be successfully managed with local wound care and healing by secondary intention
if no bone is exposed and the soft tissue defects are minimal. This is especially true in the pediatric population.

Distal fingertip amputations are common injuries seen in the emergency department. If bone is not exposed, the
wounds can be successfully treated with local wound care and dressing changes, followed by soaks in a hydrogen-
peroxide solution after 7-10 days. Some controversy exists in the pediatric population if the soft tissue loss is > 1
cm, with options for management including a V-Y advancement flap or conservative management with dressing
changes.

Quell et al. review the results of 82 patients with fingertip amputations treated conservatively; 31 of the digits were
treated with primary closure with or without shortening of bone and 54 digits were treated with semiocclusive
dressings. No complications were observed, and all healed fingertips were well padded and painless.

Tupper et al. review sixteen patients with twenty fingertip injuries who underwent V-Y plasty for transverse fingertip
amputations. Sensitivity was 73% of normal, with eight patients reporting hypersensitivity. Contrary to popular belief,
they believe normal sensation following a V-Y plasty is not a reasonable expectation.

Illustration A shows the three levels of fingertip amputations. Zone I is distal to the phalanx; Zone II is distal to the
lunula; and Zone III is proximal to the lunula.

Incorrect Answers:
Answer 1: Emergent replantation is not indicated in distal fingertip amputations.
Answer 2: Revision amputation through the DIP joint could be considered for Zone III injuries
Answer 3: Moberg advancement flaps are considered for volar thumb soft tissue loss.
Answer 4: Z-plasty is considered for soft tissue loss in the webspaces.
Illustrations: A
51
25) (OBQ08.165) The median nerve lies immediately ulnar to which of the following structures at the level of the
distal radioulnar joint? QID: 551

A. Flexor carpi radialis


B. Flexor carpi ulnaris
C. Radial artery
D. Flexor digitorum profundus
E. Pronator teres

PREFERRED RESPONSE ▼ 1
The median nerve sits immediately ulnar to the flexor carpi radialis (FCR). This anatomic relationship is
demonstrated by the fact that a median nerve injury is most likely to be associated with a deep laceration of flexor
carpi radialis (FCR) at the level of the wrist. Additionally, the risk associated in dissecting between the flexor carpi
radialis and palmaris longus is injury to palmar cutaneous branch of the median nerve.

Illustration A shows the relative position of the median nerve to FCR at the level of the pronator quadratus on cross
section anatomy.
Illustration B shows a cross-sectional MRI at the level of the wrist.

26) (OBQ12.210) A 55-year-old male laborer comes in with a chief complaint of clumsiness with his hands for the
past 2-3 months including difficulty using a hammer while at work. On physical examination, he is found to have a
positive Tinel’s sign at the level of the medial elbow. In addition he has persistent small finger abduction/extension
with finger extension and active adduction. An EMG is performed and demonstrates nerve conduction velocities of
35 m/sec. The patient symptoms are most accurately described as QID: 4570
A. Axonotmesis with ischemia origin
B. Axonotmesis with myelin disruption
C. Neurapraxia with ischemia origin
D. Neurapraxia with endoneurium disruption
E. Neurotmesis

PREFERRED RESPONSE ▼ 3
The history and clinical presentation are consistent with ulnar entrapment neuropathy at the level of the cubital
tunnel. This would be classified as a neuropraxia with ischemia origin.

Compression injuries to the peripheral nerves are often the result of micro-vascular dysfunction as the nerves
traverse a high to low pressure gradient. Peripheral nerve injury can be classified as neuropraxia, axonotmesis and
neurotmesis. Compressive neuropathies are typically neuropraxias, with local myelin damage but not compromise of
the major components of the nerve. In axonotmesis, there is Wallerian degeneration and myelin loss distal to the
site of injury. The most severe type is that of neurotmesis. Neurotmesis is composed of a spectrum of injury, in
which all components are affected except for the perineurium or the endoneurium may be intact. The worst form of
neurotmesis is that of nerve transection.

52
Elhassan et al. review the pathophysiology of cubital tunnel syndrome. They report nerve dysfunction results from
ischemic changes secondary to compression. Compressive effects on the nerves can last greater than 24 hours,
even after the source of compression has been removed.

Rempel et al. review the pathophysiology of peripheral nerve compression syndromes. The authors indicate that
deforming pressures to nerves are often the result of stenotic soft tissue canal boundaries. This leads to interference
with local microvasculature of the nerve itself.

Incorrect Answers:
Answer 1, 2: Axonotmesis is considered a second degree nerve injury, characterized by Wallerian degeneration of
axons distal to site of injury. Compression neuropathies are more often neuropraxias (1st degree nerve injury)
Answer 4: Compression neuropathies result from ischemic insult to the nerve
Answer 5: Neurotmesis may be characterized by complete disruption of all components of nerve (as in transection)
or with disruption of all components except for the perineurium or the endoneurium. This is not characteristic of a
compression neuropathy such as cubital tunnel syndrome.

Illustration A demonstrates the Wartenberg sign, where the patient has persistent small finger abduction/extension
resulting from weakness of the 3rd palmar interosseous/small finger lumbrical.
Illustration B reveals clawing which results from over powering of the intrinsic muscles by the extensors; a tenodesis
effect results in flexion of the PIP/DIP joints. This is more severe in ulnar nerve compression at Guyon’s canal.
Illustration C shows the Froment sign, where the FPL attempts to compensate for a deficient pinch, because of
weakness of the adductor pollicis. Illustration D demonstrates atrophy of the 1st dorsal webspace from chronic
compressive changes.
Illustration E demonstrates atrophy of the thenar compartment which is consistent with carpal tunnel syndrome.
Illustrations: V

27) (OBQ11.52) A 22-year-old gymnast with known ligamentous laxity has been treated in the hand therapy clinic
for 6 months for left wrist pain and discomfort. Radiographs of her left wrist are seen in Figures A and B. Which of
the following physical exam findings would be most diagnostic for midcarpal instability? QID: 3475

A B
A. Radial wrist pain with sudden ulnar deviation
B. Tenderness to palpation distal to the ulnar styloid
C. Pain and a clunk on ulnar to radial deviation of the wrist while pressure is held on the scaphoid
D. Pain in the lunate with volar directed pressure on the dorsum of hand
E. Pain and a clunk with axial and palmarly directed forces as the wrist is moved from neutral to ulnar deviation

53
PREFERRED RESPONSE ▼ 5
The clinical situation is consistent with midcarpal instability. The most common finding on physical examination is a
clunk as the wrist is moved from a neutral position and forearm pronation to ulnar deviation with an axial and
palmarly directed load. Carpal instability is complex condition marked by abnormal kinematics in the carpus. Carpal
instability dissociative (CID) is marked by intrinsic ligamentous disruption. Carpal instability non-dissociative (CIND)
is marked by extrinsic ligamentous disruption between carpal rows or between the proximal row and distal radius.
Included in CIND is midcarpal instability (MCI) and radiocarpal instability. Radiographs typically show a mild VISI
deformity or no abnormalities as in Figures A and B. Videofluorscopy is diagnostic as the proximal row assumes a
volar, flexed position, then snaps into extension the wrist is moved into ulnar deviation.

Lichman et al provided an overview and historical perspective of carpal instability. Carpal instability is divided into
dissociative and non-dissociative. They concluded that there are several causes and patterns of carpal instability
leading to carpal subluxation. An in-depth understanding is required for proper treatment.

Apergis et al described 14 cases of midcarpal instability treated with ligamentous reefing of the midcarpal joint and
or the radiolunate joint. They reported excellent results in eight cases, good in five cases, and fair in one case.

Incorrect answers:
Answer 1: A positive Finkelstein's maneuver is radial-sided pain with a clinched thumb and ulnar deviation of the
wrist
Answer 2: A positive Fovea sign is found with pain on palpation distal to the ulnar styloid
Answer 3: Watson's scaphoid shift is pain and a clunk on ulnar to radial deviation of the wrist while pressure is held
on the scaphoid tubercle.
Answer 4: Describes pain in the lunate as in Keinbock's.

28) (OBQ08.21) A 35-year-old female office worker reports 6 months of deep aching pain that is worse at night on
her lateral dominant elbow. The pain also worsens with repetitive movements. On physical exam, the patient has
tenderness located 4cm distal to the lateral epicondyle over the mobile wad, and she has subtle weakness of the
wrist extensors. Extending her long finger against resistance with a flexed wrist is very painful for her. She also
complains of her pain worsening at night. What is the most likely diagnosis?QID: 407

A. Radiocapitellar Arthritis
B. Radial tunnel syndrome
C. Carpal tunnel syndrome
D. Lateral epicondylitis
E. Intersection Syndrome

PREFERRED RESPONSE ▼ 2
The patient has radial tunnel syndrome, which often presents with insidious onset of pain and tenderness several
centimeters distal to the lateral epicondyle, and pain elicited with active extension of the long finger against
resistance can help differentiate the condition from lateral epicondylitis.

Radial tunnel syndrome is a compressive neuropathy that can occur between the mobile wad laterally and the
biceps aponeurosis and brachialis insertion medially as the nerve courses over the radiocapitellar joint into the
forearm. Patients usually have diffuse pain over the site of the radial tunnel, sometimes have radiating pain in the
distribution of the superficial radial nerve, and occasionally have subtle weakness or fatigue of the wrist and extrinsic
finger extensors. Initial treatment should include conservative measures such as stretching, activity modification,
and NSAIDS; Injections can be performed for both diagnostic and therapeutic reasons. Surgical intervention is
indicated if pain persists despite exhausting conservative measures. The most common anatomic causes of radial
tunnel syndrome are fibrous adhesions between the brachialis and brachioradialis, the Leash of Henry (radial
54
recurrent vessels), the fibrous edge of the ECRB, the arcade of Fröhse (supinator arch), and fibrous bands of the
leading edge of the supinator muscle.

Dang et al. discuss compression neuropathies of the upper extremity in their 2009 review article. They highlight the
importance of the clinical exam in diagnosing radial tunnel syndrome, especially the location of pain, which is distal
to that of lateral epicondylitis. Additionally ruling out other less common diagnoses on the differential can be assisted
by EMG (radiculopathy or plexopathies), MRI (tumor or other causes of mass effect), and diagnostic injections.

Illustration A shows the anatomy of the five common sites of compressing in the radial tunnel.

Incorrect Answers:

1. Radiocapitellar arthritis would not be antagonized by stretch of the common extensors of the wrist
3. Carpal tunnel syndrome is diagnosed by evidence of nerve compression of the median nerve at the wrist and
should not be associated with pain near the origin of the wrist extensors
4. Lateral epicondylitis can very much mimic radial tunnel syndrome; however, the location of the pain and
tenderness on exam can be a very helpful
5. Intersection syndrome is a chronic tenosynovitis of the ECRL and ECRB characterized by pain at the intersection
of the 1st and 2nd dorsal compartments of the wrist

Illustrations: A
29) (SBQ07.40) A 44-year-old man presents with ulnar-sided right wrist pain and mild constant tingling in the fourth
and fifth digits after injuring his wrist while playing golf. Although pain and function have improved with conservative
treatment 6 months following the injury, he still reports difficulty with his golf game. Which of the following should
initially be obtained in this patient to aide in the diagnosis? QID: 1425

A. Bone scan of the wrist and hand


B. EMG study of the affected extremity
C. Carpal tunnel view radiograph
D. CT scan of the distal forearm and wrist
E. Contrast enhanced magnetic resonance angiogram

PREFERRED RESPONSE ▼ 3
This patients clinical presentation is most consistent with a chronic hook of the hamate fracture, which should
initially be evaluated with a carpal tunnel view radiograph. Hook of the hamate fractures typically are associated with
pain localized to the hypothenar eminence, and chronic cases can be associated with neuropathy of the ulnar nerve.
Excision of the hook through the fracture site usually yields satisfactory results in the presence of chronic injuries.

Parker et al treated five patients with six hook of the hamate fractures over an eight year period. All patients
ultimately underwent hook resection and returned to their previous level of activity in 6 to 8 weeks after surgery
without loss of function. Based on their case series, they concluded that the entire hook should be resected to the
base of the hamate as the primary form of treatment in hook of the hamate fractures.

Illustration A: Patient positioning for carpal tunnel radiograph-wrist is extended 70 degrees, and beam is angled 25-
30 deg to the long axis of the hand(arrow).

Illustration B: Carpal tunnel view radiograph demonstrates a fracture at the base of the hook of the hamate(black
arrow) and normal pisotriquetral joint space.

Incorrect Answers:
55
1-Bone scans are not typically indicated in the diganostic setting of acute or chronic hook of the hamate fractures.
2-Imaging should be obtained to rule out bony injury prior to obtaining an EMG study.
4-CT scans can used to confirm the diagnosis of a hook of the hamate fracture after obtaining a carpal tunnel view
radiograph.
5-Contrast enhanged MRA of the wrist is typically used to diagnose hypothenar hammer syndrome or other vascular
abnormalities.

30) (OBQ10.6) The parents of a newly adopted 3-year-old boy bring the child to the office for evaluation of his
thumb. A clinical photograph is provided in figure A. Which of the following is the most important factor in
determining thumb reconstruction versus ablation and pollicization? QID: 3094

A
A. Stability of the carpometacarpal joint
B. Functional ROM of the wrist
C. Functional ROM of the index, middle, ring, and small fingers
D. Skin contracture of the first web space
E. Absence of a thumb interphalangeal joint

PREFERRED RESPONSE ▼ 1
The clinical photograph demonstrates a hypoplastic thumb. The incidence of thumb hypoplasia is 1 in every 100,000
births and associated anomalies including radial aplasia, thrombocytopenia, and renal/cardiovascular/CNS
anomalies are frequent. Stability of the carpometacarpal joint is essential for success of thumb reconstruction
procedures. If CMC stability is deficient, then ablation and pollicization is preferred.

Light et al describe the evaluation and surgical technique involved in treating the hypoplastic thumb. They note that
severe thumb hypoplasia and aplasia are best treated by thumb ablation and pollicization of the index finger.

56
31) (OBQ04.97) A 35-year-old butcher inadvertently lacerates his ring finger FDP tendon at the level of the DIP joint
which is subsequently repaired. Following the operation he notes the inability to fully flex his long and small fingers
at the DIP joints with attempted fist clenching as well as a weak grip. Which of the following intraoperative
maneuvers was likely responsible for this? QID: 1202
A. FDP reconstruction with a long tendon graft
B. FDS to FDP transfer at level of the A2 pulley
C. Inadequate repair of the C3, A4 and A5 pulleys
D. Distal advancement of lumbricals
E. Overtensioning of the FDP tendon

PREFERRED RESPONSE ▼ 5
The clinical presentation is most consistent with the quadrigia effect which is caused by overtensioning of the FDP
tendon during surgical repair.

The FDP tendons share a common muscle belly and have many interconnections. Overtensioning one tendon has a
reciprocal effect on the length-tension curve of the remaining three muscle-tendon units, weakening grip strength in
these digits.

Malerich et al performed a cadaveric study looking at FDP advancement on hand function. They determined
advancement >1cm can lead to an imbalance of muscle function in the profundus system.

Kaufmann et al. studied maximal grip strength and point of contact in the extrinsic system. They determined that
FDP grip strength was optimized when the FDP point of contact was at the DIP. Thus moving the FDP insertion
point distal or advancing a lacerated FDP tendon leads to a decrease in grip strength.
32) (OBQ09.227) A 35-year-old professional football player complains of severe wrist pain after making a tackle. He
reports paresthesias in his thumb and index finger. AP and lateral radiographs of the wrist are shown in figures A
and B respectively. What is the most appropriate next step in management? 3040

A B
A. short arm thumb spica cast
B. long arm thumb spica cast
C. urgent closed reduction and splinting
D. MR arthrogram of the wrist to assess ligamentous injuries
E. bone scan to assess vascularity

PREFERRED RESPONSE ▼ 3
This patient is presenting with a perilunate dislocation with carpal tunnel symptoms. The most important next step in
treatment is reduction of the dislocation. This is generally performed in the emergency room, and if unsuccessful,
57
immediate reduction and stabilization in the operating room is indicated.

Kozin et al note that these injuries can be overlooked and have variable propagation patterns through the
carpus/carpal ligaments. This patient has a radial styloid fracture due to avulsion of the radiocarpal ligaments.

Melone et al note that these injuries were historically treated with closed reduction and pinning, but more recently
the trend is for open reduction and fixation, for optimal anatomic restoration.

Figure A is an AP radiograph that shows obvious scapholunate diastasis due to a perilunate dislocation. Figure B
shows the 'empty teacup' sign due to the empty articulation of the distal lunate.
33) (OBQ11.246) A 68-year-old female office assistant reports left thumb pain that has progressively worsened over
the past 2 years. She is left hand dominant and reports difficulty with opening jars and holding a coffee cup. On
examination of the left hand she has a positive thumb carpometacarpal grind test and has a fixed deformity at the
thumb metacarpalphalangeal joint. Figure A demonstrates the left hand grasping an object and Figure B shows a
radiograph of the left thumb. What is the most appropriate next step in treatment? QID: 3669

A B
A. Carpometacarpal joint fusion and metacarpophalangeal joint volar capsulodesis
B. Carpometacarpal joint resection arthroplasty and metacarpophalangeal joint volar capsulodesis
C. Carpometacarpal joint resection arthroplasty and metacarpophalangeal joint fusion
D. Carpometacarpal joint resection arthroplasty and temporary metacarpophalangeal joint percutaneous pin fixation
E. Carpometacarpal joint fusion and metacarpophalangeal joint fusion

PREFERRED RESPONSE ▼ 3
The patients history, examination, and images are consistent with thumb CMC (basilar) joint arthritis with associated
MCP joint arthritis. At the MCP joint there is hyperextension of the thumb metacarpophalangeal (MCP) joint and
adduction involving the first web space of the hand (Z deformity). Arthrodesis of the MCP joint is the treatment of
choice when thumb MCP hyperextension exceeds 40°, the deformity is not passively correctable, or advanced
degenerative changes are noted to affect the articulation.

The review article by Armbruster and Tan state that when MCP joint hyperextension is:
0° to 10°= Surgical intervention is not necessary when MCP hyperextension is less than 10°.
10° to 20°= Percutaneous pinning of the MCP joint in 25° to 35° of flexion for 3-4 weeks may be performed
independently or as an adjunct to EPB transfer.
20° to 40°= Capsulodesis of the volar aspect of the MCP joint is recommened to provide a check rein for
hyperextension and Sesamoidesis has also been investigated as an adjunctive procedure.

Cooney et al performed a Level 4 review of their CMC arthroplasty patients and found 15 patients with 17 revision
arthroplasties in the treatment of mechanical pain related to instability or bone impingement. The revisions included
soft-tissue interposition alone or soft-tissue interposition with ligament reconstruction and found that this provided

58
satisfactory patient outcomes in more than 75% of the cases.

Illustration A depicts the forces accounting for the observed adduction and hyperextension deformities. The
arrowhead indicates the direction of subluxation of the base of the thumb metacarpal (due to incompetent volar beak
ligament). The arrow represents the force vector of the EPB potentiating the MCP hyperextension deformity
Illustrations: A
34) (OBQ10.14) A 50-year-old woman sustains an open both bone forearm fracture seen in Figure A and undergoes
the treatment seen in Figure B. During surgery the posterior interosseous nerve was transected and primary repair
was attempted. One year following surgery the patient continues to have no posterior interosseous nerve function.
Which of the following treatments will best restore function? QID: 3102

A B
A. Sural nerve grafting to the posterior interosseus nerve
B. Wrist fusion with transfer of the flexor carpi ulnaris to the finger extensors
C. Transfer of the flexor carpi radialis to extensor digitorum and the palmaris longus to the extensor pollicis longus
D. Transfer of the pronator teres to the wrist extensors and the palmaris longus to the finger extensors
E. Transfer of the flexor carpi ulnaris to the wrist extensors and the palmaris longus to the extensor pollicis longus

PREFERRED RESPONSE ▼ 3
Figures A and B show a pre and post-operative radiograph of a both bone forearm fracture. The posterior
interosseus nerve is at risk during surgical approaches to this fracture pattern and care should be taken.

Ropars et al retrospectively reviewed 15 patients who underwent treatment for radial nerve and PIN palsy. For PIN
palsy, they concluded the most beneficial transfers included transferring the flexor carpi radialis to the finger
extensors (to restore finger extension) and palmaris longus to the extensor pollicis longus (to restore extension of
the thumb). In contrast with a radial nerve palsy, with a PIN palsy the patient has adequate wrist extension due to
intact ECRL (providing radial wrist extension) supplied by the radial nerve proximal to the PIN.

Ustün et al in their cadaveric studies were able to show that it is possible to perform posterior interosseous nerve
neurotization via the median nerve.

Hirachi et al reviewed the results of 17 traumatic PIN palsies that were treated either with nerve repair, tendon
transfers, or nonoperatively. They noted that associated muscle damage resulted in poorer results.
The muscles involved in the suggested transfer (FCR, ED, PL, EPL) are shown in illustration A-D.
Illustrations: A B C D

59
35) (OBQ12.191) Which of following malformations is most commonly associated with Poland's syndrome? 4551

A. Figure A
B. Figure B
C. Figure C
D. Figure D
E. Figure E

PREFERRED RESPONSE ▼ 4
Figure D demonstrates symbrachydactyly which is most commonly associated with Poland's syndrome.

Poland's syndrome is a rare birth defect characterized by underdevelopment or absence of the chest muscle in
conjunction with ipsilateral symbrachydactyly. Poland syndrome most often affects the right side of the body, and
occurs more often in males than in females.

Ireland et al. reviewed 43 consecutive cases of Poland's syndrome, and reviewed the relevant literature up to that
point. The authors state that the clinical features are variable but always include congenital aplasia and syndactyly,
and the right side is affected more than the left. They also note that although the hand remains hypoplastic and
functional capacity is limited by the inherent skeletal anomalies, surgical treatment improves functional capacity and
cosmetic appearance in the majority of patients.

Van Heest summarizes normal formation and growth of the upper limb as a basis for understanding malformation,
with the goal of providing a basic understanding of the evaluation necessary for appropriate counseling and referrals
for treatment of the child with hand and upper extremity congenital deformities.

Incorrect Answers:
Answer 1: Figure A demonstrates the classic deformity associated with Apert's syndrome, which commonly consists
of complex syndactyly.
Answer 2: Figure B demonstrates the lymphedema characteristic of amniotic band syndrome.
Answer 3: Figure C demonstrates radial club hand which can be associated with VATER syndrome.
Answer 5: Figure E shows and example of thumb polydactyly which is associated with a variety of syndromes, but
not typically Poland's.
36) (OBQ07.64) A 34-year-old seamstress was diagnosed with Parsonage-Turner brachial neuritis in the right upper extremity 1
month ago. She has weak forearm pronation with the elbow in the flexed position. She denies any current sensory changes. A
clinical image of her hands attempting to make a clenched fist are shown in Figure A. Which of the following most likely
represents her diagnosis and treatment? QID: 725

A
60
A. Anterior interosseous nerve syndrome treated with observation
B. Posterior interosseous nerve syndrome with release of the Arcade of Frohse
C. Pronator syndrome with surgical release of the lacertus fibrosis
D. Anterior interosseous nerve syndrome with surgical release of Gantzer's muscle
E. Posterior interosseous nerve syndrome treated with observation

PREFERRED RESPONSE ▼ 1
This patient presents with anterior interosseous nerve (AIN) syndrome and is often seen in conjunction with brachial neuritis
(Parsonage-Turner Syndrome).

AIN syndrome leads to motor palsies of the flexor pollicis longus and the two radial profundus tendons leading to the clincal
image shown in Figure A. The pronator quadratus is also involved and can be tested with the elbow held in a flexed position to
neutralize the humeral head of the pronator teres muscle. No sensory changes occur and electromyographic (EMG) and
nerveconduction (NCV)studies are often helpful in establishing the diagnosis.

The Level 5 review article by Rodner et al. discusses that anterior interosseous nerve syndrome usually resolves with time,
particularly if the lesion is secondary to neuritis. Observation for 3 to 6 months with splinting at 90 degrees is favored before
surgical treatment.

Parsonage and Turner reported in their 1948 landmark article, on several cases of isolated AIN palsy caused by neuralgic
amyotrophy (ie, Parsonage-Turner syndrome [PTS] or brachial plexus neuritis). PTS symptoms may include pain and motor
and/or sensory dysfunction in one or in multiple peripheral nerves of the upper extremity.
figA shows spinners sign (eclusion of thumb and indx when making a fist)
37) (OBQ12.182) Which of the following statements is true regarding zone II flexor tendon injuries? QID: 4542
A. At this level, FDS and FDP are located within separate tendon sheaths
B. FDS repair has not been shown to improve outcomes
C. Improved gliding is seen with repair of 1 slip of FDS compared to repairing both slips
D. Repairing FDS does not affect post-operative digit strength
E. FDP repair has not been shown to improve outcomes

PREFERRED RESPONSE ▼ 3
In zone II flexor tendon injuries, repairing only one slip of FDS has been shown to improve gliding when compared to repair of
both slips.

Zone II flexor tendon injuries have notoriously had poor outcomes secondary to high rates of adhesion formation at the pulleys.
However, new advances in post-operative rehabilitation have significantly improved outcomes to the point where it is no longer
considered "no man's land." Management of the FDS has been a source of controversy. In the past, the FDS was occasionally
excised to theoretically make more room for the FDP. This has now been largely abandoned and the FDS is repaired whenever
possible. Whether or not to repair both slips of FDS remains controversial, with in vitro data suggesting that gliding resistance is
improved if only one slip is repaired.

Zhao et al. review the effect of partial vs. complete FDS excision following repair of FDP for zone II flexor tendon injuries.
Preserving the whole FDS resulted in a significantly larger increase in gliding resistance after FDP repair than did full or partial
FDS removal, which were not significantly different from each other.

Illustration A shows the zones of flexor tendon injury. Note that zone II injuries occur between the FDS insertion and the distal
palmar crease. Illustration B shows the anatomy of the flexor tendons in detail. Video V shows a technique for repair of zone II
injuries.

Incorrect Answers:
Answer 1: In zone II, the FDS and FDP are located within the same tendon sheath.
Answer 2: While the FDS was excised in the past, clinical outcomes have recently been shown to be improved with repair of
either one or both slips.
Answer 4: Repairing FDS has been shown to increase digit strength.

61
Answer 5: Repair of FDP has been shown to improve long-term clinical outcomes.
Illustrations: A B V

38) (OBQ12.158) A 58-year-old man presents with right middle finger swelling. Radiographs and an MRI of his hand are shown
in Figure A and B. Tissue biopsy and staging investigations are performed. The tumor is then resected en bloc with the middle
metacarpal, which is amputated 1.5cm from the carpometacarpal joint. The attached deep transverse intermetacarpal ligaments
are sacrificed. To prevent scissoring of the remaining digits and small objects falling through the gap between index and ring
fingers, which of the following procedures should be performed? QID: 4518

A B
A. iliac crest bone grafting
B. ring metacarpal transposition
C. second toe transfer
D. index metacarpal transposition
E. suture of deep transverse intermetacarpal ligaments

PREFERRED RESPONSE ▼ 4
Index metacarpal transposition is indicated to reduce the space left between the index and ring finger. In this case, the middle
ray is amputated because of malignancy.

With amputation of the middle or ring metacarpals, small objects fall through the gap and the adjacent fingers scissor. For single
central ray defects, techniques to reduce the gap include transposition of the index finger (for middle ray amputation), small
finger (for ring ray amputation), complete removal of the metacarpal (without leaving a proximal metacarpal base stump) to allow
the bases of index and ring metacarpals to migrate together and reconstruction of the deep transverse metacarpal ligament. The
technique of index transposition may vary depending on the osteotomy (straight vs step-cut) and fixation (K wires vs plate) as
seen in the illustrations below.

Muramatsu et al. describe bony transposition for reconstruction after ray amputation for malignancy. The advantage is
immediate closure of the space. The disadvantages include prolonged postoperative immobilization until union, malrotation
(leading to scissoring), mal-tension of tendon (because of different metacarpal heights), and delayed or nonunion.

Lyall et al. advocate total middle ray amputation. They believe that leaving the metacarpal base behind leads to difficulty in
aligning the adjacent rays as the index and ring must angulate over the bony obstruction to close the distal gap, leading to
scissoring. They believe that index transposition leaves an abnormally wide 1st web space and a remnant 2nd metacarpal
stump that can protrude dorsally.

Figure A is an AP radiograph of the right hand showing a destructive lesion of the proximal phalanx of the middle finger abutting
the metacarpophalageal joint. Figure B is a STIR coronal MRI image showing the tumor mass extending into surround soft
tissue. Illustration A is a diagram showing index transposition for middle ray amputation using a straight osteotomy and crossed
K-wires. Illustration B is a diagram showing index transposition using a step-cut osteotomy and multiple K-wire fixation to the
adjacent metacarpals. Illustration C is a diagram showing index transposition using a straight osteotomy and plate fixation.
Illustration D is a diagram showing an alternative technique of suturing deep transverse metacarpal ligaments together to close
the gap.

62
Incorrect Answers
Answer 1: Iliac crest bone grafting is not indicated. Together with a skin flap, iliac crest bone grafting may be used for thumb
reconstruction.
Answer 2: Ring metacarpal transposition is not indicated. Transposing a metacarpal from ring-to-long position leads to a new
gap where the ring digit used to be. For ring metacarpal defects, the adjacent small metacarpal is transposed instead.
Answer 3: Second toe transfer may be indicated for thumb reconstruction, or digital reconstruction for multiple digit losses. It
involves microsurgical repair. For solitary central ray loss, a lesser procedure is preferred.
Answer 5: Gap closure by suturing deep transverse metacarpal ligaments together is only possible in the absence of
malignancy, where the ligaments are not compromised. In this case, the ligaments were resected together with the tumor.

39) (OBQ08.9) A 31-year-old mother of a 2-month-old infant complains of radial sided wrist pain. Corticosteroid injections should
be directed into what anatomic area? QID: 395
A. First carpometacarpal joint
B. Carpal tunnel
C. First dorsal compartment near the radial styloid
D. A1 pulley of thumb
E. At the crossing of the first and second dorsal compartments

PREFERRED RESPONSE ▼ 3
There is an association between the postpartum state and de Quervain’s tenosynovitis. De Quervain’s is a pathologic process of
the 1st dorsal (extensor) compartment which contains the extensor pollicis brevis and abductor pollicis longus tendons. The best
choice is #3 because of the very common and known association of postpartum state and de Quervain’s as well as the potential
for resolution with appropriately placed steroid injection. Answer #1 refers to basal joint arthritis which is typically seen in older
patients. Answer #2 refers to carpal tunnel syndrome, which would present with paresthesias in the median nerve distribution.
Answer #4 refers to a trigger thumb. Answer #5 alludes to intersection syndrome which is generally more proximal to the wrist
and results from inflammation at crossing point of 1st dorsal compartment (APL and EPB) and 2nd dorsal compartment (ECRL,
ECRB). To review, the wrist extensor compartments (from radial to ulnar) are: 1) APL & EPB; 2) ECRL & ECRB (common radial
wrist extensors); 3) EPL; 4) EIP & EDC; 5) EDM; 6) ECU.
40) (OBQ07.78) A 30-year-old female reports 5 months of wrist pain after a fall onto her wrist. A radiograph is shown in Figure
A. If untreated, which of the following is least likely to occur during the natural progression of the disease process? QID: 739

A
A. Radial styloid osteophyte
B. Radioscaphoid arthritis
C. Midcarpal arthritis
D. Scapholunate arthritis
E. Radiolunate arthritis

63
PREFERRED RESPONSE ▼ 5
Radiographs show a scaphoid non-union which can lead to Scaphoid Nonunion Advanced Collapse (SNAC wrist) and
progressive arthritis. The natural history of degenerative changes first occurs at the radioscaphoid area and progresses to
pancarpal arthritis. All of the answers above are features of a SNAC wrist with radiolunate arthritis only occurring at the very end
stages of disease.

In the cited reference by Schuind et al, they found that professional heavy work, age of the nonunion of over 5 years, associated
radial styloidectomy, and duration of postoperative immobilization were associated with a significantly decreased likelihood of
healing of the scaphoid nonunion with operative treatment.

The study by Soejima et al found that distal scaphoid resection produces a satisfactory clinical outcome and should be
considered one of the surgical options for patients with long-standing scaphoid nonunion with either radioscaphoid or intercarpal
degenerative arthritis.
41) (OBQ12.126) A 45-year-old male smoker presents with the clinical appearance shown in Figure A. Which of the following
statements is true regarding his condition? QID: 4486

A
A. Anticoagulation with aspirin has been shown to decrease the incidence of amputation
B. Arteriography is useful in the diagnosis of his condition
C. Prophylactic amputation of unaffected digits leads to improved patient outcomes
D. The condition is usually painless
E. Involves proximal vessels first and distal vessels last

PREFERRED RESPONSE ▼ 2
The clinical description and photograph are most consistent with a diagnosis of Buerger's disease, or thromboangiitis obliterans.
Arteriography is the best method for diagnosis of this condition.

Buerger's disease is an inflammatory occlusive disorder of small and medium-sized vessels of the digits most frequently
occurring in male smokers. The condition may mimic other autoimmune and vascular diseases. If the diagnosis is uncertain,
arteriogram is the study of choice. The only treatment that has been found to reduce the risk of amputation is the cessation of
smoking.

Phillips et al. review vascular conditions of the upper extremity. They discuss that patients with Buerger's disease present with
rest pain, claudication, and ulceration, and that cessation of smoking decreases disease progression and the incidence of
amputation.

Figure A shows the classic clinical appearance of a patient with Buerger's disease. Illustration A shows an arteriogram with
"corkscrew" arteries that result from vascular destruction. In the appropriate clinical setting, this finding is diagnostic of Buerger's
disease.

Incorrect Answers:
Answer 1: Anticoagulation has not been shown to halt progression of Buerger's disease.
Answer 3: Prophylactic amputation of uninvolved digits has not been described for the treatment of Buerger's disease.
Answer 4: Buerger's disease is frequently painful.
Answer 5: Buerger's disease involves distal vessels first and proximal vessels last.

Illustrations: A

64
42) (OBQ10.151) Which of the following congenital hand deformities displayed in figures A-E is more prevalent in patients of
African-American ancestry? QID: 3239

A. Figure A
B. Figure B
C. Figure C
D. Figure D
E. Figure E

PREFERRED RESPONSE ▼ 2
Image B is consistent for postaxial polydactyly, which is more prevalent in patients of African-American ancestry.

The cohort study by Woolf found the incidence of postaxial polydactyly in African americans is 12.42 per 1,000 (1.2%) compared
to the Caucasian incidence of 0.91 per 1,000 (0.09%). If postaxial polydactyly is found in a patient of Caucasian ancestry then
further workup for underlying syndromes (chondroectodermal dysplasia or Ellis-van Creveld syndrome) is needed.

The article by Orioli is a case-control study that hypothesizes that a sex-linked recessive modifier gene occurs more frequently in
African americans and this gene then promotes the autosomal dominant polydactyly gene.

Incorrect Answers: Constriction band syndrome or amniotic band syndrome is a type of pseudosyndactyly (Figure A) and is not
the result of failure of differentiation during embryogenesis, but a result of injury by bands after the fingers are formed. Preaxial
polydactyly (Figure C) is more common in caucasians and is usually sporadic except for triphalangism which is associated with
Holt-Oram and Fanconi's Anemia. Syndactyly (Figure D) is defined as an abnormal interconnection between adjacent digits and
syndactyly variations are associated with Apert syndrome and Poland syndrome. Macrodactyly (Figure E) represents overgrowth
of all structures of the involved digit and is associated with neurofibromatosis and Klippel-Trenaunay-Weber syndrome.
43) (OBQ12.245) A 50-year-old patient presents with stiffness in her hand. A clinical photo is shown in Figure A. During surgical
exposure, the neurovascular bundle is identified and dissected. What is the clinically most important pathologic structure to
identify and what is its location relative to the neurovascular bundle in the digit? QID: 4605

A
A. Spiral cord which is central and superficial to the neurovascular bundle
B. Central cord which is midline and superficial to the neurovascular bundle
C. Retrovascular cord which is central and superficial to the neurovascular bundle
D. Spiral cord which is lateral and deep to the neurovascular bundle
E. Central cord which is lateral and deep to the neurovascular bundle

65
PREFERRED RESPONSE ▼ 4
Based on clinical findings, the patient has evidence of Dupuytren’s contracture affecting her ring finger. Relative to the
neurovascular bundle, the spiral cord will lie lateral and deep.

Dupuytren’s disease is a benign hand condition characterized by pathologic nodules and cords of existing fascial bands. The
most clinically relevant structure in Dupuytren's disease, is the spiral cord. The spiral cord is the result of pathology of 4
structures: the middle layer of the pretendinous band, the spiral band, the lateral digital sheet, and Grayson's ligament(g pls).
The spiral cord is found predominantly at the palmodigital transition. The spiral cord displaces the neurovascular bundle centrally
and superficially.

Benson et al. review the etiology, pathophysiology and treatment options for Dupuytren’s contracture. They highlight that while
the pretendinous band is located volar and central to the neurovascular bundle in the palm, the spiral band and lateral digital
sheath cause the neurovascular bundle to be displaced superficially and volarly as they become pathologically affected.

Black et al. review the pathoanatomy, diagnosis and management of Dupuytren's disease. They note that the spiral cord lies
superficial to the neurovascular bundle proximal to the MCP joint. Distal to the MCP joint it passes deep to the bundle. At that
location, the spiral cord lies lateral to the neurovascular bundle as the lateral digital sheet becomes involved

Figure A demonstrates the cord formation that is characteristic of the pathologic Dupuytren’s condition. It is the central cord that
causes contracture of the MCP, whereas the retrovascular and spiral cords cause contractures of the DIP and PIP respectively.
Illustration A shows the relationship of spiral cord formation in Dupuytren's disease relative to the normal anatomy of the palmar
fascia. The structures implicated in the formation of the spiral cord are the pretendinous band, the spiral band, the lateral digital
sheet, and Grayson's ligament. Cleland's ligament, more dorsally located, is spared in Dupuytren's disease. The neurovascular
bundle is displaced superficially and towards the midline, as the pathological cord spirals around. Illustration B shows the
presence of other affected structures, including the natatory ligament and the central band. The central band is an extension of
the pretendinous cord and attaches to the base of the middle phalanx. It may insert onto the tendon sheath of the flexor tendon
at this level. Formation of natatory cords cause webspace contractures. Formation of central cords lead to flexion contractures of
the PIP. Illustration V is a video that provides an educational overview of Dupuytren's.

Incorrect Answers:
Answer 1: The spiral cord is the most important clinically, but displaces the neurovascular bundle centrally/superficially.
Answers 2, 5: The central cord is a structure found in the midline. It is an extension of the pretendinous cord and attaches to the
base of the middle phalanx. It leads to a PIP flexion contracture, but does not affect the neurovascular bundle.
Answer 3: The retrovascular cord forms from digital fascia found dorsal to the neurovascular bundle. This may may lead to
displacement of the neurovascular bundle centrally and superficially. In conjunction with the lateral cord, it may lead to the
formation of hyperextension contractures of the DIP
Illustrations: A B V
44) (OBQ10.61) A 32-year-old carpenter complains of progressively worsening wrist pain for the last 2 months. He denies any
recent history of trauma to the wrist or hand. An MRI is obtained and a representative image is provided in Figure A. Which of
the following surgical interventions is thought to be effective for this condition by inciting a local vascular healing response? 3149

A
A. Wrist fusion
B. Ulnar shortening osteotomy
C. Distal radius core decompression
D. Proximal row carpectomy
E. Scapholunate ligament reconstruction

66
PREFERRED RESPONSE ▼ 3
This clinical scenario and imaging studies are consistent with Kienbock's disease, avascular necrosis of the lunate, in the pre-
collapse stage. Core decompression of the distal radius is an accepted treatment for Kienbock's disease. The procedure creates
a local vascular healing response facilitating vascular recovery prior to collapse and degeneration of the lunate. Other
acceptable interventions include revascularization with a pedicled graft and joint leveling procedures such as a radial shortening
osteotomy. The radial shortening osteotomy is ideal for patients with negative ulnar variance who experience greater loads
through the radiolunate fossa.

Sherman et al performed a cadaveric study demonstrating minimal change in the distribution of force between the radiocarpal
fossa and ulnocarpal fossa following core decompression of the distal radius.

Illarramendi et al reviewed 22 cases of Kienbock's treated with radial and ulnar metaphyseal core decompression. No surgical
complications occurred, and 20 of 22 reported satisfactory clinical outcomes while one patient developed intercarpal arthritis.

Incorrect Answers:
1. Proximal row carpectomy and wrist fusion would be options for the collapsed and degenerative lunate.
2. Ulnar shortening osteotomy and scapholunate ligament reconstruction are incorrect as they do not address the pathology of
Kienbock's.
4. Proximal row carpectomy and wrist fusion would be options for the collapsed and degenerative lunate.
5. Ulnar shortening osteotomy and scapholunate ligament reconstruction are incorrect as they do not address the pathology of
Kienbock's.

45) (OBQ09.146) Which of the following upper extremity congenital anomalies, represented in the figures below, if found in an
individual of Caucasian descent requires a work-up for chondroectodermal dysplasia or Ellis-van Creveld syndrome? 2959

A. Figure A
B. Figure B
C. Figure C
D. Figure D
E. Figure E

PREFERRED RESPONSE ▼ 2
Postaxial polydactyly is rare in Caucasian individuals and deserves further workup for underlying syndromes. Postaxial
polydactyly is ten times more common in African Americans and does not require further workup. Constriction band syndrome or
amniotic band syndrome is a type of pseudosyndactyly(Figure A) and is not the result of failure of differentiation during
embryogenesis, but a result of injury by bands after the fingers are formed. Preaxial polydactyly (Figure C) is more common in
Caucasians and is usually sporadic except for triphalangism which is associated with Holt-Oram and Fanconi's Anemia.
Syndactyly (Figure D) is defined as an abnormal interconnection between adjacent digits and syndactyly variations are
associated with Apert syndrome and Poland syndrome. Macrodactyly (Figure E) represents overgrowth of all structures of the
involved digit and is associated with neurofibromatosis and Klippel-Trenaunay-Weber syndrome. The referenced article by Kozin
is an excellent Current Concepts Review that summarizes upper limb embryogenesis and the most common anomalies.

67
46) (OBQ10.113) When surgically treating a trigger finger in a child, what structure may need to be released in addition to the A-
1 pulley? QID: 3207

A. One or both limbs of the sublimis tendon


B. A-4 pulley
C. Lumbrical origin
D. Dorsal interosseous insertion
E. Anomalous insertion of the MCP joint collateral ligament

PREFERRED RESPONSE ▼ 1
Unlike adults, release of the A-1 pulley in a pediatric trigger finger alone may not resolve triggering symptoms. Trigger finger in
the child may be associated with a more proximal decussation of the FDS tendon, nodules in either the FDS or FDP tendon, a
thickened A-2 pulley, or a tight A-3 pulley. Cardon et al looked at 16 pediatric pts with 18 trigger fingers and found that 6 fingers
continued to trigger after A-1 pulley release. The sublimis decussation and A-3 pulley were found to be the most common cause
of this persistent triggering. Bae et al looked at 23 pediatric trigger fingers and found that triggering was noted to occur at the
level of the FDS tendon decussation in half the cases. The conclusion was made that all pediatric trigger fingers should be
treated with A-1 pulley release and resection of a single FDS tendon slip. Illustration A shows normal decussation of the FDS
tendon near the level of the A2 pulley. The FDS decussation may be found to be more proximal in pediatric trigger fingers,
necessitating release.
Illustrations: A
47) (OBQ08.34) All of the following are contents of the carpal tunnel EXCEPT: QID: 420
A. Flexor pollicis longus (FPL)
B. Flexor digitorum sublimis (FDS)
C. Flexor digitorum profundus (FDP)
D. Flexor carpi radialis (FCR)
E. Median nerve

PREFERRED RESPONSE ▼ 4
The only neurovascular structure that runs in the carpal tunnel is the median nerve. Flexor carpi radialis is (FCR) is not a tendon
within the carpal tunnel. In summary, the carpal tunnel contains the median nerve, FPL and 4 tendons each of the FDP and
FDS. Of note, with respect to the FDS tendons, the 3rd and 4th FDS tendons are volar to the 2nd and 5th FDS tendons.
48) (OBQ09.194) What is the most frequently encountered form of osseous injury associated with dorsal proximal
interphalangeal joint(PIP) fracture-dislocations? QID: 3007
A. Middle phalanx palmar lip fractures
B. Proximal phalanx extraarticular fractures
C. Middle phalanx dorsal lip fractures
D. Middle phalanx dorsal and palmar lip fractures (pilon)
E. Proximal phalanx palmar lip fractures

PREFERRED RESPONSE ▼ 1
Middle phalanx palmar lip fractures are the most frequently encountered form of osseous injury associated with dorsal PIP joint
fracture-dislocations. Pure PIP joint hyperextension often disrupts the palmar plate either at its distal insertion or by creating a
tension fracture at the palmar lip of the middle phalanx.

The review article by Kiefhaber and Stern detail that the restoration of the middle phalangeal base to glide around the proximal
phalangeal head during the flexion arc is the primary goal. Hinging (instead of articular gliding) at the fracture site must be
avoided by eliminating joint subluxation and then re-establish joint stability to prevent recurrent subluxation. Early motion of the
PIP and anatomic restoration of the fractured joint surface is a desirable but is secondary compared to reduction of the middle
phalanx on the condyles of the proximal phalanx.

Illustration A demonstrates a middle phalanx palmar lip fracture.

Illustrations: A

68
49) (OBQ04.267) What is the name of the pathologic structure, identified by the white arrow in Figure A, that displaces the digital
neurovascular bundle and places it at risk during during surgical treatment of Dupuytren's disease? QID: 1372

A
A. Pretendinous cord
B. Pretendinous band
C. Spiral cord
D. Spiral band
E. Natatory cord

PREFERRED RESPONSE ▼ 3
The spiral cord, shown by the white arrow in Figure A, can displace the neurvascular bundle (blue arrow) and places it at risk
during surgical resection. Dupuytren's contracture is a rare and progressive condition characterized by contractures of the fascia
of the hand as seen in Illustration A. The fascial components involved in the disease include the pretendinous bands, spiral
bands, natatory bands, lateral digital sheets, and Grayson's ligament. The offending cell is the myofibroblast which causes the
normal structures to become fibrosed. Once these normal bands become pathologically involved in the disease process, they
are termed cords. An easy way to remember this is that bands are normal, and cords are abnormal. The spiral cord travels
dorsal to the NVB and displaces it volarly, placing it at risk during surgical resection. Example is shown in Illustrations B. Of note,
Cleland's ligament is not involved in this disease process.
Illustrations: A B
50) (OBQ06.274) A 32-year-old male sustains a 100% tear of his flexor tendon in the Zone 2 region after cutting his finger with a
knife. You plan a one-stage repair of the flexor tendon. Which of the following variables has the greatest effect on increasing the
strength of the tendon repair?QID: 285
A. The size of the core suture
B. Number of core strands crossing the repair site
C. Use of epitendinous suture
D. Active range of motion during the immediate postoperative period
E. Repair of the flexor tendon sheath

PREFERRED RESPONSE ▼ 2
The single most effective intervention for increasing strength of a flexor tendon repair is to increase the number of core sutures
crossing the repair site.

Hatanaka and Manske found that locking loops were better than grasping loops, and that a higher core suture diameter led to an
increase in strength. It is well known that adding an epitendinous suture increases the repair up to 10-50% in strength depending
on the depth of the suture. There is no evidence that fixing the flexor sheath after repair increases the strength of the repair nor
does it lead to improved outcome. Postoperative active range of motion would increase excursion thus decreasing the number of
potential adhesions. Active range of motion of a repaired tendon can facilitate intrinsic over extrinsic tendon healing and
increase tendon tensile strength, but the magnitude of this effect is secondary to the number of sutures crossing the repair site.

-------------------------------------------------(100)-----------------------------------------------------------------------------------------------------

69
1) (OBQ07.34) A butcher sustains a traumatic amputation of the ring finger through the distal interphalangeal joint. He is brought
to the operating room where the flexor digitorum tendon is retrieved and advanced to the distal stump. Three months after
surgery the patient notes that when he makes a fist, only his ring finger tip reaches the palm. What is this patient's clinical
problem? QID: 695
A. mallet finger
B. swan neck deformity
C. boutonniere deformity
D. lumbrical plus deformity
E. quadrigia syndrome

PREFERRED RESPONSE ▼ 5
Quadrigia syndrome occurs when a flexor digitorum profundus (FDP) tendon is shortened and advanced. Malerich et al found
the tolerable degree of FDP advancement was 1 cm. The common muscle belly of the FDP prevents the tendons to the other
fingers from reaching full excursion. Treatment is release of the shortened tendon. Lumbrical plus occurs when the FDP tendon
retracts and causes paradoxical interphalangeal extension when trying to flex. Mallet finger is an injury to the terminal extensor
tendon. Boutonniere deformity occurs from central slip injury and results in PIP flexion and DIP extension. Swan-neck deformity
consists of hyperextension at the PIP joint with flexion at the DIP joint typically caused by volar plate attenuation.

2) (OBQ09.48) A 6-year-old boy sustained a finger tip amputation shown in Figure A after grabbing a broken glass out of the
dishwasher. Your plan was to perform a bedside irrigation and debridement of the finger after digital anesthetic block and apply
antibiotic ointment with a sterile dressing. Upon exploration of the wound you notice that distal phalanx is exposed. Your plan
should change to include which of the following treatments? QID: 2861

A
A. Homodigital island flap
B. Thenar flap
C. Volar flap advancement
D. V-Y advancement flap
E. No change from your initial plan of ointment and dressing

PREFERRED RESPONSE ▼ 5
In young children with a fingertip amputation, ointment and dressing changes is the most appropriate treatment even if bone is
exposed.

When deciding on a treatment, consideration of a "reconstruction ladder" is helpful in determining the least invasive procedure to
obtain the optimal outcome. The ladder includes primary closure, healing by secondary intention, split-thickness skin grafts, full-
thickness skin grafts, random pattern local flaps, axial pattern local flaps, island pattern local flaps, distant random pattern flaps,
distant axial pattern flaps, and free tissue transfer.

Lamon et al reviewed 25 patients, with an average age of 30 years old, with fingertip injuries treated with dressings and warm
soaks started 2 days after injury and noted no healing complications. Only one patient in this cohort had bone exposed.

Soderberg et al performed a Level 3 study of 36 operative and 34 conservatively treated fingertip amputations with bone
exposure and found no benefit to surgery.

70
Farrell et al conducted a Level 4 review of 21 fingertip amputations with 6 having exposed bone and concluded that they healed
with excellent results in regards to contour, sensation, and finger length.

Illustration A shows a homodigital island flap. Illustration B shows a thenar flap. Illustration C shows a volar flap advancement.
Illustration D shows a volar V-Y flap advancement.
Illustrations: A B C D
3) (OBQ10.132) A 72-year-old female complains of progressive weakness with grasp and key pinch in her left hand. Physical
exam of the hand is significant for decreased sensation on the volar aspect of the fourth and fifth digits. Dorsal sensation
throughout the hand is normal. A clinical photo displaying bilateral key pinch is shown in Figure A. What is the most likely cause
of compression? QID: 3225

A
A. Accessory head of the FPL
B. Flexor carpi ulnaris
C. Osborne's ligament
D. Ganglion within Guyon's canal
E. Anconeus epitrochlearis

PREFERRED RESPONSE ▼ 4
Compression of the ulnar nerve within Guyon's canal, termed ulnar tunnel syndrome, is most commonly caused by a ganglion
cyst. A lack of dorsal ulnar sensory deficit helps differentiate entrapment here from at the elbow because the dorsal ulnar
cutaneous nerve branches proximal to Guyon's canal. The clinical photo demonstrates Froment's sign where the FPL is used to
substitute for the weakened adductor pollicis resulting in flexion of the thumb at the interphalangeal joint, and MCP joint
hyperextension. The AIN can be compressed by the accessory head of the FPL (Gantzer's muscle) which results in loss of FPL,
index FDP and PQ motor function and no sensory deficits. Ulnar nerve compression at Osborne's ligament, the two heads of the
FCU, or by the anconeus epitrochlearis will classically result in volar and dorsal ulnar sensory loss of the affected hand.
4) (OBQ04.10) A 23-year-old presents with a knife laceration in the flexor zone 2 of the hand. Examination of the wound is
performed and a laceration of the flexor tendon one-half the width of the tendon is identified. There is no triggering present as
the patient's finger is passively extended and flexed fully. The most appropriate treatment is: QID: 121
A. No tendon repair with early protected range of motion
B. No tendon repair with splint immobilization for 2 weeks
C. Tendon repair with 2 strand repair and early protected range of motion
D. Tendon repair with 2 strand repair with splint immobilization for 2 weeks
E. Tendon repair with 4 strand repair and early active range of motion

PREFERRED RESPONSE ▼ 1
Nonsurgical treatment with early protected range of motion is indicated for flexor tendon lacerations one-half the width of the
tendon. The article by Al-Qattan recommends that partial flexor tendon lacerations should be fixed if the laceration is greater
than 60%. Furthermore, the patient should be witnessed under digital block that they have full extension and flexion without
triggering which would be another indication to operate. Rehabilitation consists of early ROM, wrist and MP flexed in dorsal
splint, PIP and DIP extended, Passive digital flexion with wrist flexed, and wait until eight weeks postop to begin strengthening.
This concept was also tested in question 9 of the 2007 OITE with the cited reference by McGeorge and Stillwell comparing the
results of repair with non-repair in humans for zone 2 injuries and concluded that tendons lacerated by 60% or less should not
be repaired.

71
5) (OBQ04.248) Which of the following structures are slowly adapting skin receptors that detect pressure, texture, and low
frequency vibration and are best evaluated by static two-point discrimination? QID: 1353
A. Meissner's corpuscles
B. Pacinian corpuscles
C. Merkel's receptor
D. Free nerve endings
E. Ruffini corpuscles

PREFERRED RESPONSE ▼ 3
Merkel's skin receptors are slowly adapting skin receptors that detect pressure, texture, and low frequency vibration and can be
appropriately evaluated by static two-point discrimination. Merkel's disk receptors adapt slowly and sense sustained pressure,
texture, and low-frequency vibrations.

Szabo et al state in their review that static and moving two point discrimination are best to initially evaluate innervation density
for both quickly and slowly adapting fibers. Vibratory moving 2 point discrimination is best for evaluation of quickly adapting
fibers.

Meissner corpuscle, a rapidly adapting sensory receptor, is very sensitive to touch. Pacinian corpuscles are ovoid in shape,
measuring approximately 1 mm in length. They respond to high-frequency vibration and rapid indentations of the skin. Ruffini
corpuscles are slowly adapting receptors that detect stretching of the skin.

Illustration A demonstrates Meissner's corpuscles (A), Pacinian corpuscles (B), Merkel's receptor (C), free nerve ending (D), and
Ruffini corpuscles (E). Illustration B displays the function and location of the receptors.

Illustrations: A B
6) (OBQ06.265) Which of the following amputations may be considered a relative contraindication for a replantation? QID: 276

A. Ring finger through the proximal phalanx shaft


B. Mid forearm
C. Thumb through the proximal phalanx
D. Middle, ring, and small fingers through the middle phalanx shaft
E. Complete hand just proximal to the distal palmar crease

PREFERRED RESPONSE ▼ 1
Replantation of a single finger amputated proximal to the insertion of the flexor superficialis tendon is a relative contraindication
because of the severe stiffness and poor function encountered after repairs in this location.

The FDS insertion is in the middle of the middle phalanx and is also what defines the distal border of Zone II in flexor tendon
injuries. If the finger is cut proximal to the insertion of FDS, that means that FDS is also cut and needs to be repaired leading to
severe stiffness and worse PIP function. The exceptions are when there are multiple digits injured or in young children.

Urbaniak et al found replantation of a single finger amputated distal to the insertion of the flexor superficialis tendon is justified,
but that replantation of a single finger that was amputated proximal to this insertion is not indicated.

Tamai et al found that an amputation of the hand, forearm, arm, and thumb, as well as multiple digits are all criteria for
replantation.
7) (OBQ11.56) A 2-year-old child has a flexion deformity of the interphalangeal joint of his thumb as seen in Figure A. Surgical
correction of this deformity places what structure most at risk as it crosses the surgical field? : 3479

A
72
A. Princeps pollicis artery
B. Ulnar digital nerve
C. Oblique pulley
D. Ulnar digital artery
E. Radial digital nerve

PREFERRED RESPONSE ▼ 5
The patient in the scenario has a trigger thumb. Surgical correction of this condition requires the release of the A1 pulley. The A1
pulley is seen at the red arrow in Illustration A. During the dissection, the radial digital nerve crosses the operative field and is at
risk. It must be identified and protected.

Bae described the etiology, natural history and surgical indications in treatment of pediatric trigger thumbs. Currently, the
literature does not advocate for or against surgical management. He concluded that further long-term research will help guide
evidence-based treatment.

Baek at al described the natural history of pediatric trigger thumbs. They concluded that trigger thumbs in children will resolve
without treatment in >60% of patients. In patients who do not have full resolution, the flexion deformity can be expected to
gradually improve with time.

Illustrations: A
8) (OBQ08.23) A professional baseball player develops acute hand pain after fouling off a pitch. He is tender over the
hypothenar eminence and has paresthesias in the ring and small fingers. Which radiographic view is most likely to reveal the
pathology? QID: 409
A. PA wrist
B. AP wrist in ulnar deviation
C. Lateral wrist
D. Carpal tunnel
E. Scaphoid

PREFERRED RESPONSE ▼ 4
Plain radiographs usually do not reveal the fracture; carpal tunnel and supinated oblique views should be obtained. Diagnosis is
confirmed by CT scan and bone scan.

Fractures of the body of the hamate may occur from trauma and may occur in combination with fractures of the base of the
fourth and fifth metacarpals. Fractures of the hook of the hamate are more common in athletes. The incidence of hook of the
hamate fractures is 2% to 4% of all carpal fractures. The mechanism of injury is thought to be caused by abutment of the hook
on an object or by a shearing force applied by the flexor tendon of the small and ring fingers. The injury usually occurs in
athletes who participate in baseball, golf, and racquet sports because of the position of the implement in the hand.

Rettig reviewed hand injuries in athletes. He noted that hamate hook fractures occur in a watershed area that may explain the
high incidence of nonunion post fractures. Hook of the hamate fracture must be suspected in athletes participating in racquet
sports, golf, or baseball who are seen with ulnar wrist pain. Examination reveals tenderness over the hook of the hamate, which
lies on a line between the pisiform and second metacarpal head. Treatment of hook of the hamate fractures in athletes varies
from casting to open reduction and internal fixation to excision.

Bishop and Beckenbaugh reported 21 cases of this fracture: 17 were treated by excision, 3 underwent ORIF, and 1 had casting.
Although two of three fractures that were treated with ORIF healed, many authors recommend excision, which has an average
return to sport of 6 to 10 weeks.

Illustration A shows a representative carpal tunnel view, with the bony anatomy labeled.

Illustrations: A

73
9) (OBQ05.164) A 64-year-old diabetic female presents with sudden catching and locking of her ring finger when trying to extend
it. Attempts at finger extension are painful, and she notes tenderness in her distal palm. A clinical photo is shown in Figure A.
Which of the following structures are implicated in the pathogenesis of this condition? QID: 1050

A
A. Extensor digitorum tendon
B. Grayson's ligament
C. Oblique retinacular ligament
D. A1 pulley
E. Transverse carpal ligament

PREFERRED RESPONSE ▼ 4
The scenario listed above points to a diagnosis of trigger finger. In this condition, the disparity in size between the flexor tendon
and the surrounding retinacular pulley system, most commonly at the level of the first annular (A1) pulley, results in difficulty
flexing or extending the finger and the “triggering” phenomenon. Metacarpophalangeal locking should be included in the
differential, where the collateral ligament or volar plate tethers on a prominent metacarpal head or osteophyte. The referenced
text notes that a series of two corticosteroid injections should be given before surgery is considered for A1 pulley release.
Mention is also made of the possibility of diabetics being more resistant to injections. Illustration A shows the pathogenesis of
this disorder.

10) (OBQ09.56) An open dorsal approach for antegrade screw fixation of a nondisplaced scaphoid waist fracture differs in which
of the following ways compared to a percutaneous dorsal approach? QID: 2869
A. Decreased risk of proximal pole AVN
B. Increased risk of posterior interosseous nerve injury
C. Decreased risk of injury to the APL tendon
D. Increased risk of injury to the EPL tendon
E. Decreased risk of screw prominence above subchondral bone

PREFERRED RESPONSE ▼ 5
Scaphoid screw fixation should be just below the subchondral bone; this is best judged by direct visualization.

74
Adamany et al in an anatomic study using fluoroscopy to insert a scaphoid screw via a percutaneous approach found that the
scaphoid screw "was prominent (above the subchondral bone) in 2 of 12 specimens and flush with or buried in the remaining 10
specimens." As a result, they recommend using a limited dorsal incision to verify full seating of the screw. In addition, they found
the percutaneous approach was within 2.2-3.1 mm of the PIN, EDC, and EIP. Thus, all of these structures are at increased risk
of injury in a percutaneous approach. The APL tendon is not in the surgical field. Illustration A shows the AIN(arrowhead) is
deep in relation to pronator quadratus. Sensory remnant of posterior interosseous nerve (straight thick arrow) is adjacent to
interosseous membrane. White arrow is median nerve. Shaded open arrow is ulnar nerve, and long thin arrow is superficial
radial nerve.

Tumilty et al inserted a Herbert screw through a dorsal approach in 12 cadaveric wrists. They then imaged them with AP/Lateral
xrays, and 360 degree fluoroscopic views. The wrists were then dissected to evaluated for subchondral penetration, and plain x-
ray films were accurate in 5 of 6 specimens. Fluoroscopy was accurate in all 6. They concluded that fluoroscopy during
placement of the Herbert screw may decrease the rate of subchondral penetration.

11) (OBQ13.58) A healthy 50-year-old secretary is about to undergo an open carpal tunnel release. Which of the following peri-
operative steps will have the greatest influence on minimizing the risk of a surgical site infection in this patient? QID: 4693

A. Administration of cefazolin within 1 hour before incision


B. Administration of cefazolin within 1 hour before incision followed by 5 days of cephalexin post-op
C. Cleanse with bacitracin solution immediately before skin incision
D. Standard sterilization and prepping
E. Administration of one dose of cephalexin within 1 hour before incision

PREFERRED RESPONSE ▼ 4
The patient is undergoing a clean, elective hand surgery. Prophylactic antibiotics, systemic or local, are not indicated for these
procedures.

Carpal tunnel syndrome is the most common compressive neuropathy. Individuals who fail medical management (night splints,
NSAIDs, activity modification) are candidates for carpal tunnel release surgery (CTS). The surgery may be performed open or
endoscopically. The reported incidence of post-operative infections following CTS varies between studies from 0% to 8%.

Whittaker et al. performed a prospective, randomized, double-blinded, placebo controlled trial investigating the use of antibiotic
prophylaxis in clean, incised hand injuries. They found no significant difference in infection rates between patients who received
IV flucloxacillin, IV followed by oral flucloxacillin, and an oral placebo (13% vs. 4% vs. 15%, p=0.19). They did not support the
use of routine antibiotic prophylaxis prior to clean hand surgery.

Bykowski et al. retrospectively reviewed 8,850 outpatient elective hand surgeries and found no significant difference in the rate
of surgical site infection, including patients with diabetes or history of smoking. They concluded that antibiotics should not be
routinely administered prior to clean, elective hand surgeries.

Harness et al. found no statistical difference in the incidence of surgical site infection following CTS without prophylactic
antibiotic compared with patients who received prophylactic antibiotics (0.7% vs. 0.4%, p=0.354). They did not recommend

75
routine antibiotic prophylaxis.

Illustration A reviews the anatomic components of the carpal tunnel.

Incorrect Answers:
Answers 1, 2, 3, 5: Antibiotics do not decrease the risk of post-operative infection in clean, elective hand surgery. Surgeons
should consider the potential risks of antibiotics prior to administration, including Clostridium difficile colitis, antibiotic allergies,
bacterial resistance, and so on.

12) (OBQ07.244) A 37-year-old man has a 2-year history of increasing right wrist pain that is worse at night and aggravated by
activity. He denies systemic symptoms, history of trauma, or recent weight loss. On physical exam he has tenderness over the
dorsal radiocarpal joint. Radiographs of the right wrist are shown in Figure A. Which of the following imaging studies would be
most sensitive for determining the stage of this patient's underlying condition? QID: 905

A
A. Ultrasound
B. Angiography
C. CT scan of the wrist
D. Clenched fist AP radiograph of wrist
E. Bone scan of the wrist

PREFERRED RESPONSE ▼ 3
The clinical presentation of dorsal radiocarpal wrist pain is suggestive of Kienbock’s disease. Figure A shows an AP radiograph
of the right wrist with evidence of lunate sclerosis with no obvious collapse. The imaging study most sensitive for identifying early
lunate collapse in Kienbock's disease is CT scanning of the wrist.

Kienbock’s disease is defined by avascular necrosis of the lunate. It is classified into 4 stages under the Lichtman Classification.
In stage 1, plain radiographs appear normal and magnetic resonance imaging is required for diagnosis. MRI is useful for
detecting early disease when sclerosis is not evident on plain film radiographs. In stage 2, plain radiographs and/or CT scan
images will show sclerosis of the lunate but no evidence of collapse. In stage 3, radiographs and/or CT scan images will show
lunate collapse. For stage 4, radiographs show degenerative changes to the adjacent carpus and intercarpal joints.

Imaeda et al. examined the use of MRI for the diagnosis and staging of Kienbock's disease. They found that MRI was most
sensitive in detecting early focal loss of signal intensity in the lunate on T1-weighted images. This was a key diagnostic feature
in early stages of Kienböck's disease when plain radiographs appear normal.

Cross et al. reviewed the latest concepts for diagnosis, staging, and management of Keinbock's disease. They suggest that
computed tomography (CT) or tomography will better characterize lunate necrosis and trabecular destruction once collapse or
sclerosis has occurred in late stage disease.
76
Illustration A is a collection of CT scanning images that show osteonecrosis of the lunate. The blue arrow shows lunate flattening
and sclerosis. The red double arrow shows a loss of lunate height and the yellow shows fragmentation of the bone.

Incorrect Answers:
Answer 1: Ultrasound is not used in the staging of Kienbock's disease.
Answer 3: Angiography would not be warranted in this scenario.
Answer 4: A clenched fist AP radiograph of the wrist is used to evaluate widening of the scapholunate interval.
Answer 5: A bone scan of the wrist is a non-specific test, which would likely be positive in almost all patients with chronic wrist
pain.
Illustrations: A
13) (OBQ09.65) You are taking care of an adult patient with significant scar contracture in her first web space after a thermal
burn. Which of the following techniques will allow you to lengthen her scar approximately 75%?QID: 2878
A. Cross-finger flap
B. Two-flap Z-plasty with 60 degree limbs
C. Two flap Z-plasty with 25 degree limbs
D. Island pedical flap
E. Split-thickness skin graft

PREFERRED RESPONSE ▼ 2
One of the most commonly used techniques for lengthening scar contracture in hand surgery is the Z-plasty. When the two 60
degree triangular flaps are transposed and closed, the original direction of the scar is rotated and the scar length is increased by
approximately 75% Because of its history the 60 degree Z-plasty is the technique to which other methods of contracture
lengthening are compared.

Hove et al describe the technique, various applications, and different types of Z-plasty used today. Neither the cross-finger flap
nor island pedical flap are useful for this amount of scar release. Two flap Z-plasty with 25 degree limbs does not offer enough
lengthening. Split-thicknes skin grafts are not useful for either lengthening or the volar aspect of the hand due to the significant
contracture they experience.

Illustration A and B depict the Z-plasty technique.

Illustrations: A B
14) (OBQ12.98) A 22-year-old man presents with pain and hyperesthesias in the dorsoradial wrist and thumb 6 months after
undergoing wrist arthroscopy for an acute wrist injury. Placement of which of the following portals is likely responsible for his
symptoms? QID: 4458
A. 6U
B. 6R
C. 1-2
D. MCU
E. 4-5

PREFERRED RESPONSE ▼ 3
The 1-2 wrist arthroscopy portal is the only portal listed that places the superficial branch of the radial nerve (SBRN) at risk.

Wrist arthroscopy plays an important diagnostic and therapeutic role in injuries about the wrist, but carries a 2% complication
rate. The 1-2 portal is placed between the ECRB and APL. Care must be taken when accessing this portal, due to reported
injuries to the superficial branch of the radial nerve, as well as radial artery.

Kilic et al. dissected 6 cadavers to determine the course of the SBRN. They found that it was .9mm from the 1-2 portal at its
closest, and that care should be taken when creating this portal given frequent variations in course.

Auerbach et al. dissected 20 cadavers to determine the course of the SBRN. In all specimens, the nerve arose between the
brachioradialis and ECRL ~8cm proximal to the radial styloid. On average, 5.8 branches of the SBRN crossed the wrist joint.
They found some variability within the course, and urge surgeons to be respectful of the nerve during dissection.

77
Tryfonidis et al performed a cadaveric study to determine the anatomic relation of dorsal wrist arthroscopy portals and superficial
nerves. They found that the 1-2 portal was only 1.82 mm from the SBRN. Compared to the 3-4 portal, 4-5 portal, 6U portal and
midcarpal portal, the 1-2 portal was the closest in proximity to any sensory nerve.

Illustration A shows the various portals used in wrist arthroscopy. Illustration B shows the relation of the SBRN to the various
arthroscopic portals.

Incorrect Answers:
Answer 1: The 6U portal places the dorsal sensory branch of the ulnar nerve at risk.
Answer 2: The 6R portal places the dorsal sensory branch of the ulnar nerve at risk.
Answer 4: The MCU portal places the EDC and EDM tendons at risk.
Answer 5: The 4-5 portal places the EDC and EDM tendons at risk.
Illustrations: A B
15) (OBQ11.99) Which of the following dorsal wrist compartments is incorrectly paired with its contents? QID: 3522
A. Compartment 6: Extensor carpi ulnaris
B. Compartment 5: Extensor digiti minimi
C. Compartment 4: Extensor digiti communis, posterior interosseous nerve
D. Compartment 3: Extensor pollicus longus, extensor carpi radialis longus
E. Compartment 1: Abductor pollicus longus, extensor pollicus brevis

PREFERRED RESPONSE ▼ 4
The dorsum of the wrist is subdivided into six compartments. The PIN is the only nerve found in the dorsal compartments. It is
consistently found on the base of the fourth compartment. Anatomic structures within each compartment are:

Compartment 1: Abductor pollicus longus, extensor pollicus brevis.


Compartment 2: Extensor carpi radialis longus and brevis
Compartment 3: Extensor pollicus longus
Compartment 4: Extensor digiti communis, extensor indicis propius, posterior interosseous nerve
Compartment 5: Extensor digiti minimi
Compartment 6: Extensor carpi ulnaris

Iwamoto et al performed a cadaveric study of the six compartments and septa. They determined that the 1/2 compartment had
the thickest radial surface while the 3/4 septa was the thinnest. The fifth dorsal compartment had the lowest resistance to failure.

Palmer et al also performed a cadaveric study on the extensor retinaculum. They concluded that the retinaculum provides a
check rein to bowstringing and should be repaired during surgery.

Grutter et al injected the PIN and AIN in fresh-frozen cadavers. In the first group, the AIN and PIN were injected via a single
injection 1cm ulnar and 3cm proximal to Lister's tubercle. For group 2 (PIN alone), the injection site was 1cm ulnar to the
proximal aspect of Lister's tubercle. Their technique led to accurate delivery of anesthetic to the PIN and AIN in 100% of the
cadavers.
Illustration A shows the 6 dorsal wrist compartments. Illustration B shows the injection site in the Grutter article.
Illustrations: A B
16) (OBQ08.215) An 8-year-old boy's parents are concerned about the appearance of the child's middle finger. The child denies
pain and his digital neurovascular status is normal. A clinical photograph and radiograph are provided in figures A and B. For
children with this condition, which of the following is the best intervention to achieve a finger that is proportional to the rest of the
hand? QID: 601

A B
78
A. Epiphysiodesis now
B. Epiphysiodesis when the finger reaches adult length of the father
C. Compression wrapping until proportional size is achieved
D. Resection of hypertrophic nerves
E. Osteotomy at skeletal maturity.

PREFERRED RESPONSE ▼ 2
Clinical photograph and radiographs demonstrate macrodactyly of the middle finger, a rare congenital malformation enlarging all
structures of the digit.

Ishida et al reviews 23 cases of surgically treated macrodactyly finding favorable results with epiphysiodesis/epiphysiodectomy
while resection of hypertrophic nerves was unsuccessful in preventing overgrowth. The epiphysiodesis is performed once the
finger reaches the length of the same sex parent, using their digit as a template for final growth.
17) (OBQ11.128) The physical exam finding demonstrated on the patient's right hand in the video (Figure V) is found with
neuropathy of which of the following nerves? FIGURES: V QID: 3551

A. Musculocutaneous nerve
B. Anterior Interosseious Nerve (AIN
C. Radial nerve
D. Ulnar nerve
E. Median nerve

PREFERRED RESPONSE ▼ 4
The video demonstrates Froment's sign on the patient's right hand, which is characterized by interphalangeal (IP) flexion during
attempted key pinch, and is found in patients with ulnar neuropathy. Therefore it can be found with ulnar nerve compression in
the cubital tunnel (Cubital Tunnel Syndrome) or in Guyon's Canal (Ulnar Tunnel Syndrome).

Froment's sign is performed by having the patient pinch a piece of paper with the thumb IP joint extended against resistance
(pulling paper away). It should be done with both hands side by side to compare them to each other.

In a hand with a ulnar neuropathy, adductor pollicis (ulnar n.) is deficient, and can not flex the MCP joint to give pinch strength
with an extended IP joint. The thumb compensates by recruiting the FPL (median n.) to flex the IP joint to give pinch strength.
The result is, in a positive Froment's sign, the IP joint will flex (buckle) to try to give increased strength to the pinch.

Illustration V shows a demonstration of the Froment's sign.


Illustrations: V
18) (OBQ08.91) Which of the following muscles provide the primary deforming forces to Bennett and Rolando fractures (base of
the 1st metacarpal fractures)? QID: 477
A. Pronator quadratus
B. Flexor pollicis longus
C. Extensor pollicis longus
D. Adductor pollicis longus and abductor pollicis
E. Abductor pollicis longus and adductor pollicis

PREFERRED RESPONSE ▼ 5
The primary deforming forces in Bennett and Rolando fractures are the Abductor pollicis longus and adductor pollicis.

In a Bennet's or Rolando fracture-dislocation the volar-ulnar fracture fragment is held reduced by the anterior oblique ligament
while strong deforming forces pull the remaining metacarpal shaft proximally and dorsally, angulate the shaft ulnarly and
supinate the shaft. Most important in these deforming forces are the abductor pollicis longus (APL) inserting on the base of the
metacarpal which pulls the metacarpal shaft proximally and dorsally and the adductor pollicis (AP) which inserts on the ulnar
base of the proximal phalanx and angulates the metacarpal shaft ulnarly and supinates the shaft. Less important is the extensor
pollicis longus (EPL) which inserts on the base of the distal phalanx and also adds to the ulnar angulation of the distal fragment.

Soyer reviews the diagnosis, pathoanatomy, and treatment for fractures at the base of the 1st metacarpal. Understanding the

79
biomechanics, anatomical deforming forces, and the exact fracture pattern aids the treating surgeon in determining the most
appropriate method of fixation. The most essential factor for obtaining a good functional result is anatomic restoration of the
articular surface.

Elgafy et al. examined the terminal anatomy of the posterior interosseous nerve in their cadaver study - identifing six terminal
branches and describing methods to avoid injury. They describe how treating surgeons can maximize function and recovery
after base of the 1st metacarpal fractures by understanding these nervous branches and specific fracture pattern treatment to
avoid iatrogenic injury to the PIN.
Illustrations: A
19) (OBQ06.16) A 27-year-old professional cowboy is thrown from a bull during the rodeo and lands on his hand. No deformity is
identified and the hand is completely neurovascularly intact. Pain is present upon palpation of the anatomic snuffbox. A
radiograph is provided in Figure A. The cowboy wants to return to competitive riding tomorrow. Which of the following is the best
next step in management? QID: 27

A
A. Cock-up wrist splint and immediate return to sport as tolerated by pain
B. Steroid injection of the snuffbox, taping of the wrist and return to sport
C. Wrist MRI
D. Percutaneous screw fixation of the nondisplaced fracture
E. Scapholunate ligament repair and percutaneous pin fixation

PREFERRED RESPONSE ▼ 3
Tenderness with palpation of the anatomic snuffbox should raise suspicion of a scaphoid fracture. The radiograph does not
show any findings, but scaphoid fractures are often not initially visualized on plain radiographs. Appropriate treatment for any
patient with snuffbox tenderness entails cast immobilization with repeat radiographs at 2-3 weeks or advanced imaging with MRI
to evaluate for a fracture that is not identified with plain radiographs.

The MRI that correlates with this patient's radiograph is provided in Illustration A and demonstrates a nondisplaced scaphoid
fracture.

Gaebler et al performed an MRI study of 32 consecutive patients who were clinically suspicious for a scaphoid fracture, but no
fracture could be indentified on wrist radiographs. The MRI was 100% sensitive and specific in diagnosing scaphoid fracture.

Treatment for this patient following the MRI would be debatable. Cast immobilization would be appropriate, but screw fixation
may allow earlier return to sport. A percutaneous compression screw would be an appropriate technique for this scaphoid
fracture.

Illustrations: A

80
20) (OBQ05.91) A 29-year-old intravenous drug user undergoes irrigation and debridement of a ring finger abscess. After
adequate eradication of the infection, he is left with the skin defect shown in Figure A. What is the most appropriate treatment at
this time? QID: 977

A
A. Local woundcare and healing by secondary intention
B. V-Y advancement flap
C. Thenar flap
D. Moberg flap
E. Cross-finger flap

PREFERRED RESPONSE ▼ 5
Based on the location of the lesion, a cross-finger flap would be most appropriate.

Cross finger flaps are indicated in patients > 30 years of age when the lesion is a volar oblique finger tip lacerations or a volar
proximal finger lesions. The advantage is it leads to less stiffness.

Martin et al review the treatment options available for digit injuries. They report treatment of fingertip injuries is a continuous
focus of controversy among hand and orthopaedic surgeons. Different treatment options have been described, depending on the
affected segment and finger, type of lesion, gender and age of the patient, location, size, and depth of the defect.

Fassler et al reviews the proper management of fingertip injuries discussing variables such as the severity of soft tissue loss and
whether bone is exposed.

Incorrect Answers:
Answer 1: Secondary intention healing of this wound is inappropriate due to size and exposed tendon.
Answer 2: V-Y advancement flaps are for dorsal injuries.
Answer 3: Thenar flaps are good for getting more bulk for distal fingertip injuries.
Answer 4: A Moberg flap is performed on the thumb. A cross-finger flap is a full-thickness flap useful for volar soft tissue loss
distal to PIP.
21) (OBQ05.103) A 4-year-old boy sustains a flexor tendon laceration in Zone 2 of his 4th digit when he attempts to grab a knife.
Optimal surgical management and postoperative rehabilitation consists ofQID: 989
A. 2 strand core suture technique and gentle active flexion and extension exercises with wrist in extension
B. 2 strand core suture technique and cast immobilization for 8 weeks
C. 4 strand core suture technique and gentle active flexion and extension exercises with wrist in extension
D. 4 strand core suture technique and cast immobilization for 4 weeks
E. 4 strand core suture technique and cast immobilization for 8 weeks

PREFERRED RESPONSE ▼ 4
4 strand core suture technique and cast immobilization for 4 weeks is the preferred postoperative rehabiltation in a 4 year old
child.

Ordinarily, adult flexor tendon repair postoperative rehab protocols call for early light active digital flexion with wrist in gentle
flexion as long as the tendon has been repaired with a 4 or 6 strand core suture technique and strong epitendinous suture.
However, this method cannot succeed without the cooperation of a mature and motivated patient. Children or the mentally
disabled are often lacking some of these prerequisites. Therefore, a flexor tendon repair in a child should be treated like a flexor

81
tendon repair with interposed graft in an adult. Immobilization for a minimum of 3 – 4 weeks with a posterior molded plaster
splint or cast from the tips of the fingers to just above the elbow. Wrist is flexed 35 degrees, MCPs flexed 60 – 70 degrees and
IP joints relaxed in extension. Active motion can be started after the cast is removed at 4 weeks.
22) (OBQ05.46) A 42-year-old construction worker presents with pain in his right wrist. A current radiograph of the wrist is shown
in Figure A. He reports that rotating activities, such as turning a screw driver, are bothersome and the pain is preventing him
from working. A current MRI reveals a TFCC tear, and nonsurgical treatment has failed to provide relief. Treatment should now
consist of: QID: 82

A
A. Repair of the ulnar styloid nonunion
B. Darrach resection of the distal ulna
C. Complete ulnar head resection
D. Ulnar hemiresection arthroplasty and TFCC reconstruction/repair
E. Isolated arthroscopic TFCC reconstruction

PREFERRED RESPONSE ▼ 4
The clinical presentation is consistent with DRUJ arthritis in a heavy laborer. Of the options listed, ulnar hemiresection
arthroplasty with concurrent TFCC reconstruction or repair would be the most appropriate treatment.

While there are multiple treatment options, the ulnar hemiresection arthroplasty with concurrent TFCC reconstruction or repair is
considered most appropriate in heavy laborers, as it would likely resolve the pain and enable them to return to work sooner. The
TFCC should be intact when performing an ulnar hemiresection arthroplasty to prevent distal ulna instability with forearm
rotation. One could also consider performing a Suave-Kapandji procedure. This procedure creates a distal radioulnar fusion and
an ulnar pseudarthrosis proximal to the fusion site through which rotation can occur. The advantage is that the ulnocarpal joint is
not sacrificed, and a stable wrist is created.

Scheker et al reported on the outcome of ulnar shortening performed on 32 wrists with early osteoarthritis of the DRUJ. The
postoperative wrist ratings were 7/32 excellent, 11/32 good, 9/32 fair, 5/32 poor, with plate irritation being the most frequent
postoperative complication.

Figure A is a radiograph showing significant DRUJ arthritis. Illustration A shows ulnar hemiresection arthroplasty. Illustration B
shows a Darrach procedure. Illustration C shows a Sauve-Kapandji procedure. Illustration D is a treatment schematic of TFCC
reconstruction.
Incorrect Answers:
Answer 1: There is no obvious ulnar styloid non-union.
Answer 2: As mentioned in Miller's review text, the Darrach procedure is typically reserved for low-demand, elderly patients and
may lead to painful proximal ulna stump instability.
Answer 3: Complete ulnar head resection is not indicated.
Answer 5: TFCC reconstruction will not improve or treat the DRUJ arthritic changes.

82
23) (OBQ12.253) A 45-year-old patient presents with recurrence of radial sided wrist pain after undergoing a first dorsal
compartment release about 3 months ago. The surgery was completed by one of your partners; operative reports indicate that
the sheath was incised on the dorsal edge. On physical exam she is found to have normal appearing skin, a negative Tinel’s
sign, and a positive Finklestein test. What is the most likely cause of the recurrence of her symptoms? QID: 4613

A. Development of neuroma
B. Complex regional pain syndrome
C. Failure to decompress the EPB sub-sheath
D. Failure to decompress the EPL sub-sheath
E. Failure to decompress the APB sub-sheath

PREFERRED RESPONSE ▼ 3
Based on the history and clinical findings this patient has de Quervain’s tenosynovitis. The recurrence of her symptoms can be
attributed to a failure to recognize and decompress the EPB sub-sheath.

De Quervain’s tenosynovitis is a stenosing inflammatory condition of the first dorsal compartment of the wrist (APL/EPB).
Surgical release of the compartment is indicated after conservative measures have failed. At the time of the operation, the
incision is made on the dorsal side of the sheath to prevent volar subluxation of the tendons. Failure to identify and release a
distinct EPB sub-sheath or a separate fibro-osseous compartment of the APL can lead to a recurrence of symptoms.

Alegado et al. report a case of a patient with dysesthsias in the superficial radial nerve distribution 3 months after undergoing
first dorsal compartment release for de Quervain’s tenosynovitis. They found a persistent fibrous remnant of the dorsal aspect of
the sheath causing elevation of the superficial radial nerve. They recommend sheath excision or incision of the sheath at its
dorsal attachment to avoid this complication.

Ashurst et al. report a case of a patient presenting with bilateral de Quervain’s tenosynovitis secondary to excessive text
messaging. Conservative measures afforded the patient complete symptomatic recovery. They recommend limitation of texting,
in conjunction with other standard treatments, to treat text messaging- associated de Quervain’s tenosynovitis

Ilyas et al. review the etiology, diagnosis and management of De Quervain’s tenosynovitis. Non-surgical management is largely
successful and includes splinting and cortisone injections. In refractory cases, surgical release of the first dorsal compartment is
completed. They recommend meticulous care of the radial sensory nerve and identification of all separate sub-sheaths.

Illustration A shows an operative photo in a patient with multiple APL slips and an EPB that is hidden within a sub-sheath. Video
V gives a brief overview of de Quervain’s tenosynovitis.

Incorrect Answers
Answer 1: Given the negative Tinel’s sign on physical exam, the patient is less likely to have a neuroma.
Answer 2: Her history, symptoms and lack of skin changes are not consistent with complex regional pain syndrome.
Answers 4, 5: The first dorsal compartment is composed of the APL/EPB. The EPL is in the third dorsal compartment and the
APB is in the thenar compartment.
Illustrations: A V

83
24) (OBQ12.89) What is the optimal treatment for the proximal phalanx fracture shown in Figure A? QID: 4449

A
A. Open reduction and placement of two 0.045-inch K-wires placed longitudinally through the metacarpal head
B. Application of a 1.5-mm straight plate applied dorsally through and extensor tendon splitting approach
C. Open reduction and lag screw fixation with 1.3mm screws through a radial approach
D. Placement of a 1.5-mm condylar blade plate through a radial approach
E. Open reduction and retrograde passage of two 0.045-inch K-wires retrograde trough the PIP joint

PREFERRED RESPONSE ▼ 3
Open reduction and lag screw fixation through a radial approach is the treatment of choice for long oblique proximal phalanx
fractures.

Lag screws (1.3 mm is preferred to 1.5 mm) can achieve stability through interfragmentary compression in a two-part spiral or
long oblique fracture of the proximal phalanx that rivals the stability of an intact bone. This capability, combined with the need to
achieve precise correction of rotation in spiral fractures, makes open reduction and lag screw fixation the treatment of choice for
the spiral P1 shaft fracture.

Kawamura et al. discuss the treatment options for closed phalangeal and metacarpal fractures. They state that percutanous
fixation with K-wires can be successful in the setting of an adequate closed reduction. However, lag screw fixation may be the
best choice for open fixation of long oblique phalangeal and metacarpal fractures.

Henry et al. outline the methods of stablization for phalangeal and metacarpal fractures. With regards to long oblique proximal
phalanx fractures, they state that open reduction and lag screw fixation the treatment of choice for the spiral P1 shaft fracture.
This is due to the lack of inherent stability of the long fracture segment, and the tendency for the fracture to shorten even in the
presence of smooth k-wire fixation.

Freeland et al. provide a review which cites new developments in the treatment of extra-articular hand fractures in adults.

Figure A shows a long obligue fracture of the index finger proximal phalanx. Illustration A demonstrates fixation of this injury with
interfragmentary lag screws.

Incorrect Answers:
Answers 1,5: Most transverse P1 fractures without comminution are best stabilized by two 0.045-inch
K-wires placed longitudinally through the metacarpal head or PIP joint, depending on the location of the fracture.
Answers 2,4: Application of plates is usually reserved for comminuted bicondylar phalanx fractures, and is performed through a
radial approach if possible to avoid postoperative adhesions.

Illustrations: A

84
25) (OBQ09.54) The Bilhaut-Cloquet procedure for thumb duplication, where the central portions of bone and nail are removed
and the radial half of one thumb is combined with the ulnar half of the other to create one thumb, is most appropriate in which
Wassel Type shown in Figure A? QID: 2867

A
A. Type 2
B. Type 4
C. Type 5
D. Type 6
E. Type 7

PREFERRED RESPONSE ▼ 1
The Bilhaut-Cloquet procedure has been shown to be successful in Wassel Type 1, 2, and 3 deformities. Asymmetrical Type I or
II duplications with distinct components may also be treated with ablation of the smaller thumb with transfer of the collateral
ligament and centralization of the extensor tendon. Types 3 and 4 duplications (Type 4= most common duplication) are often
treated with selection of a dominant thumb and ablation of the lesser counterpart after preservation and transfer of intrinics and
collateral ligaments.

The article by Ogino states that the ablation of the radial thumb is most common and Miller's Review states radial thumb ablation
is preferred. Treatment of Types 5 and 6 duplication utilizes similar principles as Type 4 deformities, with the added complexity
of additional intrinsic reconstruction. Type 7 is a triphalangeal thumb and has variable treatment based on the presentation, and
reconstruction includes excision versus fusion of the extra phalanx.

Baek el al conducted a Level 4 review of 7 patients with Type 3 and Type 4 duplication treated with an extra-articular modified
Bilhaut-Cloquet and found excellent IP joint range of motion, no nail deformities, and no episodes of growth arrest.

26) (OBQ05.267) In a patient with -2.5mm of ulnar variance, which of the following statements best describes the distribution of
compressive load across the wrist? QID: 1153

A. Approximately 50% of the wrist load is accepted by distal radius and 50% is accepted by the distal ulna
B. Approximately 80% of the wrist load is accepted by the distal radius and 20% is accepted by the distal ulna
C. Approximately 80% of the wrist load is accepted by the distal ulna and 20% is accepted by the distal radius
D. Approximately 95% of the wrist load is accepted by the distal radius and 5% is accepted by the distal ulna
E. Approximately 60% of the wrist load is accepted by the distal radius and 40% is accepted by the distal ulna

PREFERRED RESPONSE ▼ 4
Ulnar variance describes the cranio-caudal position of the distal ulna in relation to the distal radius at the wrist. In neutral ulnar
variance, 80% of the compressive load across the wrist is accepted by the distal radius, and 20% is accepted by the distal ulna.
With -2.5mm of ulnar variance (negative ulnar variance), approximately 5% of the wrist load is accepted by the distal ulna. With
+2.5mm of ulnar variance (positive ulnar variance), approximately 40% of the wrist load is accepted by the distal ulna. As
discussed in the biomechanical study by Palmer and Werner, the loading characteristics of the wrist are dependent on the
radio/ulnar variance. Specifically, a 2.5 mm increase in ulnar variance increases load accepted by ulno-carpal joint from 18% to
42%; a 2.5 mm decrease in the ulno-carpal variance will decrease the load accepted by the ulno-carpal joint to 4.3%. Friedman
and Palmer review the clinical diagnosis, pathophysiology, and treatment of ulnar impaction syndrome.

85
27) (OBQ11.265) A 44-year-old male factory worker presents with a 7-month history of pain and paresthesias involving the
palmar aspect of the left thumb, index finger, long finger, and the radial half of the ring finger. He reports that this often occurs at
night when trying to go to sleep. He has a history of anemia and obstructive sleep apnea. Percussion over the volar wrist crease
produces electric sensation distally in the hand and wrist flexion with the elbow in extension produces thumb paresthesias within
18 seconds. Figure A demonstrates a radiograph of the left hand. A sensory nerve conduction velocity test shows a distal
sensory latency of 5.7 ms. Which of the following is the most appropriate next step in management? QID: 3688

A
A. Phonophoresis and 6-week course of Vitamin B6 (pyridoxine)
B. Open carpal tunnel release
C. Wrist splinting
D. 1-month course of nonsteroidal anti-inflammatory drugs [NSAIDs] and physical therapy
E. 1-month course of bumetanide, smoking cessation, and physical therapy

PREFERRED RESPONSE ▼ 3
The patients history, examination, and nerve conduction velocity tests (normal distal sensory latency is <3.5 ms) are consistent
with carpal tunnel syndrome. There is Level 1 and 2 evidence supporting local steroid injection or splinting for the nonoperative
treatment of carpal tunnel syndrome. Phonophoresis, Vitamin B6 (pyridoxine), heat therapy, bumetanide, and physical therapy
are not considered the most appropriate options for carpal tunnel syndrome management.

The AAOS clinical guidelines for carpal tunnel syndrome consist of 9 clinical recommendations supported with a grading of the
recommendation and levels of evidence for the literature contributing to the recommendation.

The use of neutral wrist splints for carpal tunnel syndrome is most useful for improving night-time symptoms. However wrist
splinting is most functional at 30 degrees of extension, and the neutral splints can be functionally limiting when used during
productive daytime hours.
28) (OBQ04.48) A 67-year-old male has soft tissue defect on the palmar aspect of his right hand following a resection of mass
as shown in Figures A and B. Which of the following is most appropriate for achieving coverage of the defect? QID: 109

A B
86
A. Vacuum-assisted wound closure only
B. Flap coverage with full-thickness skin coverage
C. Wet-to-dry dressings only
D. Split-thickness skin coverage only
E. Flap coverage with meshed split-thickness skin coverage

PREFERRED RESPONSE ▼ 2
Soft tissue defects of the palm are most appropriately treated with flap coverage followed with full-thickness grafts. A flap is a
unit of tissue supported by blood vessels and moved from a donor site to a recipient site to cover a defect in tissue. This
patient's full-thickness coverage was created from a posterior interosseous artery island flap as shown in Illustrations A-C. The
skin of the dorsal hand is similar to that of the rest of the body and thus may be adequately replaced by split-thickness skin
grafts from the skin of most donor sites. In contrast, palmar hand skin differs from that of the dorsal hand in that it (1) lacks both
hair and sebaceous glands and (2) has specialized encapsulated nerve endings (Meissner’s corpuscles and Vater-Pacini
corpuscles) that confer enhanced sensation via mechanoreception. Full thickness skin grafts (FTSG) transfer all of the skin
appendages and nerve endings except those sweat glands located in the subcutaneous tissue and some of the Vater-Pacini
corpuscles of palmar and plantar skin. It is necessary to remove all fat and subcutaneous tissue from the undersurface of a full-
thickness skin graft, as this will otherwise act as a barrier preventing vascularization and graft survival.

Illustrations: A B C
29) (OBQ12.262) A 28-year-old NFL running back complains of continued hand pain three days following an injury sustained
while being tackled. He was splinted on the field. He has tenderness over the long finger metacarpal head, with subluxation of
the extensor tendon into the intermetacarpal area during active metacarpophalangeal joint flexion. A representative MRI is
shown in Figure A. What is the next best step in management of this patient? QID: 4622

A
A. Observation alone
B. Continued splinting in flexion
C. Continued splinting in extension
D. Open repair of the disrupted junctura tendinae
E. Open repair of the disrupted sagittal band

PREFERRED RESPONSE ▼ 5
Based on the history and physical exam findings this patient has sustained a traumatic rupture of the sagittal band. In this
professional athlete, the next best step would be to perform an open repair of the sagittal band. This will allow for earlier
aggressive rehabilitation and a quicker return to sport.

Sagittal band ruptures may be traumatic (as in this case) or attritional in nature (as in rheumatoid arthritis). A direct blow to the
MCP leads to forced flexion of the digit and subsequent stretching/rupture of the affected structure. On physical exam the
tendons are most unstable with the wrist flexed; MCP flexion will lead to dislocation of the tendon into the intermetacarpal gutter.
Acute injuries may be treated with extension bracing for 4-6 weeks, but in professional athletes, direct open repair of the sagittal
band is indicated.

Catalano et al. review sagittal band injuries treated with a thermally molded plastic splint that held the MCP in ~25-35 degrees of
hyperextension. Patients were evaluated over 14 months; out of 11 sagittal band injuries, splinting was successful in eight of
them. They recommend initial nonsurgical management with custom splinting.

87
Hame et al. review the results of the management of sagittal band injuries in the professional athlete. The lesion commonly
found was the disruption of the extensor mechanism with predictable sagittal band tears. In their series, all patients regained full
range of motion and returned to their respective sports. They recommend surgical intervention in elite athletes in the form of
extensor tendon centralization and sagittal band repair.

Figure A shows a T1 weighted axial cut of the affected hand; subluxation of the tendon (arrow) can be identified with disruption
of the sagittal band (arrowhead).

The video provided briefly reviews injury to the sagittal band.

Incorrect Answers
Answer 1: Observation is not indicated in this patient
Answer 2, 3: Splinting in extension would be an acceptable option in the non-athlete, but direct repair is indicated in a
professional athlete
Answer 4: The junctura tendinae are not injured in this patient

Illustrations: V
30) (OBQ06.92) A 20-year-old man has pain, swelling, and popping over his index metacarpophalangeal joint after striking a
wall a 3 days ago. Radiographs are normal, but physical exam reveals a palpable defect over the dorsum of the joint with
clenching of the fist, and this defect is resolved with extension of the metacarpophalangeal joint. What is the next most
appropriate step in treatment? QID: 203
A. Trigger finger steroid injection
B. Extension splinting of the metacarpophalangeal joint
C. Metacarpophalangeal synovectomy
D. Extensor hood reconstruction
E. Metacarpophalangeal joint arthrodesis

PREFERRED RESPONSE ▼ 2
“Boxer’s knuckle” refers to injury to the extensor hood mechanism that results following resisted extension ("flicking") of the
finger or direct trauma to the MP joint, usually involving the radial sagittal band of the middle or ring finger. Often, both the
sagittal band and the dorsal capsule are torn. The hallmark of the physical examination is pain over the MCP with a palpable
defect in the dorsal capsule, and it is important to examine for EDC subluxation with MP flexion. Sagittal band injuries seen
within 3 weeks of injury may be treated nonoperatively with an MP joint flexion blocking splint. Patients presenting later than 2 to
3 weeks after the injury or patients who failed a trial of splinting are candidates for surgical repair.

Hame et al reviewed 27 patients who were treated for Boxer’s Knuckle. The authors concluded that in cases in which
conservative treatment has failed, these injuries should be treated with sagittal band repair with centralization of the extensor
tendon without repair of the capsule. In the acute period however (less than 3 weeks), as is the scenario for this patient,
conservative management with extension splinting should be attempted first.

Araki et al peformed a study of 16 cases of rupture of the extensor hood initially treated conservatively with splinting. While 8/16
responded successfully to nonoperative management, the remaining 8 did not improve with conservative treatment and were
eventually treated with surgical repair and closure of the joint capsule when injured.

Illustration A shows a clinical image of a boxer's knuckle and Illustration B displays an axial T2 MRI with a sagittal band rupture.
Video V demonstrates a sagittal band reconstruction.

Illustrations: A B V

88
31) (OBQ12.251) An infant is brought to your office for evaluation of his hands. Clinical photos are shown in Figures A and B.
The clinical features are most consistent with a genetic mutation in which of the following: QID: 4611

A B
A. Sonic Hedgehog (SHH)
B. FGFR2
C. FGFR3
D. PMP22
E. COL1A1

PREFERRED RESPONSE ▼ 2
Based on the clinical features seen in the figures provided, the most likely syndrome is that of Apert syndrome, which is
consistent with a mutation in FGFR2.

Apert syndrome is an autosomal dominant condition that gives rise to facial dysmorphism and complex syndactyly of the hands.
The craniosynostosis that develops causes flattening of the skull and facial features.

Goldberg et al review congenital hand conditions and the malformations associated with them. They indicate that not only does
identification allow for natural history to be better elucidated, but also timing of surgical intervention can be better gauged.

Figures A and B demonstrate clinical features consistent with Apert Syndrome. The “rosebud” hand is a complex syndactyly that
affects the index, middle and ring fingers most commonly. Hypertelorism is exemplified with increased distance between the
eyes; additionally, acrocephaly is noted with forehead broadening and skull flattening.

Incorrect Answers

1: Mutation in sonic hedgehog gene (SHH) is associated with a longitudinal deficiency of the radius. This is seen in conditions
like TAR, Holt-Oram and VACTERL syndromes.
3: Mutation in FGFR3 leads to achondroplasia
4: Mutation in PMP22 gives rise to Charcot Marie Tooth syndrome
5: Mutation in COL1A leads to osteogenesis imperfecta

32) (OBQ05.21) A 34-year-old man sustains a finger flexor tendon laceration and undergoes operative repair. Which of the
following statements best describes the tendon motion rehabilitation protocol as depicted in Figures A where the splint holds the
wrist at 45 degrees of flexion? : 58

89
A. Low force and low excursion
B. Moderate force and potentially high tendon excursion
C. Low force and high tendon excursion
D. High force and high tendon excursion
E. High force and low tendon excursion

PREFERRED RESPONSE ▼ 1
The rehabilitation protocol depicted in Figure A is the Kleinert protocol which is categorized as a low force and low excursion
rehabiliation. This uses a dorsal blocking splint with the wrist in 45° of flexion and elastic bands secured to the patient’s nails and
a more proximal point on the splint. Once the interphalangeal (IP) joints are actively fully extended, recoil of the elastic bands
flexes them down passively. The Duran protocol (Illustration A) is similar but the wrist is in 20° of flexion and relies on the patient
to alternately passively extend the DIP and PIP joints with the other joints of the finger flexed. Early active motion protocols that
include "place and hold" finger exercises are considered moderate force and potentially high excursion protocols.

The review article by Lilly and Messer reports that synergistic motion protocols are low force and high tendon excursion and are
the best at minimizing peritendonous adhesions. In this splint, passive digit flexion is combined with active wrist extension,
followed by active digit extension coupled with active wrist flexion (Illustration B and C).

33) (OBQ10.97) A 42-year-old male diesel mechanic presents to your clinic 3 days after he was power washing automotive parts
with a high-pressure solvent and accidently hit the tip of his finger with the spray gun. A clinical photo is shown in Figure A. What
is the most appropriate first line of treatment? QID: 3191

A
A. Irrigation and debridement at the bedside using a digital block
B. Immediate debridement in the operating room
C. A dose of IV antibiotics in the ER, followed by a 10 day oral course
D. Immediate finger tip amputation
E. Arrange for follow-up with a hand specialist

PREFERRED RESPONSE ▼ 2
This patient suffered a high-pressure injection injury to his finger. These are uncommon soft tissue trauma injuries of the hand
which are frequently underestimated. The prognostic factors are the type, amount and temperature of the material and the
pressure of injection. Most of these hand injuries need to undergo immediate formal operative debridement and foreign body
removal. This ideally should be done under a formal operative setting to decrease infection risk, and to allow for microvascular
repair if needed. Delayed treatment can lead to further soft tissue damage and ultimately amputation.

The article by Bekler et al looked at the results of 14 surgically treated high-pressure injection injuries of the hand with a
minimum of two years follow-up. Ten of the injuries required formal operative debridement and foreign body removal. Six
required reconstructive microsurgical procedures and one underwent digital tip amputation. They concluded that high-pressure
injection injury to the hand is a significant problem, which can easily lead to serious sequelae and, even, amputation.
90
34) (OBQ09.229) A 32-year-old carpenter has a 6 month history of ulnar wrist pain that is worsened opening a jar, squeezing a
wet towel, typing, or changing a gearshift. Radiograph and MRI images are detailed in Figures A through C. All of the following
concerning ulnar shortening osteotomy are true EXCEPT: QID: 3042

A B C
A. Care should be taken to avoid the dorsal sensory branch of the ulnar nerve
B. Indicated for patients with concomitant Madelung deformity
C. Placement of the plate to the dorsal surface of the ulna can cause tendinitis of the extensor carpi ulnaris
D. Concomitant arthroscopy is indicated for patients who are seen to have cystic changes of the carpus on radiographs
E. Degenerative cystic changes of the ulnar carpal bones resolve after the ulnar shortening osteotomy

PREFERRED RESPONSE ▼ 2
Ulnocarpal impaction syndrome results from abutment of the ulnar head into the proximal ulnar aspect of the lunate. It is
worsened by activities that have wrist rotation and ulnar deviation. A positive ulnar variance with or without cystic changes of the
carpus is often seen on plain radiographs. In the absence of structural abnormalities, such as a Madelung deformity or DRUJ
arthrosis, the most commonly performed procedure is an ulnar shortening osteotomy. Coexisting central TFCC tears are
common and can be addressed by arthroscopic or open débridement. The Level 4 study by Baek et al describes 31 patients that
had improved Gartland and Werley scores following ulnar shortening osteotomy. They also noted that all patients with
degenerative cystic changes had resolution of the cysts at 1-2 year followup and they include a detailed outline of their surgical
technique. The Level 4 study by Chun et al showed very good outcomes with minimal complications and no ulnar nonunions.
35) (OBQ10.84) Cubital tunnel syndrome is caused by compression of the ulnar nerve between what two structures as it passes
posterior to the medial epicondyle? QID: 3172
A. Osborne's ligament and the MCL
B. MCL and Arcade of Struthers
C. Osborne's ligament and the intermuscular septum
D. MCL and medial head of the triceps
E. Ulnar and humeral heads of the flexor carpi ulnaris muscle

PREFERRED RESPONSE ▼ 1
The ulnar nerve passes posterior to the medial epicondyle and medial to the olecranon, then enters the cubital tunnel. The roof
of the cubital tunnel is primarily made up of Osborne's ligament, and the floor consists of the medial collateral ligament.

These soft tissue structures can cause narrowing of the tunnel, especially with elbow flexion, leading to ulnar nerve compression
and cubital tunnel syndrome. This is shown in Illustration A. The Arcade of Struthers is a band of deep fascia that attaches to the
intermuscular septum and covers the ulnar nerve 8 cm proximal to the medial epicondyle. The intramuscular septum is
continuous from the medial epicondyle to the coracobrachialis muscle. The ulnar nerve travels through the two heads of the
FCU distal to the cubital tunnel. These anatomic landmarks are shown in Illustration B.

Morrey evaluated 26 patients with post-traumatic contracture of the elbow who were treated with either operative release alone,
or operative release and distraction arthroplasty. Twenty-four (96%) of the patients had improved elbow function and two had
persistent ulnar neuritis treated with nerve transposition.

Cheung et al discuss the various surgical approaches to the elbow and the indications for each.

91
Video V is an educational lecture that discusses common nerve entrapment diagnosis and managment.
Illustrations: V

36) (OBQ10.170) A 24-year-old male cuts his left middle finger with a knife while chopping vegetables. Physical exam reveals a
zone 2 flexor tendon laceration. He undergoes a 2-strand core suture repair with epitendinous suture. This particular repair is
strong enough for each of the following rehabilitation protocols EXCEPT: QID: 3263
A. Kleinert protocol
B. Duran protocol
C. Synergistic motion protocol
D. Low force and low tendon excursion passive range of motion
E. Early digit active range of motion protocol

PREFERRED RESPONSE ▼ 5
Early active range of motion protocols are thought to decrease adhesions but risk rerupture or gap formation.

Strickland et al notes that the generation of muscle forces to either assist digit flexion or perform “place and hold” exercises
require at least a 4-strand core suture with epitendinous repair. This patient only had a 2-strand repair.

The Kleinert and Duran protocols are both forms of low force and low tendon excursion programs, that include passive digit
flexion range of motion. Kleinert includes a dorsal block splint with the wrist in 45° of flexion and elastic bands secured to the
patient’s nails and a more proximal attachment point. Once the interphalangeal joints are actively fully extended, recoil of the
elastic bands flexes them down passively. The Duran protocol utilizes the other hand to passively flex the affected DIP and PIP
joints and a higher amount of patient compliance is needed. Synergistic motion regimens allow passive digit flexion combined
with active wrist extension, followed by active digit extension coupled with active wrist flexion to produce low forces and high
tendon excursions at the involved digit.
37) (OBQ11.130) A 24-year-old racquetball player presents after accidentally striking his racket against the wall during a match
two months ago. He is tender to palpation over the hypothenar mass, and his pain is aggrevated by grasping. A radiograph and
CT scan of his wrist are shown in Figures A and B. Which of the following treatment methods has been definitively shown in the
literature to have a favorable outcome, and a high chance to return to pre-injury activities in patients with this injury? QID: 3553

A B
A. Activity restriction and continued monitoring
B. Open reduction and internal fixation
C. Casting for 6 weeks, followed by physical therapy
D. Corticosteroid injection and immediate return to play
E. Surgical excision
92
PREFERRED RESPONSE ▼ 5
The patients history and imaging are consistent with a subacute hook of the hamate fracture. This is demonstrated by the carpal
tunnel view radiograph in Figure A, and confirmed by the CT scan of the wrist in Figure B. CT scan of the wrist is usually
indicated to definitively diagnose these fractures. Current literature supports the most favorable results and ability to return to
pre-injury activities with excision of the fracture fragment. There is little available literature reporting the results of open reduction
and internal fixation of these fractures.

Rettig et al review traumatic wrist injuries in athletes. With regards to treatment of hook of the hamate fractures, they state that
ORIF and excision are the two viable treatment options in athletes. Of these, the literature supports fragment excision, which
has an average return to sport time of 7-10 weeks.

Welling et al determined which wrist fractures are not diagnosed with initial radiography, using CT as a gold standard and
identified specific fracture patterns. In their series, they found that only 40% of hamate fractures were diagnosed on plain
radiography, suggesting that CT should be considered after a negative radiographic finding if clinically warranted.
38) (OBQ10.247) A 4-year-old child has flattened facial features, wide set eyes, and the hand deformity pictured in Figure A.
Which of the following is the most likely diagnosis? QID: 3346

A
A. Apert's syndrome
B. Multiple epiphyseal dysplasia
C. Cleidocranial dysplasia
D. Noonan syndrome
E. Achondroplasia

PREFERRED RESPONSE ▼ 1
Apert's syndrome is a congenital disorder causing deformity of the skull, face, hands, and feet. It affects 1/80,000 children. An
autosomal dominant mode of inheritance exists, but the majority of new cases are sporadic. Early fusion of the cranial and facial
suture lines (craniofacial synostosis) results in a variety of skull and facial deformities. The primary deformity of the hands and
feet is severe syndactyly, often with fusion of the digits. The index, middle, and ring fingers are affected most often. Cognitive
function may be normal or moderately disabled.

Rebelo et al reviews the hand deformities of 170 patients with Apert's syndrome and their clinical outcomes following surgical
treatment.

Al-Qattan et al review the complex syndactyly of the hand associated with Apert's syndrome and suggests a new classification
scheme.

Illustrations A-C are further examples of the face, hand, and foot deformities associated with Apert's syndrome

Illustrations: A B C

93
39) (OBQ11.63) A 39-year-old male sustained an index finger injury 6 months ago and has failed eight weeks of splinting. A
radiograph taken at the time of injury is shown in Figure A, and a current radiograph is shown in Figure B. Which of the following
is true regarding open reduction and screw fixation of this injury? QID: 3486

A B
A. High risk of symptomatic implant
B. Immobilization of the distal interphalangeal joint is required for 2 weeks post-operatively
C. High rates of post-operative infection are common
D. Open reduction via an approach through the nail bed leads to significant post-operative nail deformity
E. Range of motion of the DIP joint in the affected finger is usually less than 10 degrees post-operatively

PREFERRED RESPONSE ▼ 1
Open reduction and internal fixation of distal phalanx fracture non-unions frequently requires the post-operative removal of the
fixation implant after complete fracture healing.

Chim et al followed 14 patients with non-union of fractures of the shaft of the distal phalanx who were treated with open
reduction and screw fixation. The implants required removal in 13/14 patients, and the mean post-operative range of motion of
the DIP joints was 56 degrees. No immobilization was required postoperatively, and bone grafting was only necessary in two
patients with severely comminuted fractures. Finally, the authors recommended approaching the fracture through the nailbed for
the best exposure, and found no postoperative nail growth complications. Postoperative infections were not common in their
series.

Mejis et al describe two patients with non-unions of the thumb distal phalanx treated with a single compression screw using a
minimally invasive approach. Both patients healed their fractures using this technique.
40) (OBQ09.67) A 10-year-old boy presents with a painless mass on the dorsal aspect of his wrist that has been present for 3
weeks. A clinical image is shown in Figure A. T1 and T2 magnetic resonance images are shown in Figure B and C, respectively.
On your exam, the mass transilluminates and Allen test reveals patent radial and ulnar arteries. What is the most appropriate
next step in management? QID: 2880

A B C
94
A. Referral to a orthopaedic oncologist
B. Surgical excision with wide margins
C. Observation
D. Autologus bone marrow aspirate injection
E. Injection of N-Butyl-Cyanoacrylate

PREFERRED RESPONSE ▼ 3
This child has a ganglion cyst on the dorsal aspect of his wrist. Imaging provided shows a well-marginated, homogenous signal
intensity mass consistent with a ganglion cyst. Physical examination findings of a mass transilluminating corroborate the MRI
findings of a ganglion cyst. Performing an Allen's test to evaluate radial and ulnar artery collateral blood flow is especially
important when evaluating ganglion cysts on the volar aspect of the wrist as they are often adjacent to the radial artery. Wang et
al. peformed a Level 4 review of 14 children with hand and wrist ganglion cysts and found that 79% of these cysts resolved
spontaneously within 1 year. Autologus bone marrow aspirate injection is a treatment option for unicameral bone cysts and N-
Butyl-Cyanoacrylate injections have been described for treatment of hemangiomas. Referral to an orthopaedic oncologist is not
indicated.
41) (OBQ09.234) Which of the following statements is true about force transmission based on wrist position?QID: 3047
A. neutral wrist position decreases force through the lunate fossa
B. extended wrist position increases force through the lunate fossa
C. neutral wrist position increases force through the scaphoid fossa
D. extended wrist position increases force through the scaphoid fossa
E. wrist position has no effect on force transmission

PREFERRED RESPONSE ▼ 4
Majima et al tested force transmission through the wrist in different positions in a cadaveric model. They found that force through
the scaphoid fossa is less in neutral and more in extension, while the force through the lunate fossa is more in neutral and less
in extension. The authors hypothesize that this may explain scaphoid fractures and intra-articular distal radius fractures as the
result of a fall onto an extended wrist.
42) (OBQ04.33) Extrinsic imbalance from splinting a crushed hand with metacarpophalangeal joint extension causes what
characteristic hand deformity? QID: 94
A. Distal interphalangeal joint extension
B. Ulnar subluxation of the metacarpophalangeal joints
C. Proximal interphalangeal joint extension
D. Proximal interphalangeal joint flexion
E. Swan-neck deformity

PREFERRED RESPONSE ▼ 4
Failure to splint the hand in an intrinsic positive position leads to increased extrinsic finger flexor tension, leading the DIP and
PIP joints to have an increasing flexion position. Illustration A and B show a clinical image and illustration of intrinsic minus hand.

von Schroeder et al present a Level 5 review of hand crush injuries. They conclude that early diagnosis and treatment is critical,
but the functional outcome is often poor with associated Volkmann's contracture.

95
43) (OBQ10.74) A 30-year-old female undergoes arthroscopy for a chronically painful right wrist that failed to improve with 4
months of immobilization and NSAIDS. Her clinical examination revealed point tenderness dorsally over the lunate but no
tenderness elsewhere in the wrist. A picture from the procedure is shown in Figure A where 'R' identifies the distal radius, 'L' the
lunate, and '*' represents a chondral flap. The articular surface of the lunate is stable to probing. A radiograph and MRI image of
the patients wrist are shown in Figures B and C respectively. What is the most appropriate next step in treatment? QID: 3162

A B C
A. Continue Immobilization and NSAIDS
B. Radial shortening osteotomy
C. Proximal row carpectomy
D. Scaphotrapeziotrapezoid fusion
E. Wrist fusion

PREFERRED RESPONSE ▼ 2
The patients clinical presentation and radiographs are consistent with Stage 2 Kienbock's disease in the setting of negative ulnar
variance. Radial shortening osteotomy is the most appropriate treatment option listed for Stage 2 disease which is defined as
lunate sclerosis without significant collapse. Shortening osteotomy can alter DRUJ contact pressures leading to remodeling,
especially in the presence of a Tolat Type II DRUJ, such as that shown in the radiographs. However, this remodeling has been
shown to occur without the development of arthritis, and therefore is not a contraindication to this procedure.

This patients radiographs shows some slight sclerosis of the lunate and negative ulnar variance, and the MRI shows diffuse
edema and early osteonecrosis of the lunate. The arthroscopic image shows a cartilage flap with a stable base left on the lunate.
Based on these images, the patient has Stage 2 disease and should be treated with a joint leveling procedure; or radial
shortening osteotomy in this case.

Sltusky et al provide a review article which focuses on the methodology behind a normal arthroscopic wrist examination and
discusses some of the more standard arthroscopic procedures along with the expected outcomes.

Bain et al review the arthroscopic staging of Kienbock's disease, and state that this techinique is a valuable assessment tool
which allows for not only classification of Kienbock's disease, but also may guide treatment.

Schuind et al. provide a review of the pathogenesis of Kienbock's. They conclude that the natural history of the condition is not
well known, and the symptoms do not correlate well with the changes in shape of the lunate and the degree of carpal collapse.
They also state that there is no strong evidence to support any particular form of treatment.

Illustration A shows a table which outlines the Stages of Kienbock's Disease.

Illustration B shows a table which outlines the general treatment options for each stage of Kienbock's Disease.

Incorrect Answers:
Answer 1: Immobilization and NSAIDS is indicated in Stage I disease or as a first line of treatment for Stage 2, which this patient
has failed.
Answer 3: Proximal row carpectomy is indicated in Stage 3B.

96
Answer 4: STT Fusion is indicated in Stage 3B.
Answer 5: Wrist fusion is indicated in Stage 4.

44) (SBQ07.38) A 32-year-old professional baseball player presents with wrist pain after a fall on his outstretched wrist 10 days
ago. He initially thought it was a sprain, but presents due to continued pain worsened by push-ups. His physical exam shows
dorsal wrist tenderness and is positive for the provocative test shown in Figure V. Standard PA radiograph of the wrist is normal.
Which of the following radiographic views shown in Figures A to E would be most helpful in establishing the diagnosis? QID:
1423 FIGURES: V

A. A
B. B
C. C
D. D
E. E

PREFERRED RESPONSE ▼ 1
The clinical description and video of the patient's physical exam are consistent with an acute scapho-lunate ligament tear. The
video shown in the question stem demonstrates the Watson test. When positive, the patient will feel dorsal wrist pain and/or a
"clunk" when the wrist is brought from extension/ulnar deviation to radial deviation. If plain radiographs are normal, a PA
clenched fist radiograph as seen in Figure A should be performed.

In patients with a acute scapho-lunate ligament tear, initial radiographs may not show the characteristic "Terry Thomas" sign, or
widening of the SL gap > 3mm. When making a clenched fist, the capitate is drawn proximally, stressing the SL ligament. This is
an easy view to obtain during the initial patient visit and should strongly be considered if this diagnosis is suspected.

Walsh et al review the various aspects of scapholunate ligament injuries. While they agree imaging is helpful in establishing the
diagnosis, they emphasize that wrist arthroscopy is the gold standard in the diagnosis of SL injuries.

Illustration A shows demonstrates a clenched fist view with obvious widening of the scapho-lunate gap.

Incorrect Answers
97
Answer 2: Shows a lateral radiograph in 30 degrees of supination. It is excellent for assessment of pisotriquetral arthrosis.
Answer 3: Shows a PA of the wrist in radial deviation. This view will actually close the SL gap.
Answer 4: Shows a a carpal tunnel view, used for assessment of hook of hamate fractures.
Answer 5: Shows a a stardard PA wrist in neutral aligment.

Illustrations: A
45) (OBQ09.36) A 22-year-old male snowboarder falls on an outstretched hand and presents with the radiograph shown in
Figure A. Which of the following techniques is MOST important in optimizing biomechanical fixation? QID: 2849

A
A. Using a longer screw placed in the central axis of the scaphoid
B. Using a supplementary K-wire transfixing the distal pole of the scaphoid to the capitate
C. Using a longer screw placed in the dorsal axis of the scaphoid
D. Using a larger diameter screw placed in the dorsal axis of the scaphoid
E. Using a larger diameter screw placed in the volar axis of the scaphoid

PREFERRED RESPONSE ▼ 1
Several studies have shown a longer screw placed in the central axis of the scaphoid optimizes biomechanical fixation of
scaphoid waist fractures. Many studies have discussed the amount of compression generated by various internal fixation screws
(e.g headless vs. headed, variable pitch, partially vs. fully threaded, cannulated vs. noncannulated), but it is believed that rigidity
of fixation is probably the most important factor in promoting healing of scaphoid fractures.

The first reference by McCallister et al is a cadaveric, biomechanical study that demonstrated a centrally placed screw had 43%
more stiffness than an eccentrically placed screw. They recommend using surgical techniques that optimize central placement
and screw length, such as using a cannulated screw.

The study by Dodds et al supported these findings and added that a longer screw with 2mm of bone coverage provided greater
stability than a shorter screw. A more centrally placed screw is generally longer and has more length of screw on each side of
the fracture than does a peripherally placed screw due to the anatomic dimensions of the scaphoid.
46) (SBQ11.1) A 40-year-old male sheet metal worker sustained a crush injury to his hand. His hand was treated in a short arm
splint after closed reduction and percutaneous pinning of multiple metacarpal fractures. The patient’s fractures healed
uneventfully however, he presented six months later with the deformity shown in Figure A. What pathoanatomic process is
responsible for his deformity? QID: 4236

A
98
A. Volar plate laxity and tethering of the lateral bands at the proximal interphalangeal joints
B. FDP laceration distal to the origin of the lumbricals
C. Adhered FDP tendon of the middle finger
D. Imbalance between spastic intrinsics and weak extrinsics
E. Imbalance between strong extrinsics and deficient intrinsics

PREFERRED RESPONSE ▼ 5
The clinical presenatation is consistent with a claw hand deformity characterized by MCP hyperextension and IP joint flexion.

Imbalance between strong extrinsics and deficient intrinsics is the pathoanatomic process of a claw hand, also called intrinsic
minus hand deformity. Intrinsic minus hand posture can result from a variety of causes including ulnar or median nerve palsy,
Volkmann’s ischemic contracture, leprosy, hereditary motor-sensory neuropathy, failure to splint a crush-injured hand using
intrinsic plus posture, or compartment syndrome of the hand, as was the case in this clinical vignette.

Ouellette et al performed a retrospective review of nineteen patients managed with fasciotomy for compartment syndrome of the
hand. They found that the most consistent clinical finding in making the diagnosis of compartment syndrome was a tense,
swollen hand in an intrinsic minus position.

Dellaero et al, in their review of compartment syndrome of the hand, discuss the etiology, diagnosis, and treatment of acute
hand compartment syndrome. They emphasize that the main goal in the management of ischemic contracture is restoration of
function; however the return of normal limb functionality is an unlikely result.

Figure A is a clinical photograph showing a classic claw hand deformity. Notice the MCP hyperextension and IP joint flexion.

Incorrect Answers:
1. Answer choice 1 describes the pathoanatomy of swan neck deformity leading to hyperextension of the PIP joint and DIP
flexion.
2. Answer choice 2 describes the mechanism for lumbrical plus deformity characterized by paradoxical extension of the IP joints
while attempting to flex the digits.
3. Answer choice 3 describes the mechanism for a quadrigia effect characterized by an active flexion lag in multiple digits
adjacent to an FDP injury.
4. Answer choice 4 describes the pathoanatomy of an intrinsic plus hand characterized by MCP flexion and IP joint extension.

47) (OBQ07.55) All of the following can be found on the electromyography (EMG) portion of an electrodiagnostic study during
the evaluation of a patient with carpal tunnel syndrome EXCEPT: QID: 716
A. Fibrillations at rest
B. Positive sharp waves
C. Decreased motor recruitment
D. Increased insertional activity
E. Increased distal sensory latency

PREFERRED RESPONSE ▼ 5
EMG's detect the electrical potential generated by muscle cells when these cells are electrically activated. They give information
about the muscle motor unit and can display the presence of fibrillations, sharp waves, motor recruitment, and insertional activity
of the muscle. The nerve conduction (NCV) portion of the electrodiagnostic study measures the speed at which the nerve
impulse travels down the axon. Large, myelinated nerve fibers conduct impulses the fastest and thus only these fibers are
evaluated in the nerve conduction portion of the electrodiagnostic study. Distal latencies and conduction velocities are measured
with NCV's. General parameters for NCV diagnosis of carpal tunnel syndrome include a distal motor latency of >4.5 msec, a
distal sensory latency of >3.5msec, or a conduction velocity of < 52 m/sec.

The articles by Brumback et al and Gooch et al is a review of electrodiagnostic studies for compression neuropathies.

99
48) (OBQ08.61) What is a potential complication of an amputation at the level of the distal interphalangeal joint? QID: 447

A. Central slip rupture


B. Swan neck deformity
C. Boutonniere deformity
D. Lumbrical plus finger
E. Quadrigia effect

PREFERRED RESPONSE ▼ 4
A lumbrical plus finger is descibed as paradoxical extension of the IP joints while attempting to flex the fingers. In the case a
lumbrical plus finger secondary to a DIP amputation, the PIP will extend upon attempted finger flexion.

The review article by Parkes describes how the lumbricals originate from the FDP. When the FDP is lacerated or amputated,
FDP contraction leads to pull on the lumbricals. This leads to shortening of the lateral bands and paraodoxical PIP and DIP
extension. (Illustrations A-C). There are several causes of lumbrical plus finger including (1) FDP laceration or rupture distal to
the lumbrical origin, (2) amputation of the DIP distal to central slip insertion, and (3) excessively long flexor tendon graft.
Treatment consists of lumbrical release at the level of the flexor sheath in the palm, which then prevents paradoxical PIP
extension.

Quadrigia may occur when the profundus is advanced of greater than 1 cm in repair. The FDP tendons share a common muscle
belly, and distal advancement of one tendon will effect the flexion strength of the adjacent digits.
V

49) (OBQ07.102) A 28-year-old man fell while ice skating 6 months ago and has had ulnar-sided wrist pain ever since. The
patient's lateral radiograph of the wrist is shown in Figure A and a CT scan is shown in Figure B. What is the most appropriate
treatment? QID: 763

A B
A. Scapholunate ligament repair
B. Excision of the hook hamate
C. Excision of the pisiform
D. Open reduction internal fixation of the hamate
E. Open reduction internal fixation of the pisiform

PREFERRED RESPONSE ▼ 3
100
Based on clinical history and imaging shown, this patient has developed a pisiform fracture nonunion. Treatment of symptomatic
nonunions of the pisiform is by pisiformectomy

Fractures of the pisiform are rare. They often occur in conjunction with injuries to the distal radius or carpus. Non-operative
management with cast immobilization in 30 degrees of wrist flexion is the first line of treatment. Symptomatic nonunions are
treated with pisiformectomy.

Palmieri et al. performed pisiformectomies on 21 patients who had pisiform area pain that was refractory to conservative
management. Patients had a history of painful union or nonunion of pisiform fractures, arthritis or FCU tendonitis. In all cases,
wrist strength and mobility was retained.

Lam et al. reviewed the effect of pisiform excision on wrist function in patients with piso-triquetral dysfunction. After an average
follow up of 65 months, 75% of patients had complete relief of pisiform area symptoms. No differences in grip, wrist motion,
strength or power were found in comparison to the contralateral side.

Figure A shows a lateral radiograph of a pisiform fracture nonunion. Figure B shows an axial CT scan sequence of the wrist. A
pisiform fracture nonunion is identified with subtle comminution. The pisotriquetral joint appears to be congruent.

Incorrect Answers
Answer 1: The scapholunate ligament is not affected in this clinical situation.
Answers 2, 4: Although the hook of hamate can be a source of ulnar sided pain, it is not implicated in this clinical situation
Answer 5: An ORIF of the pisiform is not typically used for symptomatic pisiform fracture nonunions
50) (OBQ05.3) The sural artery provides the vascular supply to which musculocutaneous flap? QID: 40
A. soleus
B. gastrocnemius
C. latissimus dorsi
D. tibialis anterior
E. peroneus longus

PREFERRED RESPONSE ▼ 2
The sural artery supplies the both heads of the gastrocnemius and is the pedicle for rotational flaps. Eighty-five percent of the
time there is a single vascular source.

----------------------------------------------------------------------------(150)-------------------------------------------------------------------

1) (OBQ05.66) Axon regeneration almost always occurs following a Sunderland second-degree nerve injury because which
anatomic structure is not injured? QID: 952

A. Epineurium
B. Endoneurium
C. Perineurium
D. Myelin sheath
E. Schwann cell

PREFERRED RESPONSE ▼ 2
Following a Sunderland second-degree injury, axon regeneration is possible because the endoneurium is intact.

There are two classification schemes for peripheral nerve injuries, which include the Seddon and the Sunderland systems.
Under the Sunderland classification, a second-degree injury is considered a part of the axonotmesis spectrum. The
endoneurium, perineurium and epineurium are still intact. This enables complete functional recovery.

Lee et al. review the pathophysiology and evaluation of peripheral nerve injuries. They note that in Sunderland type two injuries,
there is physiologic disruption of the axons. Because the endoneurium is still intact, axons are able to regenerate. This process
takes months.
101
Illustration A is a schematic of the various stages of peripheral nerve injury.

Incorrect Answers
Answers 1, 3: Although the epineurium and perineurium are intact in a Sunderland type 2 injury, axon regeneration is possible
because of an intact endoneurium.
Answers 4, 5: The myelin sheath and Schwann cells are disrupted in axonotmesis.

2) (OBQ11.125) Figure A depicts a child with syndactyly. Following surgical treatment, the most common complication involves
which of the following? QID: 3548

A
A. Nail plate
B. Nail bed
C. Web commissure
D. Radial digital nerve
E. Radial digital artery

PREFERRED RESPONSE ▼ 3
Web creep, the most common complication of this procedure, is the distal migration of the web commissure seen in surgically
corrected syndactyly patients. It is caused by abnormal scar tissue formation and increasing growth of underlying osseous
structures. Informing parents of this complication preoperatively is advised.

Deunk et al reviewed the long-term results of 27 patients treated with either STSG or FTSG at 21 years. The STSG group had
increased flexion and extension lags but decreased finger abduction and increased graft breakdown. The FTSG had higher
rates of web creep, hyperpigmentation and hair presence. The authors did not favor one technique over the other.

Ricterman et al performed a radiographic analysis of web height in children. They were able to radiographically determine a
standard web height quantification system using anatomic landmarks. This technique serves as the foundation for measuring
web creep in syndactlyly.

Illustration A is a clinical photo demonstrating web creep as a late complication.

102
3) (OBQ12.85) A 27-year-old male presents with finger pain 2 days after suffering an injury while playing basketball. Physical
exam shows swelling of the distal interphalangeal joint with no evidence of open injury. A radiograph is shown in Figure A.
Which of the following is the most appropriate treatment at this time? QID: 4445

A
A. Extension splinting of DIP joint for 6-8 weeks
B. Closed reduction and percutaneous pinning
C. Open reduction and internal fixation
D. DIP arthrodesis
E. Swan neck deformity correction

PREFERRED RESPONSE ▼ 1
The clinical presentation is consistent with a non-displaced bony mallet finger without joint subluxation. Extension splinting of the
DIP joint for 6-8 weeks is the most appropriate treatment.

A mallet finger is a deformity caused by disruption of the terminal extensor tendon distal to DIP joint. Treatment is dictated by the
degree of displacement and acuity of injury. Acute injuries with minimal displacement and no joint subluxation are treated with
extension bracing for 6-8 weeks. ORIF or closed reduction and percutaneous fixation is indicated for chronic injuries or acute
injuries with volar displacement of the distal phalanx, a >2mm articular step-off, or when a majority (>50%) of the articular
surface is involved.

Pegoli et al. report the results of extension block Kirschner wire fixation for the treatment of mallet fractures of the distal phalanx
in 65 consecutive patients. Their results showed 46% excellent, 32% good, 20% fair and 2% poor results. The recommend the
following indications for operative treatment: presence of a large bone fragment, palmar subluxation, or the loss of joint congruity
of the distal interphalangeal joint.

Theivendran et al. report operative fixation is indicated when more than 30% of the articular surface is involved with or without
subluxation of the joint. They summarize the management options for intra-articular distal interphalangeal fractures, placing
particular emphasis on surgical treatment.

Figure A shows a non-displaced bony mallet Injury. Illustration A shows an example of an extension splint used for non-
operative management of mallet injuries. Illustration V is a video showing the surgical technique for a Mallet finger.

Incorrect Answers:
Answer 2: Closed reduction and percutaneous pinning is indicated for a displaced mallet finger injury with joint subluxation.
Answer 3: Open reduction and internal fixation is indicated for displaced, subluxed mallet finger injuries that can not be reduced
closed.
Answer 4: DIP arthrodesis is indicated in patients with a painful, stiff, arthritic DIP joint.
Answer 5: Swan neck deformity correction is indicated for a chronic mallet finger that has led to a swan neck deformity.

Illustrations: A V

103
4) (OBQ10.258) A 38-year-old woman complains of a painful finger mass of 4 months duration. A photograph of the mass is
provided in Figure A. The decision is made to proceed with surgical excision. Which of the following is an advantage of surgical
excision with joint debridement as opposed to aspiration? QID: 3354

A
A. Reduced rate of infection of the DIP joint
B. Less post-procedure pain
C. Improved DIP range of motion
D. Decreased risk of mass recurrence
E. Reduced risk of metastasis from seeding the mass into the joint

PREFERRED RESPONSE ▼ 4
Figure A demonstrates a mucous cyst. This benign mass originates from the DIP joint, and is secondary to arthritis. It may be
treated with aspiration or surgical excision. However, recurrence occurs frequently with aspiration. Debridement of any
osteophytes from the DIP joint is crucial to preventing recurrence with surgical excision. Rizzo et al retrospectively evaluated the
results of 154 mucous cysts treated with either aspiration or surgery. Aspiration resulted in a 40% recurrence rate. There were
zero recurrences with surgical excision and joint debridement.
5) (OBQ04.120) Using the Blauth classification of thumb hypoplasia, grade IIIA can be treated with thumb reconstruction
whereas grade IIIB is treated with thumb amputation & pollicization. What is the key difference between these two grades? 1225

A. Presence of complete osseous structures


B. Presence of intact musculotendinous structures
C. Carpometacarpal joint stability
D. Metacarpophalangeal joint stability
E. Presence of an extensor pollicis longus tendon

PREFERRED RESPONSE ▼ 3
The key difference between a Blauth IIIA and IIIB is the presence of carpometacarpal joint stability in Blauth IIIA. The CMC joint
must be sufficiently stable to provide resistance during grasp and pinch.

Thumb hypoplasia is a rare congenital deformity affecting 1 in 100,000 live births. It is commonly associated with syndromes
such as VACTERL, and is classified using the Blauth classification. To function correctly, the thumb must be positioned so that it
can oppose the adjacent medial fingers and grasp objects securely from an antiposed (abducted, slightly extended, and
pronated) position. The decision to treat with reconstruction vs. amputation and pollicization depends on carpometacarpal joint
stability.

Light and Gaffey discussed the management of the hypoplastic thumb. Types II and IIIA (intact CMC joint) can be treated with
thumb reconstruction, whereas Types IIIB to V are treated with pollicization or amputation.

Tonkin reviewed the classification of congenital thumb hypoplasia and suggested a modification to the Blauth Classification.
Illustration A shows the Blauth Classification.

Incorrect Answers:

104
Answer 1: Type II has complete osseos structures. Types IIIA and IIIB both have osseous deficiencies.
Answer 2: Type I has complete musculotendinous structures. Types IIIA and IIIB both have musculotendinous deficiencies.
Answer 4: Types IIIA and IIIB both have absence of MCP stability.
Answer 5: Types IIIA and IIIB both tendon deficiencies.

Illustrations: A
6) (OBQ11.18) A 25-year-old female is involved in a motorcycle collision and presents with the injuries seen in Figures A
through D. What is the best option for definitive management of the injuries seen in Figure D? QID: 3441

A B C D
A. Open reduction and internal fixation
B. Closed reduction and casting
C. External fixation
D. Immediate therapy
E. Removable splint

PREFERRED RESPONSE ▼ 1
The patient presents with mutliple injuries including a subtalar dislocation (Figure A), femoral shaft fracture (Figure B), tibia shaft
fracture (Figure C) and multiple metacarpal shaft fractures (Figure D). Multiple metacarpal shaft fractures are best managed with
open reduction and internal fixation as non-operative management is associated with loss of motion, asynchronous grasp and
decreased grip strength.

Souer and Mudgal retrospectively reviewed their experience treating patients with multiple metacarpal fractures utilizing hand-
specific implants. They argue that rigid internal fixation of multiple metacarpal fractures allows for early mobilisation and tendon
excursion, and found excellent results in 18 of 19 patients with a 230 degree total arc of motion.

Kawamura and Chung review fixation options for treating unstable oblique phalangeal and metacarpal fractures. They found low
complication rates regarding tendon adhesion and stiffness with published studies examing dorsal plating of oblique metacarpal
fractures as the extensor tendons are less adherent to bone at the level of the metacarpal.

Incorrect Answers:
Answer 2. Closed reduction and casting would lead to stiffness due to immobilization
Answer 3. External fixation would bind the extensor mechanism and would not allow for early motion
Answer 4. Immediate therapy, although beneficial, would be difficult to accomplish without rigid fixation
Answer 5. Removeable splinting would not facilitate early motion and and would likely lead to loss of metacarpal length and
deformity as the stabilizing effect of the adjacent metacarpals is lost with multiple fractures
7) (OBQ06.45) What is the most important factor influencing immediate and late outcome of digit replantation? QID: 156
A. Gender
B. Anastamosing the proximal profundus tendon stump to the superficialis tendon of the amputated digit
C. Regular cigarette smoking
D. Utilization of composite skin and subcutaneous vein graft
E. Mechanism of digit amputation injury

105
PREFERRED RESPONSE ▼ 5
The Level 2 study by Waikakul et al is a cohort of 552 patients that underwent 1018 digit replantation. Mechanism of injury was
the most important factor influencing the survival rate with an odds ratio of 46.3. Specifically, avulsion, degloving and extensive
crushed amputation resulted in a low survival rate and poor functional outcome. Cigarette smoking and male gender were
associated with worse results but not to the degree of the mechanism of injury. Utilization of composite skin and subcutaneous
vein grafts as well as connecting the profundus to the superficialis at the anastomoses correlated to better outcomes. After the
operation, 329 of 946 survived digits (34.7%) in 180 patients (35.4%) needed further reconstructive surgery to improve their
function. Tenolysis was the most common procedure followed by staged tendon grafting and capsulotomy.

The review article by Wang found that tendon procedures, specifically tenolysis, accounted for 47.2% secondary surgeries
following digit replantation.
8) (OBQ07.142) Apert's syndrome is caused by a mutation in what gene? QID: 803

A. Fibroblast growth factor receptor 2 (FGFR2)


B. Fibroblast growth factor receptor 3 (FGFR3)
C. Collagen type II alpha 1 chain (COL2A1)
D. SED late (SEDL)
E. Fibrillin

PREFERRED RESPONSE ▼ 1
Apert's syndrome (acrocephalosyndactyly type 1) is characterized by anomalies of the cranium, hands, and feet. Mutations in
the FGFR2 gene cause Apert syndrome.

Anderson et al report that in Apert's syndrome there is widespread anomalies of the feet, with defects including both predictable
dysmorphic changes and progressive fusions of the skeletal components during skeletal maturity.

Incorrect Answers:
2: Achondroplasia is related to abnormalities in the FGFR3, not FGFR2.
3: SED congenita is caused by mutations in COL2A1 (type II collagen alpha 1 chain) on chromosome 12. These result in
abnormal type II collagen.
4:The X-linked form of SED tarda is caused by mutation in SEDL (SED late) gene.
5: Marfan syndrome is caused by defects in the fibrillin gene.
9) (OBQ10.110) Which of the following patients with Dupuytren's contracture would benefit the most from dermatofasciectomy
and full-thickness skin grafting opposed to traditional fasciectomy? QID: 3204
A. 70-year-old sedentary male with small finger involvement isolated to the MCP joint
B. 50-year-old male systems analyst with ring and small finger involvement limited to the MCP joints
C. 65-year-old female golfer with ring and small finger involvement including MCP and PIP joints
D. 40-year-old female stenographer with middle, ring, and small finger involvement including MCP and PIP joints with 50 and 55
degree contractures of ring and small finger MCP joints, respectively
E. None of the above as no difference in outcome has been demonstrated between the two procedures

PREFERRED RESPONSE ▼ 5
Dermatofasciectomy and full-thickness grafting has not demonstrated superior finger range of motion, recurrence rate, or patient
satisfaction in comparison with traditional fasciectomy.

The main reference from Ullah et al conducted a prospective randomized study of 84 Dupuytren's cases treated with
fasciectomy alone or dermatofasciectomy with full-thickness skin grafting. The question was whether the overlying skin needed
to be excised. No difference in clinical outcome or recurrence rate was discovered.

Roy et al reviewed 79 cases of advanced Dupuytren's treated with radical fasciectomy (but preservation of the overlying skin)
and then adding full-thickness skin grafting to the open areas once the fingers were extended. They found their results of
fasciectomy to be similar to those published for dermofasciectomy.
Indications for sx: Hueston test +ve(inability to lie hand flat,MCP flex contracture >30,any PIP contracture

106
10) (OBQ08.30) Which of the following nerves has the most favorable regenerative potential in restoring motor function after a
graft repair within half a year after being injured? QID: 416

A. Median
B. Ulnar
C. Radial
D. Tibial
E. Peroneal

PREFERRED RESPONSE ▼ 3
Of the choices listed, the radial nerve has the best opportunity for recovery.

Roganovic performed a prospective study of 393 graft repairs of the median, ulnar, radial, tibial, peroneal, femoral, and
musculocutaneous nerves which showed that peripheral nerves differ significantly regarding the motor recovery potential, and
the difference depends on the level of nerve repair. The following nerves had excellent recovery potential: the radial,
musculocutaneous, and femoral nerves. The following nerves had moderate recovery potential: the median, ulnar, and tibial
nerves. The following nerve had poor recovery potential: the peroneal nerve.

Mohler et al, recommends testing nerve action potentials at the time of nerve exploration to guide surgical treatment.

11) (OBQ12.213) Which of the following hand injuries seen in Figures A-E is most appropriately treated with a first dorsal
metacarpal artery flap? QID: 4573

A. Figure A
B. Figure B
C. Figure C
D. Figure D
E. Figure E

PREFERRED RESPONSE ▼ 3
Figure C shows a dorsal thumb laceration with exposed tendon that would be most appropriately treated with a first dorsal
metacarpal artery (FDMA) flap.

The first dorsal metacarpal artery is a branch of the radial artery that supplies the dorsal hand skin from the thumb metacarpal to
the long metacarpal, as well as the skin on the dorsal surfaces of the thumb and index to the proximal interphalangeal joint. The
flap is raised distal to proximal as an island flap containing the FDMA, branches of the radial nerve, fascia of the underlying
interosseous muscle of the first web space, and skin overlying the MP joint and proximal phalanx of the finger. It is an excellent
option for large soft tissue defects on either side of the thumb. In this case, skin grafting is contraindicated because of exposed
tendon without paratenon.

Sherif et al. detail the anatomy of the first dorsal metacarpal artery. They found three consistent branches, including the radial,
ulnar, and intermediate branch. In part II of their study, they review the results of 23 patients where the FDMA flap was used as
a fasciocutaneous or fascial flap for the coverage of soft tissue hand defects.

107
Illustration A shows a FDMA flap being raised for coverage of a thumb defect.

Incorrect Answers:
Answer 1: Fingertip amputations with minimal soft tissue loss and no exposed bone can be allowed to heal through secondary
intention.
Answer 2: The posterior interosseous fasciocutaneous flap is an excellent option for lacerations to the first web space.
Answer 4: This large soft tissue defect on the dorsum of the hand may be treated with a groin flap.
Answer 5: Fingertip amputations with exposed bone are best treated with local advancement flaps such as a VY advancement
flap.
Illustrations: A
12) (OBQ05.256) What is the most important variable when determining treatment of a hypoplastic thumb? QID: 1142
A. contralateral thumb function
B. thenar muscle function
C. presence of an extensor pollicis longus tendon
D. carpometacarpal joint stability
E. metacarpophalangeal joint stability

PREFERRED RESPONSE ▼ 4
Congenital thumb hypoplasia presents as a wide range of first ray deficiencies and frequently occur as part of a larger systemic
syndrome such as Holt-Oram, Fanconi’s, or Apert’s syndrome. To function correctly, the thumb must be positioned so that it can
oppose the adjacent medial fingers and grasp objects securely from an antiposed (abducted, slightly extended, and pronated)
position. Although normal motion is usually not required at the MP or interphalangeal joints, thumb function is greatly dependent
on preserving a full arc of circumduction at the carpometacarpal (CMC) joint. The CMC joint must be sufficiently stable to
provide resistance during grasp and pinch. The Blauth's classification ranges from type I to V and Types IIIB to V are treated
with pollicization. The key difference between a Blauth IIIA and IIIB is the presence of carpometacarpal joint stability in Blauth
IIIA.
13) (OBQ13.95) A 55-year-old female patient presents with pain along the thumb ray and increasing deformity of her right hand.
Key pinch causes her pain. The appearance of her hand is seen in Figure A. Range of motion of her thumb is seen in Figure B.
What is the most likely cause of her deformity? QID: 4730

A B
A. Type II hypoplastic thumb
B. Median nerve neuropathy
C. Lupus thumb deformity
D. Extensor tendon rupture
E. Osteoarthritis of the trapeziometacarpal joint

PREFERRED RESPONSE ▼ 5
The patient has 1st carpometacarpal (CMC) arthritis.

With 1st CMC arthritis, the patient avoids painful thumb abduction and an adduction deformity gradually develops, with 1st
webspace contracture. With progressive 1st CMC stiffness, the thumb metacarpophalangeal joint (MCP) develops
hyperextension deformity to compensate for the loss of motion, leading to a secondary "Z" deformity.

Rozental et al. reviewed hand and wrist reconstruction. They believe that arthrosis arises from loss of the anterior oblique
("beak") ligament. Compensatory MCP hyperextension should be treated with MCP capsulodesis or arthrodesis.

Van Heest et al. reviewed thumb CMC arthritis. Treatment for Eaton stage I/II arthritis is open/arthroscopic debridement, volar
ligament reconstruction (with APL or FCR tendons), or metacarpal extension osteotomy. For stage III/IV arthritis, treatment
108
options include implant arthroplasty or resection arthroplasty +/- LRTI (with APL, FCR or palmaris longus), and fusion (young
patients).

Figure A shows adduction contracture of the 1st webspace, with hyperextension deformity of the 1st MCP joint. Figure B
illustrates decreased thumb abduction because of adduction contracture with decreased palmar abduction (normal, 45deg) and
decreased radial abduction (normal, 60deg). Illustration A is a radiograph showing thumb CMC arthritis with Z deformity.
Illustration B shows lupus thumb deformity ("hitchhiker thumb"). Illustration C shows hand changes in inflammatory arthritis.

Incorrect Answers:
Answer 1: Type II thumb hypoplasia does not present with "Z" deformity.
Answer 2: Chronic median nerve neuropathy leads to flattening of the thenar eminence and an ape hand deformity from loss of
opponens pollicis. There is no loss of thenar bulk or thumb opposition in Figure A.
Answer 3: There is no lupus thumb deformity (Illustration A). Lupus thumb deformity is characterized by flexion at the MCP and
hyperextension at the interphalangeal joint.
Answer 4: Extensor tendon rupture will not lead to hyperextension deformity of the MCP joint.

14) (OBQ09.97) You are seeing a 26-year-old man after he was involved in a knife fight. He has pain when flexing and
extending his index finger. You explore a 2 centimeter wound in zone 2 and find his flexor tendons to the index are 40%
lacerated. What is the preferred method of treatment? QID: 2910

A. Trim the frayed tendon edges and begin early range of motion
B. Trim the frayed tendon edges and cast in an intrinsic positive position for 2 weeks
C. Peritendinous 6/0 and Core 4/0 suture repair
D. Core 4/0 suture repair
E. Core 6/0 suture repair

PREFERRED RESPONSE ▼ 1
The management of partially lacerated flexor tendon injuries is controversial.

Bishop et al. developed a nonweightbearing canine model for testing partial tendon lacerations and found early motion improved
tendon excursion/stiffness resulting in more normal morphology.

McGeorge and Stillwell compared the results of repair versus non-repair in humans for zone 2 injuries and concluded that
tendons lacerated by 60% or less should not be repaired.

109
15) (OBQ12.64) A 54-year-old male presents with a slowly enlarging mass on the dorsum of his left wrist which has been
present for 3 years. He denies any significant symptoms. Physical exams shows a 1 cm palpable mass. A MRI is shown in
Figure A. A biopsy of this lesion would most likely show? QID: 4424

A
A. Synovial cells with mucin accumulation
B. Proliferating histiocytes of moderate cellularity and frequent multinucleated giant cells
C. Polymorphonuclear neutrophils
D. Spindle cells arranged in intersecting bundles
E. Lipocytes, spindle cells, and scattered atypical giant cells

PREFERRED RESPONSE ▼ 1
The clinical presentation is consistent with a ganglion cyst. Histology of a ganglion cyst would show a mucin filled synovial cyst.

Ganglion cysts are the most common mass found on the hand or wrist. Dorsal ganglions originating from the scapholunate (SL)
ligament are the most common (60%). They are caused by trauma, mucoid degeneration, or synovial herniation. On exam, they
appear fixed to the underlying deep tissue, but not to the skin and are commonly translucent to light illumination. Radiographs of
a ganglion will be normal, although a T2-weighted MRI axial image of the wrist will show increased signal where the cyst is
located.

Nahra et al. give a thorough review of ganglion cysts including known epidemiology, etiology, and treatment. He notes that
dorsal ganglions are the most common (60% to 70%) and are found between the third and fourth dorsal compartments arising
most commonly from the scapholunate ligament. Supportive splints and anti-inflammatories in conjunction with aspiration are an
important part of nonoperative management. Aspiration of dorsal ganglions (not recommended for volar ganglions)(due to artary
proximity) yields a recurrence rate of around 50%. Surgical intervention is not common, but there is a low recurrence rate when
performed.
Peh et al. reviewed the MRI features found for benign soft tissue masses in the hand, especially ganglion cysts. They noted that
ganglion cysts show an increased signal intensity on T2-weighted MRI images.
Figure A is a T2 weighted MRI showing signal intensity in the fluid of the cyst consistent with a ganglion cyst. Illustration A is a
clinical photo of a ganglion cyst.

Illustration B shows a histologic specimen of a ganglion cyst


Incorrect Answers:
Answer 2: Proliferating histiocytes of moderate cellularity and frequent multinucleated giant cells would be consistent with Giant-
cell tumor of tendon sheath.
Answer 3: Polymorphonuclear neutrophils would be the primary cell type with infection and are the predominant cells in pus,
accounting for its whitish/yellowish appearance.
Answer 4: Spindle cells arranged in intersecting bundles would be consistent with a Schwannoma.
Answer 5: Lipocytes, spindle cells, and scattered atypical giant cells are the histologic findings of a pleomorphic lipoma.

110
16) (OBQ12.49) A 34-year-old male sustains the closed finger injury shown in Figure A one week ago. He undergoes closed
reduction and pinning shown in Figure B to correct alignment. Which of the following is responsible for the apex palmar fracture
deformity noted on the preoperative radiographs? QID: 4409

A B
A. Indirect pull of the central slip on the distal fragment and the interossei insertions at the base of the proximal phalanx
B. Intrinsic muscle fibrosis and intrinsic minus contracture
C. PIP joint volar plate attenuation and extensor tendon disruption
D. Rupture of the central slip with attenuation of the triangular ligament and palmar migration of the lateral bands
E. Flexor tendon disruption with associated overpull of the extensor mechanism

PREFERRED RESPONSE ▼ 1
The clinical presentation is consistent with a transverse proximal phalanx fracture. These fracture have an apex palmar
angulated deformity under the indirect pull of the central slip on the distal fragment and the interossei insertions at the base of
the proximal phalanx.

If proximal phalanx fractures are allowed to heal with the apex palmar deformity, an extensor lag will result. Therefore CRPP or
ORIF is indicated in transverse fractures with > 10° angulation. To correct this deformity prior to surgical fixation, the MCP joint
should be flexed, which allows the extensor mechanism as a whole to function as a tension band to help reduce the fracture.
This is referred to as intrinsic plus splinting. Collateral ligament, capsule, and intrinsic muscle attachments render transverse
fractures in the proximal 6 to 9 mm of the P1 base more stable than fractures located distally.

Henry provides a review of fractures of the proximal phalanx and metacarpals. He states that most transverse or short oblique
P1 fractures without comminution are best stabilized by two 0.045-inch K-wires placed longitudinally through the fully flexed
MCP joint. A single wire alone risks rotational malunion, but some fracture patterns may provide inherent rotational stability that
would allow use of one wire for angular control.

Figure A shows a transverse fracture of the proximal phalanx with apex volar angulation. Figure B shows two K-wires placed
transarticular through the MCP joint in a flexed (intrinsic plus) posture to correct the deformity and stabilize the fracture.

Incorrect Answers:
Answer 2: Intrinsic muscle fibrosis and contracture is usually associated with chronic crush injuries and significant soft tissue
damage.
Answer 3: This is describing a swan neck deformity.
Answer 4: This is describing a Boutonnierre deformity.
Answer 5: Flexor tendon disruption is not likely in this closed injury pattern.

111
17) (OBQ13.225) A 28-year-old man sustained a complete laceration of the flexor digitorum profundus of his index finger while
cutting a watermelon 3 days ago. A clinical photograph is shown in Figure A. The surgeon plans to repair the tendon using a 4-
strand core suture technique. Which method of tendon repair will give him the best results in terms of load to failure and gliding
resistance? QID: 4860

A
A. Repair with core suture purchase 5mm from the cut edge only. No epitendinous suture
B. Repair with core suture purchase 10mm from the cut edge only. No epitendinous suture
C. Repair with core suture purchase 5mm from the cut edge. Circumferential simple running epitendinous suture.
D. Repair with core suture purchase 10mm from the cut edge. Circumferential Silfverskiold epitendinous suture.
E. Repair with core suture purchase 10mm from the cut edge. Circumferential simple running epitendinous suture.

PREFERRED RESPONSE ▼ 5
Repair with core suture purchase 10mm from the cut edge, coupled with circumferential simple running epitendinous suture will
give him the best load to failure and gliding resistance.

The strength of tendon repairs depend on the number of strands crossing the repair site. Ideally, repairs should have 4-6 strands
to allow for early active motion. A running epitendinous suture is recommended to improve tendon gliding and repair strength.

Gulihar et al. compared 3 different epitendinous suture techniques. They found that compared with an intact tendon, gliding
resistance increased 100% with the Halsted repair, 80% with the Silfverskiold repair and 60% with a running suture. They thus
recommend a simple running suture when an epitendinous suture is needed.

Lee et al. compared core suture purchase at 3, 5, 7 and 10mm from the cut edge. The 10mm-repair group had the highest 2-mm
gap force and ultimate failure load. They recommend 10-mm suture purchase for optimal performance and to allow early active
motion.

Figure A shows a laceration to the volar aspect of the index finger in flexor zone II. Illustration A shows a core suture purchase
distance from the cut edge (represented by "X", where 10mm is the ideal distance). Illustration B shows 3 different epitendinous
suture techniques (A, simple running; B, Silfverskiold; C, Halsted).

Incorrect Answers:
Answers 1, 3: Suture purchase 5mm from the cut edge is inferior to purchase at 10mm from the cut edge.
Answer 2: An epitendinous suture adds to repair strength and improves gliding compared with no epitendinous suture.
Answer 4: The Silfverskiold technique has increased gliding resistance compared with a simple running suture.

112
18) (OBQ12.15) A 30-year-old healthy female sustains a traumatic digit amputation while working at a factory. Which of the
following is a absolute indication for digit replantation in this patient? QID: 4375
A. Isolated amputation of the index finger proximal to the FDS insertion
B. Amputation due to crush injury
C. Replantation of an index finger with a segmental injury
D. Isolated amputation of the thumb proximal to the FPL insertion
E. Presentation 13 hours after the amputation occurred

PREFERRED RESPONSE ▼ 4
The unique functional role of the thumb in opposition and pinch dictates that it be replanted whenever possible in a healthy
patient, regardless of the level of amputation. The remainder of the answer choices are relative contraindications for digit
replantation.

Boulas et al outline indications and contraindications for digit replantation after traumatic amputation. Contraindications to
replantation include multilevel or segmental injury, a single digit proximal to the FDS insertion, a severe crush or mangling injury,
extreme contamination, prior impaired function, concomitant life-threatening injury, severe medical problems, anesthetic risk,
and major psychiatric disorder.

Waikakul et al determined the influencing factors of the immediate and late outcome of replantation and revascularization of
amputated digits. They found that the type of injury was the most important factor influencing immediate and late outcomes.
They also determined that connecting the profundus tendon stump of the proximal part to the superficialis tendon of the
amputated part gave a better result than two tendon repair and repairing only the profundus tendon.

Illustration A shows a clinical photo of an isolated amputation of the thumb proximal to the FPL insertion.

Incorrect Answers:
Answer 1: Replantation of a single digit proximal to the FDS insertion is associatedwith poor results related to the loss of PIP
joint motion due to flexor sheath adhesion formation.
Answer 2: Crush or mangling injury is associated with serious damage to tissues, which are at risk for infection, problematic
healing, and scarring, thereby contributing to a poor outcome.
Answer 3: Segmental injury to the index finger is a contraindication to replantation due to poor function post-operatively.
Answer 5: Prolonged warm ischemia time, defined as more than 12 hours, is associated with replantation failure.
Illustrations: A

------------------------------------------------------------------------------------------------------(168)-------------------------------------------------

113

You might also like